SóProvas



Questões de Sequências Lógicas de Números, Letras, Palavras e Figuras


ID
2092
Banca
FCC
Órgão
TRT - 24ª REGIÃO (MS)
Ano
2006
Provas
Disciplina
Raciocínio Lógico
Assuntos

A sentença seguinte é seguida de um número entre parênteses, que corresponde ao número de letras de uma palavra que se aplica à definição dada.

"Tudo aquilo que não é cópia ou imitação." (8)

A alternativa onde se encontra a letra inicial de tal palavra é

Alternativas
Comentários
  • Tudo aquilo que não eh cópia ou imitação eh

    O R I G I N A L !!!!(8 LETRAS)

    BONS ESTUDOS!!!
  • É ORIGINAL
    8 letras.
    Bons estudos ;)
  • Autêntico tem 9 letras, então não poderia ser.
  • KKKKK MEU DEUS AS PESSOAS COMENTARAM "ORIGINAL"E EU PENSEI QUE ERA "OURO" NÃO ME ATENTEI PARA O NÚMERO DE LETRAS E MESMO ASSIM MARQUEI A ALTERNATIVA QUE CONTINHA A LETRA O. RACIOCINEI ERRADO, MAS ACERTEI A QUESTÃO. KKK RINDO MUITO PQ EU CONHEÇO UMA FRASE QUE FALA SOBRE O OURO E É PARECIDA COM ESSA.

  • meu deus!

  • O R I G I N A L: negação de imitação.

  • É FGV não brinca mesmo.

  • Letra B

    É a definição da palavra OU que no raciocínio lógico significa EXCLUSÃO(8)

  • Pra quem está precisando da uma alavancada nos estudos recomendo esses SIMULADOS PARA PPMG, ajudou-me bastante!

    https://go.hotmart.com/V65499332X

    (FORÇA GUERREIROS, PPMG 2022)

  • Muito original a pergunta. Rsrs


ID
2095
Banca
FCC
Órgão
TRT - 24ª REGIÃO (MS)
Ano
2006
Provas
Disciplina
Raciocínio Lógico
Assuntos

Note que, dos pares de números seguintes, quatro têm uma característica comum.

(1;5) - (3;7) - (4;8) - (7;10) - (8;12)

O único par que não tem tal característica é

Alternativas
Comentários
  • Em cada par ordenado, a diferença entre y e x é sempre igual a 4. No par ordenado (7;10) não temos o padrão esperado.
  • É a diferença

    cada par tem 4 numeros de diferença menos (7;10)

    Então a resposta certa é a letra E

     

    Bons Estudos Pessoal, e Boa Sorte

    Paulo.

  • Cada sequencia estão representadas pelos dois numeros impares ou por dois numeros pares, exceto a sequencia (7;10) que está representada por um número ímpar e um número par.
  • Existem várias formas de resolver esse tipo de questão, por exemplo, eu percebi também que a quantidade de números entre os dois (7;10) é igual a 2 enquanto nos outros pares podemos encontrar 3 números. exe.: (1;5) igual a 3. 

  • (1;5) = 2, 3, 4,  

    (3;7) = 4, 5, 6

    (4;8) = 5, 6, 7

    (7;10) = 8 , 9  < GAB]

    (8;12) = 9, 10, 11

            Matei assim. 

  • Fiz de forma diferente dos colega abaixo, mas deu certo tbm kkkkk somei os números dentro dos parentes e o único resultado que deu ímpar foi (7;10).

    (1;5) = 6

    (3;7) = 10

    (4;8) = 12

    (7;10) = 17

    (8;12) = 20

  • Cada sequência tem ou 2 NÚMEROS PARES OU 2 ÍMPARES, EXCETO A OPÇÃO E) (7;10), que tem 1 ÍMPAR E UM PAR, tornando notória a DIFERENÇA das demais opções.

    Resposta: E) (7;10).

  • Eu matei assim:

    (1;5) = 6

    (3;7) = 10

    (4;8) = 12

    (7;10) = 17

    (8;12) = 20

  • (1;5) ambos impar

    (3;7) ambos impar

    (4;8) ambos par

    (7;10) ímpar e par ***

    (8;12)ambos par

  • Não sei se é a forma correta de resolver, mas os dois números ou eram números pares ou ímpares, sendo esta uma característica comum entre eles. O único par de números que não possuía esta característica era o (7;10), sendo 7 ímpar, 10 par. Acabou dando certo.

  • EU FIZ DIFERENTE EU REPAREI QUE TODOS TINHA RELACAO COM O 4 TIPO DO MENOR PARA O MAIOR

    ( 1;5 ) 1 PARA 5 FALTA 4

    ( 3;7 ) 3 PARA 7 FALTA 4

    ( 4;8 ) 4 PARA 8 FALTA 4

    ( 8;12 ) 8 PARA 12 FALTA 4

    ( 7; 10 ) 7 PARA 10 FALTA 3

  • eu pensei nos primos pq (7;10) um é primo o outro não, já nos outros pares, ou os dois são primos ou não.
  • Basta fazer a

    subtração :

    (1;5) 5-1=4

    (3;7) 7-3=4

    (4;8) 8-4=4

    (8;12) 12-8=4

    (7;10) 10-7=3 único resultado diferente.

  • 1+4=5

    3+4=7

    4+4=8

    7+3= 10 (opa, aqui não usou +4 e sim +3)

    8+4=12

  • Eu fiz um monte de análises loucas. (1;5) ímpares (3;7) ímpares (4;8) pares (8;12) pares (7;10) par e ímpar. Pelo jeito, acertei na sorte.
  • é que estão ímpares com ímpares e pares com pares

  • Fiz diferente: cada grupo são ambos os números ímpar ou par... o único grupo que tá diferente é o 7,10 porque tem número ímpar e par junto..

    Se o raciocínio tá certo? não sei.. mas deu pra acertar a questão!


ID
2101
Banca
FCC
Órgão
TRT - 24ª REGIÃO (MS)
Ano
2006
Provas
Disciplina
Raciocínio Lógico
Assuntos

Considerando que a ordem alfabética adotada é a oficial e exclui as letras K, W e Y, observe a relação existente entre o primeiro e o segundo grupos de letras mostrados no esquema seguinte:

LMNL : PQRP :: GHIG : ?

Se a mesma relação deve existir entre o terceiro grupo e o quarto, que está faltando, o grupo de letras que substituiria corretamente o ponto de interrogação é

Alternativas
Comentários
  • 1. As três primeiras letras do grupo exigem uma continuidade no próprio alfabeto, no entanto, com o salto sobre uma letra. Ex: GHI - pula o "J" - segue: LMN
    2. Depois é só repetir a 1ª letra do grupo "L".
  • Gostei muito das aulas. Sugeria que fosse realizada uma aula sobre prioridade.

  • Não entendi

  • entendi que a letra g que foi repetida no final das letras substitui o j então o próximo começa com a letra l

  • Gente, demorei uns 3 minutos até chegar a uma possível conclusão: fiz da seguinte maneira: LMNL deixo de fora (O) PQRP (s) deixo de fora, GHIG (j)(k) deixo de fora, e no final a próxima será: LMNL. OU SEJA, vai aumentando a as letras conforme a sequência... ou então estou viajando na maionese rsrs.

  • Não entendi

  • Eu fiz da seguinte forma:

    Para chegar do L ao P são: L + 4 letras

    L,M,N,O, P

    Diante disso somei na letra G + 4 letras, lembrando que exclui K.

    G,H,I,J,<K>,L

    Logo LMNL

  • PQ não poderia ser a letra D ou E?

  • cada grupo é formado pela sequência do alfabeto, excluindo o 4o elemento e repetindo o 1o. o segundo grupo é uma continuação do primeiro, pulando a letra excluída.

    LMN(O)L --> PQR(S)P

    logo se pede a sequência de GHIG, só seguir o alfabeto

    GHI(J)G --> (pula o K) LMN(O)L

  • varias lógicas darão vários resultados, nessas questões o importante é acertar a lógica que a pessoa usou! ou seja adivinhar qual linha a pessoa usou, temos várias linhas , é questão de sorte também!

  • Pra quem está precisando da uma alavancada nos estudos recomendo esses SIMULADOS PARA PPMG, ajudou-me bastante!

    https://go.hotmart.com/V65499332X

    (FORÇA GUERREIROS, PPMG 2022)


ID
3655
Banca
FCC
Órgão
TRF - 4ª REGIÃO
Ano
2007
Provas
Disciplina
Raciocínio Lógico
Assuntos

Observe que, no esquema abaixo as letras que compõem os dois primeiros grupos foram dispostas segundo determinado padrão. Esse mesmo padrão deve existir entre o terceiro grupo e o quarto, que está faltando.

ZUVX : TQRS : : HEFG : ?

Considerando que a ordem alfabética adotada, que é a oficial, exclui as letras K, W e Y, o grupo de letras que substitui corretamente o ponto de interrogação é

Alternativas
Comentários
  • Siga a sequência: deslocando a 1ª letra para últ: UVX Z = ZUVX
    QRS T = TQRS
    agora a outra sequência: EFG H = HEFG
    A PRÓXIMA FICA: ABC D = DABC
  • RACIOCÍNIO LÓGICO - LÓGICA SEQUENCIALSE O ALFABETO É O OFICIAL ENTÃO(ABCD):(EFGH)::(IJLM):(NOP)::(QRST):(UVXZ)NOTE QUE EXISTE UM SIMBOLO ":" (ESTA PARA)SE A QUESTÃO É DE LÓGICA SEQUENCIAL TEREMOS QUEZUVX : TQRS :: HEFG : DABC
  • Z - U - V - X     T - Q - R - S      H - E - F - G        D - A - B - C          Enumerar as letras e seguir a sequência 4123 
    23-20-21-22    19-16-17-18     8 -5 -6 -7             4 - 1 - 2 - 3

  • Eu diria o seguinte: não basta inverter a ordem da última letra na sequência, pois que IFGH;QNOP;FCDE também obedecem a essa regra.

    Assim, certamente o padrão é inverter a última letra da sequência, porém a sequência tem que possuir letras que não se repetem nas outras sequências já transcritas. Vê-se que não há letras repetidas, sendo assim, a única resposta possível é D A B C que obdece à regra da sequência e da não repetição.

  • BONITA

  • O bizu da questão é entender que a letra da alternativa correta não pode estar em nenhuma das sequências anteriores.


ID
4534
Banca
FCC
Órgão
TRE-MS
Ano
2007
Provas
Disciplina
Raciocínio Lógico
Assuntos

A sucessão dos números naturais pares é escrita sem que os algarismos sejam separados, ou seja, da seguinte forma:

0 2 4 6 8 1 0 1 2 1 4 1 6 1 8 2 0 2 2 2 4 2 6 2 8 ...

Nessa sucessão, o algarismo que deve ocupar a 127ª posição é o

Alternativas
Comentários
  • 0 2 4 6 8 => 5 números e 5 algarimos

    10 12 ... 18 => 5 números e 10 algarimos


    Até agora: 15

    Faltam: 127 - 15 = 112

    20 22 ... 28

    ............

    90 92 ... 98

    -------------

    8* 10 = 80 algarismos

    Total até agora: 80 + 15 = 95 algarismos

    Faltam: 127 - 95 = 32

    Note:

    100 102 ... 108 => 5 números e 15 algarismos

    110 112 ... 118 => 5 números e 15 algarismos

    Temos mais 30 algarismos, então. Até agora são 125 números na seqüência inicial.

    Faltam 2 algarismos. Digitando 120 (que virá depois do 118), vê-se que o "2" ocupará a 127ª posição.

  • Muito boa a explicação a baixo, mas vamos simplificar ainda mais:

    com 1 algarismo = temos 5 (de 0 a 8)
    com 2 algarismo = temos 90 (de 10 a 98)
    com 3 algarismo = temos 127 - 95 = 32 (de 100 ao número que pretendemos descobrir)

    Agora, temos que:

    100 a 108 = temos 15
    110 a 118 = temos 15, entamos já temos 30 (15 + 15)

    Pra 32 faltam duas casas. Como o último número usado foi 118, em seguida temos 120, onde o algarismo 2 ocupa a 127ª casa.

    RESPOSTA = "B".
  • Bom, eu fiz assim. 0 2 4 6 8 1 0 1 2 1 4 1 6 1 8 2 0 2 2 2 4 2 6 2 8 ... Veja que após esses cinco números, começa certas repetições de 0 a 8. De 10 a 18. De 20 a 28. Sempre em números pares, ou seja, dentro dessa sequencia há sempre 5 números que são pares e fazem parte da sequencia.
    Eu peguei o 127 e dividi por 5. Deu 25 com resto 2. Logo a resposta é resto da divisão. Resposta: letra B.
    Obs.: esse dois não representa o número dois propriamente dito, mas sim o segundo número dessa sequencia. O segundo número é dois.



  • Acredito que a solução da divisão por 5 foi ao acaso., porque o algarismo que é buscado é posicional,  e se for maior do que 99 por exemplo será necessário 3 algarismos para representar um número lógico.
  • 0 2 4 6 8 1 0 1 2 1 4 1 6 1 8 2 0 2 2 2 4 2 6 2 8 3 0 3 2 3 4 3 6 3 8 4 0 4 2 4 4 4 6 4 8 5 0 5 2 5 4 5 6 5 8 6 0 6 2 6 4 6 6 6 8 7 0 7 2 7 4 7 6 7 8 8 0 8 2 8 4 8 6 8 8 9 0 9 2 9 4 9 6 9 8 1 0 0 1 0 2 1 0 4 1 0 6 1 0 8 1 1 0 1 1 2 1 1 4 1 1 6 1 1 8 1 2 0 ( 2 = 127º)

    É só observar que apesar de estarem separados os algarismo, mas a sequência é sempre um númro par = 98, 100, 102, 104 ...

    ENTÃO:

    Temos 5 números de 1 algarismo = 5 algarismos
    Temos 45 números pares de 2 algarismo = 90 algarismos
    Então 127 - 95 = 32 algarismos 
    Como agora os números ainda pares são de 3 algarismos = 32 / 3 = 10 RESTO 2 (OU 2 ALGARISMOS)
    100, 102, 104, 106, 108, 110, 112, 114, 116, 118 (TOTAL 10 NÚMEROS PARES)

    próximo número par é 1 2 0 ( 1 PRIMEIRO ALGARISMO E O 2 SEGUNDO ALGARISMO DO RESTO DA DIVISÃO.)

    PORTANTO, 2 POSIÇÃO 127º.
  • O nº 28  está na 25ª casa. Então 127/25 =5 e restam 2.
    28x5=140, então este estará na posição 125. 
    Como quero a posição 127, somo 125 ao 2 que sobrou no início e encontro a 127ª posição que será 140+2.
    Portanto será o 2. letra B


  • 0 2 4 6 8 1 0 1 2 1 4 1 6 1 8 2 0 2 2 2 4 2 6 2 8 ...

    0 2 4 6 8                                                    5

    1 0 1 2 1 4 1 6 1 8                                    1 0

    2 0 2 2 2 4 2 6 2 8                                    1 0

    3 0 3 2 3 4 3 6 3 8                                    1 0

    4 0 4 2 4 4 4 6 4 8                                    1 0

    5 0 5 2 5 4 5 6 5 8                                    1 0  

    6 0 6 2 6 4 6 6 6 8                                    1 0

    7 0 7 2 7 4 7 6 7 8                                    1 0

    8 0 8 2 8 4 8 6 8 8                                    1 0

    9 0 9 2 9 4 9 6 9 8                                    1 0

    1 0 0 1 0 2 1 0 4 1 0 6 1 0 8                     1 5

    1 1 0 1 1 2 1 1 4 1 1 6 1 1 8                     1 5

    1 2     ( 2 = 127º)                                      = 125

  • Eu resolvi da seguinte forma, como os numeros são pares eu pensei os primeiros 50 numeros vai chegar ate o numeral 100, mais 50 numeros vai passar mais 100, ou seja, o numeral 200, então já foram 100 posições, só faltam 27, então eu deduzi que ficaria na casa do 200 então o numero seria 2, mas vou ser bem sincero, fiz isso de cabeça e não tinha a certeza que iria acerta a questão.

  • Gabarito: B

    Observe que se repetem os números 0 2 4 6 8 em toda a sequência

    10 12 14 16 18 etc.....

    com isso, basta DIVIDIR a posição 127 / 5

    127 l. 5.

    27. 25

    (2) O resto equivale ao algarismo 2


ID
11299
Banca
FCC
Órgão
TRF - 3ª REGIÃO
Ano
2007
Provas
Disciplina
Raciocínio Lógico
Assuntos

Em relação à disposição numérica seguinte, assinale a alternativa que preenche a vaga assinalada pela interrogação:

2     8     5     6     8     ?     11

Alternativas
Comentários
  • Questãozinha chata!

    (2)+6(8)-3(5)+1(6)+2(8)-4(4)+7(11)

    Aqui está a lógica!
    +6-3 = 3
    -3+1 = -2
    +1+2 = 3
    +2-4 = -2
    -4+7 = 3

  • Analisando os números intercaladamente (número sim, número não) temos as sequências:

    2 5 8 11 (foi somando 3)
    8 6 ? (foi diminuindo 2) - portanto o próximo é o 4
  • Raciocinei pensando que o examinador estava querendo saber como fazer para encontar o nº seguinte, dessa forma teremos o nº 3. Alternativa "C". Teremos(2+6)=8; (5+1)=6; (8+3)=11.Como é outra forma de resolver e tem outra alternativa como resposta, acho que a questão teria de ser anulada.
  • galera é o seguinte, não olhem para os numeros um a um
    a sequencia é a seguinte:
    -----
    28 56 84 11... o resto dos numeros seriam 112 140 168...
    -----
    reparem que tomando por base o numero 28, todos os outros são multiplos dele, primeiro x2 = 56. depois x3 = 84.... e assim por diante, pensei em olhar por numeros maiores por causa das duas ultimas alternativas que são valores altos.
  • NÃO JULGUEM A QUESTÃO, POIS ELA ESTÁ LEGALMENTE APLICADA, ENVOLVENDO NÚMEROS NATURAIS E INTEIROS CONFORMA AS RESPOSTASO EXAMINADOR REQUER QUE VOCÊ DESCUBRA O NUMÉRO COMUM QUE MAIS APAREÇA NOS PRODUTOS DAS OPERAÇÕES ADIÇÃO, SUBTRAÇÃO, DIVISÃO E MULTIPLICAÇÃOADIÇÃO = 40SUBTRAÇÃO = -36MULTIPLICAÇÃO 42240DIVISÃO = 9,46EM RELAÇÃO À DISPOSIÇÃO, E QUEM ESTÁ MAIS DISPOSTO É O 4
  • Na verdade a lógica desta sequencia está na soma dos número consecutivos:2 8 5 6 8 ? 112+8= 108+5= 135+6= 116+8= 148+X= 12 (X=4)x+11= 15 Veja que o resultado começando do 10 é sempre +3 e depois -2 (10 +3 = 13, 13-2= 11, 11+3=14 ...)
  • muito chata essa questão, mas fiz assim...

    somei os termos:

    2 + 8 = 10

    5 + 6 = 11

    8 + 4 = 12

    obs.: 2 + 3 = 5; 5 + 3 = 8 ( + 3 )

    8 - 2 = 6; 6 - 2 = 4  ( - 2 )

    10 + 1 = 11, 11 + 1 = 12 ( + 1 )

     

  • Continuei sem entender..cada um falou uma coisa....snif
  • Olá, pessoal!
    Alternativa b
    Veja se isso ajuda:
    1ª Posição → 2
    2ª Posição → 8
    3ª Posição → 5
    4ª Posição → 6
    5ª Posição → 8
    6ª Posição → ?
    7ª Posição → 11
    Para posições ímpares somamos 3 unidades para obter a próxima; já para as pares, subtraímos 2 unidades. Então:
    6ª posição = 4ª posição – 2
    6ª posição = 6 – 2
    Portanto, temos:

    2          8          5         6          8          4          11
    Bons Estudos!
  • Achei a sequência muito curta para poder identificar um padrão. Fiquei pensando se o 8 intercalava a sequência, ou seja, se depois do 11 viria outro 8. No caso da soma de 3 para os números impares havia 3 sequências, o que já era suficiente para identificar o padrão. Mas no caso dos números pares, onde subtraía-se 2, havia somente 1 caso, não deixando claro que esse era o padrão, pois não tem como identificar um padrão com apenas um caso (o 2º caso era justamente a incógnita). Questão feita para ninguém acertar. Àqueles que perceberam a lógica da sequência, parabéns!!       

  • Analise por posição: 2 8 6 8 ? 11

    Negrito você soma 3
    Sublinhado você subtrai 2
    Pronto, acabou!

    Obs: É tão fácil que é de se envergonhar, fala ai?! kk..

  • GABARITO: B

     

     

    2,8,5,6,8, ? ,11

     

    Duas sequências

    - 2, 5, 8, 11. --> Progressiva de 3

    -8,6, ?          -->  Decresciva de 2

     

    O ponto de interrogação é o número 4.

     

     

  • Resposta: b) 4

    2   8   5   6   8   ?    11

    28+28=56 (2  8) (5  6)

    56+28=84 (2  8) (5  6) (8  4)

    84+28=112 (2 8) (5 6) (8 4) (11..)

     

  • O ruim é que em questões antigas não existem comentários dos professores.

    Tinham que ter obrigatoriamente vídeos explicando cada questão.

    Mas muitas questões só tem comentários escritos, ai não da pra entender

  • Questão de sequência dupla: 2 8 5 6 8 ? 11 ou seja pega o 2 5 8 11 ( 2 +3 =5 / 5+3=8 / 8+3=11) agora pega 8 6 ? ( 8 -2= 6 / 6 -2= 4) ? =4
  • 28 56 8? 11

    28+28 = 56

    56+28 =84 4=?

    84+28=112 Esse seria o próximo número.

  • Coloca em pé e vê a diferença

    +3 2 8 - 2

    +3 5 6 - 2

    +3 8 (4) - 2

    11 (2)

    em vermelho foi acrescentando +3 e em azul foi diminuindo -2


ID
12532
Banca
FCC
Órgão
TRF - 2ª REGIÃO
Ano
2007
Provas
Disciplina
Raciocínio Lógico
Assuntos

Considere que a seqüência (C, E, G, F, H, J, I, L, N, M, O, Q, ...) foi formada a partir de certo critério. Se o alfabeto usado é o oficial, que tem 23 letras, então, de acordo com esse critério, a próxima letra dessa seqüência deve ser

Alternativas
Comentários
  • Primeiro, troquemos as letras pelas respectivas posições no alfabeto. É uma forma interessante de se começar:

    (C, E, G, F, H, J, I, L, N, M, O, Q, ...)

    (3,5,7,6,8,10,9,11,13,12,14,16,...)

    Repare que podemos dividir nos seguintes blocos interessantes:

    3,5,7

    6,8,10

    9,11,13

    12,14,16 ...

    Nos blocos acima, o elemento central é média aritimética dos demais elementos. Ainda, repare que, FIXADA A POSIÇÃO DE UM ELEMENTO NUM GRUPO,OU SEJA, SE É O CENTRAL, O DA DIREITA OU O DA ESQUERDA (tomando como referência o central), passa-se para o grupo imediatamente posterior, somando 3 a todos os elementos do grupo de partida.

    Dito isso, o próximo grupo será: 15 17 19.

    Pronto. Que letra ocupa a 15ª posição no alfabeto?

    Resposta: P.

    Ufa...
  • 1) Enumerar as letras destacadas de acordo com a posição do alfabeto.
    2) Perceba que temos uma fileira impar outra par
    3, 5, 7 (C,E,G)
    6, 8, 10 (F, H,J)
    9, 11, 13 (I, L, N)
    12, 14,16 (M, O, Q)
    15,(P)
  • Olá galera encontrei uma solução diferente, embora não tanto ortodoxa, mas consider outra forma de ver o problema e resolvê-lo. Fiz o seguinte:
    1ª possibilidade é:
    1º . numerei de 1 a 12 conforme os numeros apresentados ex. C é o1º a parecer, E é 2º, G é o 3º e assim sucessivamente.
    2º. 2 X ele mesmo é = 4
    3º. 4 – 1 = 3
    4º. 3+2 = 5
    5º. 5+2=7
    6º. 7- 1= 8
    Daí sabe-se que a partir da letra G a sequüência é: +2, +2, -1
    Logo é Letra P cuja seqüência vai ser -1

    2ª possibilidade:
    A partir da letra g temos um bloco de 3 ímpares, e 3 pares
    A letra P vai representar o nº 13, já que o Q está no bloco dos numerous pares e será 12.

    Tipo da questão que talvez possa ser anulada, uma vez que apresenta duas soluções.
  • A forma que acho mais fácil de fazer este tipo de questão é escrever o alfabeto e ir desenhando o padrão apresentado em cima dele:A B C D E F G H I J L M N O P Q R S T U V X ZComeça na letra C, vc marca elaA B C D E F G H I J L M N O P Q R S T U V X Za segunda é E, vc marca que andaram 2 letras pra chegarA B C D E F G H I J L M N O P Q R S T U V X Zandou mais duasA B C D E F G H I J L M N O P Q R S T U V X ZVoltou umaA B C D E F G H I J L M N O P Q R S T U V X ZAndou duasA B C D E F G H I J L M N O P Q R S T U V X ZAndou duasA B C D E F G H I J L M N O P Q R S T U V X ZVoltou umaPronto o padrão está formado(+ 2, + 2, - 1 , + 2, +2, - 1 ...), aqui é mais complicado para explicar, mas desenhando isso na prova, vc mata a questão em segundos.
  • è muito fácil.

    C,E,G,F,H,J,I,L,N,N,O,Q = Vamos começar pela a letra E, dessa letra sempre vai pular uma pra formar o alfbeto (E, o G pula ,F aí depois volta pro G)

    è nessa sequência que eu descobri o resultado

    Quando Chega na letra O depois do O vem o P e depois do P volta pro Q que é o anterior

     

    Resposta Letra A

    Bons Estudos Pessoal

    Espero ter Ajudado

     

    Paulo.

     

  • Achei a resposta assim:

    C E   G   F   H   J   I   L   N   M   O   Q...

     2   2   -1   2   2   -1   3   2   -1   2   2

    De C para E, notem que a diferença no alfabeto é de 2.
    De E para G, notem que a diferença é de 2 também.
    De G para F, notem que a letra F seria anterior a letra G, então representei ela por -1.

    E assim por diante...a sequência ficou assim: 2   2   -1   2   2   -1   3   2   -1   2   2

    Portanto, após o (2, 2) vem o (-1), ou seja, a letra anterior a Q = P

  • C,E,G,F,H,J,I,L,N,M,O,Q,...

    C,d,E,F,G(foram invertidos),H(permanece na posição correta),I,J(foram invertidos),L(permanece na posição correta),M,N(foram invertidos),O(permanece na posição correta),P,Q(foram invertidos),...
  • Anda 2 ímpares

    volta1

    Anda 2 pares

    volta1

    A B C D E F G H I J L M N O P

    1 2 3 4 5 6 7 8 9 10 11 12 13 14 15

    3... 5, 7, volta um

    6...8, 10  volta um

    9... 11, 13  volta um

    12... 14, 16  volta um

    15

     

  • volta 1, pula 2. Dica mortal !


ID
17815
Banca
CESGRANRIO
Órgão
BNDES
Ano
2008
Provas
Disciplina
Raciocínio Lógico
Assuntos

A expressão

(NOT A AND B) OR ((B AND NOT A) OR B)

equivale a

Alternativas
Comentários
  • A B Resultado
    0 0 0
    1 0 0
    0 1 1
    1 1 1

    Valor de Resultado é sempre igual ao valor de B
  • Construindo a tabela verdade, temos:(V é OR e ^ é AND)(~A ^ B) V [(B ^ ~A) V B ]A B (~A ^B) ((B^~A) V B) ResultadoV V F V VV F F F FF V V V VF F F F FVisto que o resultado é sempre o mesmo que B, logo pode-se trocar toda a expressão por B, sendo essa a resposta da questão.
  • A questão pode ser resolvida através do uso da tabela-verdade para o problema, ficando da seguinte formaA B ~A ~A . B B . ~A (B . ~A) + B (~A . B) + ((B . ~A) + B)0 0 1 0 0 0 00 1 1 1 1 1 11 0 0 0 0 0 01 1 0 0 0 1 1Apartir da tabela podemos ver que a alternativa correta é a B.
  • Outra forma de resolver a questão é simplificar a expressão. 

    Notação: 
    • ~ --> NOT
    • . --> AND
    • + --> OR

    (~A . B) + ((B . ~A) + B) = (~A . B) + (~A . B + B) = (~A . B) + (B . (~A + 1)) = (~A . B) + (B . 1) = (~A . B) + (B) = B . (~A + 1) = B . 1 = B

    Resposta: B

    Obs.: Foram feitas as seguintes simplificações:
    • ~A . B + B = B . (~A + 1)
    • ~A + 1 = 1
  • O comentário acima está equivocado, e ter chegado na resposta com ele pode-se dizer que foi coincidência.
    Da forma que foi exposta, houve violação da lei distributiva:
    A x (B + C) = A x B + A x C

    o trecho ((B x ¬A) + B) não pode ser convertido em  (¬A x B + B) >  (B . (~A + 1)) = (~A . B) + (B . 1) 

    O que pode ser aplicado neste trecho é a propriedade da absorção: A + (A x B) = A
    com isso teríamos: ((B x ¬A) + B) = B

    resultando

    (¬A x B) + (B + B)
    (¬A x B) + B

    Aplicando novamente a absorção, resta:

    B
  • Não entendi nada.

    De onde está saindo 0,1 ?

  • Resolvi a questão pela tabela-verdade.

    Assuma :

    Not= ~

    OR= ou


ID
19066
Banca
FCC
Órgão
TRF - 4ª REGIÃO
Ano
2007
Provas
Disciplina
Raciocínio Lógico
Assuntos

Note que, em cada um dos dois primeiros pares de palavras dadas, a palavra da direita foi formada a partir da palavra da esquerda segundo um determinado critério.

       acatei - teia
       assumir - iras
       moradia - ?

Se o mesmo critério for usado para completar a terceira linha, a palavra que substituirá corretamente o ponto de interrogação é

Alternativas
Comentários
  • ACATEI - TEIA = 3 ÚLTIMAS LETRAS + A PRIMEIRA
    ASSUMIR - IRAS = 2 ÚLTIMAS LETRAS + AS 2 PRIMEIRAS
    MORADIA - amor = A ÚLTIMA LETRA = AS 3 PRIMEIRAS

ID
19069
Banca
FCC
Órgão
TRF - 4ª REGIÃO
Ano
2007
Provas
Disciplina
Raciocínio Lógico
Assuntos

Considere que os termos da sucessão (0, 1, 3, 4, 12, 13, ...) obedecem a uma lei de formação. Somando o oitavo e o décimo termos dessa sucessão obtém-se um número compreendido entre

Alternativas
Comentários
  • 0,1,3,4,12,13,...39, 40,120, 121

    1+3=4
    4*3=12
    12+1=13
    13*3=39
    39+1=40
    40*3=120
    120+1=121

    Então temos a soma do 8º e 10º, 40+121=161 resposta A
  • A lógica do exercício é são duas, a saber:

    Usa-se um número e seu subseqüente, como demonstrado abaixo: 0 e 1, 3 e 4, 12 e 13, 39 e 40, 120 e 121;

    Para chegarmos à próxima seqüência, multiplicar-se-á o segundo número de cada grupo por dois, somando-se o resultado com o próprio número multiplicado.

    0 e 1 ; 1 x 2 = 2 ; 2 + 1 = 3

    3 e 4 ; 4 x 2 = 8 ; 8 + 4 = 12

    12 e 13 ; 13 x 2 = 26 ; 26 + 13 = 39

    39 e 40 ; 40 x 2 = 80 ; 80 + 40 = 120

    120 e 121 ; 40 + 121 = 161

    Gabarito: A
  • 0, 1 (0+1) pular 1 número3, 4 (3+4) pular 7 números12, 13 (12+13) pular 25 números39, 40 (39+40) pular 79 números120,121(0, 1, 3, 4, 12, 13, 39, 40, 120, 121, ...)oitavo número: 40décimo número: 121Total: 161Alternativa A
  • Começando do 0 você vai sempre somar 1, multiplicar 3, somar 1, multiplicar 3...0 + 1 = 11 * 3 = 33 + 1 = 44 * 3 = 1212 + 1 = 1313 * 3 = 3939 + 1 = 4040 * 3 = 120120 + 1 = 121Simples assim...
  • Como cada termo de ordem par é igual ao seu antecessor acrescido de uma unidade, basta que determinemos uma lógica de formação dos números que ocupam as posições ímpares, a partir do 3º número, ou seja:primeiro número = 0 ............................... terceiro número = (3^1) = 3 ....................... quinto número = (3^1)+(3^2) = 12 ................sétimo número = (3^1)+(3^2)+(3^3) = 39 ..........nono número = (3^1)+(3^2)+(3^3)+(3^4) = 120 ... segundo número = 0 + 1 = 1quarto número = 3 + 1 = 4sexto número = 12 + 1 = 13oitavo número = 39 + 1 = 40décimo número = 120 + 1 = 121Portanto, a soma do oitavo com o décimo termos é igual a (40 + 121 = 161).Obs: (3^2) entende-se como 3 elevado a 2.
  • Simplificando...

    A sequência lógica é:

    +1, *3, +1, *3 ...

    Se vc seguir isso vai ver que 13x3=39 (7o termo), 39+1=40(8otermo) e assim por diante até chegar no 10o (121) e pronto!
  • Encontrei uma outra forma de resolver a questão.

    observando a sucessão (0, 1, 3, 4, 12, 13 ...)

    percebe-se que ela é formada pelos pares de sequencias 0 e 1, 3 e 4, 12 e 13. Isto é um padrão.
    Resta saber, então, qual é o padrão de sequencia entre estes pares. Isto é, por que de 1 pula para 3, e de 4 pula para 12.

    veja que entre 1 e 3 há um único número que é 2. Isto é, a quantidade de números entre 1 e 3 é 1.
    Assim, se somarmos 1 + (1... que nada mais é do que a quantidade de números entre 1 e 3), teremos 2 como resultado. Logo, o próximo número será 3.

    Por este padrão, acharemos a próxima sequencia...
    assim, veja que entre 4 e 12 a quantidade de números que existem é 7. Basta contar... 5, 6, 7, 8, 9, 10, 11. Ou seja, 7 números estão compreendidos entre 4 e 12.

    Veja também que essa quantidade de números que existe entre os pares é igual à soma dos dois números anteriores.
    Pois, 3 + 4 = 7.
    assim, 4 + 7 = 11. Logo, o próximo número é 12. O que de fato ocorre.

    Neste sentido, o próximo número da sequencia de pares 12, 13 é = 
    12 + 13 nos revela a quantidade de números que estão compreendidos, que é 25.
    Assim, 13 + 25 = 38. Logo, na próxima sequencia de pares o próximo número é 39, 40.

    Agora, fazendo a mesma coisa...
    39 + 40 = 79.
    40 + 79 = 119. Logo, na próxima sequencia de pares os números serão 120, 121.

    colocando agora na sucessão dada, (0, 1, 3, 4, 12, 13, 39, 40, 120, 121...)

    8º número + 10º número = 40 + 121 = 161... que está no intervalo da LETRA A (150 - 170)


    Deus seja louvado!


  • Sequência: +1 e em seguida *3
    0+1= 1
    1*3= 3
    3+1= 4
    4*3= 12
    12+1= 13
    13*3= 39
    39+1= 40
    40*3= 120
    120+1= 121
    oitavo + décimo -> 40+121= 161
  • Não entendi essa


ID
21892
Banca
CESPE / CEBRASPE
Órgão
Banco do Brasil
Ano
2003
Provas
Disciplina
Raciocínio Lógico
Assuntos

Um banco oferece um sistema de aposentadoria privada em que cada participante faz um depósito mensal correspondente a x% do seu salário por um período de 30 anos, realizando o primeiro depósito 1 mês após o ingresso no sistema e totalizando 360 depósitos. Nesse sistema, todo o montante recolhido é corrigido a uma taxa de juros compostos de t% ao mês. Considere que os salários dos participantes desse sistema são mantidos constantes durante todos os anos de contribuição e que, um mês depois de ter efetuado o 360.º depósito, quando da sua aposentadoria, o participante passa a receber, todos os meses, uma pensão igual ao salário S que ele possuía, a qual é descontada do montante que ele tem aplicado no sistema. Considere ainda que, para cada participante, o montante residual que ele possui no sistema após o pagamento de n pensões - R(n), n > 0 - continua a ser corrigido pela mesma taxa de juros t% após a sua aposentadoria. Sabendo que R(0) é igual ao montante acumulado e corrigido no momento do 360o depósito, julgue os itens a seguir, relativos ao sistema de aposentadoria descrito e à previdência social brasileira, tendo como base as informações apresentadas.

A seqüência numérica correspondente aos depósitos mensais de um participante desse sistema de aposentadoria durante 5 anos é, ao mesmo tempo, uma progressão aritmética e geométrica.

Alternativas
Comentários
  • É aritmética pois o participante adiciona uma valor mensalmente.É geométrica pois o montante é corrigido mensalmente.
  • O salário são mantidos constantes.

    São feitos depósitos de x% do salário (constante).

    Ex: Salário R$ 1.000, e depósitos de 10% --> depósitos mensais de R$ 100.

    A questão pede a sequência numérica em relação aos depósitos e não ao saldo após os depósitos. 

    Depósitos     100  100  100  100

    Deveria ser Errada. Não há PA nem PG.

  • PA: 100,200,300,400,500...

    PG: 100 + x%, (100 + x%) +x% ... (Juros sobre juros: Juros compostos = formam uma PG)

  • GABARITO: C

  • Pode ser uma PG constante com razão igual a 1 ou uma PA constante com razão igual a 0. Gabarito: CERTO.

  • Questão de Primário

    PA para cada depósito de X%

    PG para acréscimo de juros t%


ID
21910
Banca
CESPE / CEBRASPE
Órgão
Banco do Brasil
Ano
2003
Provas
Disciplina
Raciocínio Lógico
Assuntos

Um banco oferece um sistema de aposentadoria privada em que cada participante faz um depósito mensal correspondente a x% do seu salário por um período de 30 anos, realizando o primeiro depósito 1 mês após o ingresso no sistema e totalizando 360 depósitos. Nesse sistema, todo o montante recolhido é corrigido a uma taxa de juros compostos de t% ao mês. Considere que os salários dos participantes desse sistema são mantidos constantes durante todos os anos de contribuição e que, um mês depois de ter efetuado o 360.º depósito, quando da sua aposentadoria, o participante passa a receber, todos os meses, uma pensão igual ao salário S que ele possuía, a qual é descontada do montante que ele tem aplicado no sistema. Considere ainda que, para cada participante, o montante residual que ele possui no sistema após o pagamento de n pensões - R(n), n > 0 - continua a ser corrigido pela mesma taxa de juros t% após a sua aposentadoria. Sabendo que R(0) é igual ao montante acumulado e corrigido no momento do 360o depósito, julgue os itens a seguir, relativos ao sistema de aposentadoria descrito e à previdência social brasileira, tendo como base as informações apresentadas.

R(n) é uma progressão aritmética de razão negativa e igual a -S.

Alternativas
Comentários
  • Para quem não tem acesso a resposta:

    Gaba: Errado

  • Não é (-S), apesar de todo mês após a ultima parcela, ele começar a descontar o salário que ele ganhava, nesse tempo de deposito, correram juros e ainda continuam correndo juros com o montante que vai restando a cada mês. Por isso R(n) não será uma PA de razão -S, pois há outros valores acrescidos.


ID
22204
Banca
CESPE / CEBRASPE
Órgão
Banco do Brasil
Ano
2003
Provas
Disciplina
Raciocínio Lógico
Assuntos

Texto V - questões 13 e 14

Preparando-se para custear as despesas com a educação dos seus filhos, Carlos decidiu abrir uma poupança programada para 120 meses de duração, com rendimento mensal de 1%, em que os depósitos devem ser feitos no primeiro dia de cada mês. O valor d(k), em reais, do depósito a ser efetuado nessa poupança no k-ésimo mês obedece às seguintes regras:

. d(k) = 100, para k = 1, 2, ... , 12;
. d(k + 12) - d(k) = 100, para k > 1.

Com base nas informações do texto V, julgue os itens abaixo.

Para k1 = 3, se k1, k2, ..., k10 estão, nessa ordem, em progressão aritmética crescente de razão 13, então os valores d(k1), d(k2), ..., d(k10) estão, nessa ordem, em uma progressão aritmética de razão 100.

Alternativas
Comentários
  • RESOLUÇÃO: (Apresentada pelo Professor Vilson Cortez, com adaptações)5) Sendo k1 = 3, se, então os valores d(k1), d(k2), ..., d(k10) estão, nessa ordem, em uma progressão aritmética de razão 100, Se k1, k2, ..., k10 estão, nessa ordem, em progressão aritmética crescente de razão 13, tem-se: Equacionando, temos: tem-se: d(k2) = d(k1 + 13) – d(k1) = 100k1 = 03 logo d(k1) = d(03) = 100k2 = 03 + 13 = 16 logo d(k2) = d(16) – d(03) = 100 = d(16) = 100 + d(03) = 200k3 = 16 + 13 = 29 logo d(k3) = d(29) – d(16) = 100 = d(29) = 100 + d(16) = 300k4 = 29 + 13 = 42 logo d(k4) = d(42) – d(29) = 100 = d(42) = 100 + d(29) = 400k5 = 42 + 13 = 55 logo d(k5) = d(55) – d(42) = 100 = d(55) = 100 + d(42) = 500k6 = 55 + 13 = 68 logo d(k6) = d(68) – d(55) = 100 = d(68) = 100 + d(55) = 600k7 = 68 + 13 = 81 logo d(k7) = d(81) – d(68) = 100 = d(81) = 100 + d(68) = 700k8 = 81 + 13 = 94 logo d(k8) = d(94) – d(81) = 100 = d(94) = 100 + d(81) = 800k9 = 94 + 13 = 107 logo d(k9) = d(107) – d(94) = 100 = d(107) = 100 + d(94) = 900k10 = 107 + 13 = 120 logo d(k10) = d(120) – d(107) = 100 = d(120) = 100 + d(107) = 1000Então os valores d(k1), d(k2), ..., d(k10) ou 100, 200, 300, 400, 500, 600, 700, 800, 900, 1000, estão, nessa ordem, em uma progressão aritmética de razão igual a 100.Alternativa CORRETA
  • Agradeço o colega pelo esforço em tentar nos explicar a questão, pois ele é um dos poucos. Eu juro que estou tentando entender a explicação, mas simplesmente nao consigo acompanhar o raciocínio. Esta seria mais uma questão em que eu chutaria ou deixaria em branco.

    Infelizmente as questões de raciocínio lógico, nas quais eu encontro mais dificuldade, são as que têm menos comentários (não sei se a maioria das pessoas se encontra na mesma situação que a minha). Por isso, mesmo sem entender a explicação do colega, eu não poderia dar poucas estrelas como outros deram, pois sou agradecido apenas pela sua intenção.  Penso que para dar poucas estrelas eu deveria elaborar um comentário melhor.

    Será que alguém poderia fornecer uma resolução mais didática da questão?

    Lanço um apelo a todos colegas que sao bons em matemática: POR FAVOR, COMENTEM NAS QUESTÕES DE RACIOCÍNIO LÓGICO!!!(se possível de modo compreensível para leigos como eu)

    Bons estudos a todos concurseiros!!

  • Buonasera!

    Questão um pouco trabalhosa, mas fácil. Amigos, nesse tipo de questão devemos primeiro manter a calma, depois só fazer sunstituições. Vamos lá??

    Para saber a questão é necessário saber o que é uma P.A. (progressão aritmética): é uma sequência numérica em que cada termo, a partir do segundo, é igual à soma do termo anterior com uma constante (constante = razão = 13). Sendo assim, teremos:

    K1=3

    K2=16

    K3=29

    K4=42 

    E assim por diante ...

    A questão quer saber quem é d(K1), d(K2), ... , e a razão desses valores. Então vamos lá:

    Quem é d(K1)?

    Já que k1=3, então substituindo teremos d(3). E quem é d(3) agora? d(3) foi dado no início da questão que era 100 (pois para valores de k=1, k=2, k=3, até k=12, d(k) será 100, ou seja, d(1)=d(2)=d(3)=d(4)=d(5)=d(6)=d(7)=d(8)=d(9)=d(10)=d(11)=d(12)=100). Sendo assim, d(k1) = d(3) = 100.

    Nosso próximo passo é achar d(k2). Fazendo a simples substituição teremos: d(k2) = d(16), e quem é d(16)? Simples! Para chá-lo é só colocar na fórmula dada no início da questão: d(k + 12) - d(k) = 100. Então teremos:

    d(4 + 12) - d(4) = 100

    d(16) - 100 = 100

    d(16) = 200     Já poderia parar por aqui que já saberíamos que a razão é 100, pois 200 - 100 = 100, mas como pode haver algum companheiro com dúvida, eu farei mais uma. Então: d(k3)=?

    k3=29, substituindo teremos d(29). Mas quem é d(29)?

    d(k + 12) - d(k) = 100

    d(17 + 12) - d(17) = 100

    d(29) - d(17)  = 100 (ops... teremos que achar d(17) para achar o d(29) hahaha)

    Substituindo novamente na fórmula:

    d(5 + 12) - d(5) = 100

    d(17) - 100 = 100

    d(17) = 200, voltando para o dado anterior teremos:

    d(29) - d(17) = 100

    d(29) - 200 = 100

    d(29) = 300

    E para finalizar, para encontrar a razão de uma P.A. basta diminuir o segundo termo pelo primeiro, ou diminuir o terceiro pelo segundo, ou diminuir o quarto pelo terceiro, e assim sucessivamente.

    Espero ter ajudado!

    "Os limites somos nós que os criamos"

    Avante!


ID
22213
Banca
CESPE / CEBRASPE
Órgão
Banco do Brasil
Ano
2003
Provas
Disciplina
Raciocínio Lógico
Assuntos

Texto V - questões 13 e 14

Preparando-se para custear as despesas com a educação dos seus filhos, Carlos decidiu abrir uma poupança programada para 120 meses de duração, com rendimento mensal de 1%, em que os depósitos devem ser feitos no primeiro dia de cada mês. O valor d(k), em reais, do depósito a ser efetuado nessa poupança no k-ésimo mês obedece às seguintes regras:

. d(k) = 100, para k = 1, 2, ... , 12;
. d(k + 12) - d(k) = 100, para k > 1. 

 Considerando as informações do texto V e representando por S(k) o saldo, em reais, existente na poupança de Carlos por ocasião da realização do k-ésimo depósito (incluindo esse último depósito), julgue os itens subseqüentes.

Os valores S(1), S(13) - S(12) × 1,01 e S(25) - S(24) × 1,01 formam, nessa ordem, uma progressão aritmética de razão igual a 100.

Alternativas
Comentários
  • 1º termo => S(1) = 100
    2º termo => S(13) - S(12) × 1,01
    o termo S(13) é S(12).1,01 + 200
    1,01 por causa dos juros de 1% do mês anterior
    200 devido o depósito
    S(12).1,01 + 200 - S(12).1,01 = 200 
    3º termo => S(25) - S(24) × 1,01
    raciocínio análogo, mas com depósito de 300
    S(24).1,01 + 300 - S(24).1,01 = 300 

    100, 200 e 300, PA de razão 100!
  • GABARITO: C


ID
32467
Banca
CESGRANRIO
Órgão
BNDES
Ano
2008
Provas
Disciplina
Raciocínio Lógico
Assuntos

Uma torneira enche de água um tanque de 500 litros em 2 horas. Em quantos minutos 3 torneiras idênticas à primeira encherão um tanque de 600 litros, sabendo que todas as torneiras despejam água à mesma vazão da primeira e que, juntamente com as torneiras, há uma bomba que retira desse tanque 2,5 litros de água por minuto?

Alternativas
Comentários
  • Como a questão quer a resposta em minutos, primeiro converte-se as horas para essa unidade, no caso 2 horas * 60 minutos = 120 minutos;
    Os elementos para o cálculo são:
    500 litros >>>> 1 torneira >>>> 120 minutos
    600 litros >>>> 3 torneiras >>>> X minutos
    Usando regra de três composta, temos:
    120/x = 500/600 . 3/1
    (observando que o tempo é inversamente proporcional a quantidade de torneiras, temos que inverter a segunda fração)
    Resolvendo sabemos que, com 3 torneiras, levamos 48min para encher um tanque de 600L, todavia não podemos esquecer que existe uma bomba que retira 2,5L de água por minuto.
    Assim temos, através da regra de três simples:
    2,5 litros para 1 minuto
    X litros para 48 minutos
    Resolvida a conta, descobrimos que em 48 minutos conseguimos encher apenas 480L do tanque com capacidade para 600L, no entanto, a pergunta é em quantos minutos 3 torneiras idênticas à primeira encherão um tanque de 600 litros?
    Assim temos:
    48 minutos para 480 litros
    X minutos para 600 litros
    Portanto, para completar o tanque de 600L levamos 60min.
    Alternativa correta: b)
  • Bem, eu resolvi assim:

    Se 1 torneira enche um tanque de 500 litros em duas horas, sei que em uma hora 1 torneira consegue encher esse tanque com 250 litros. Logo, se as outras três torneiras são idênticas, 3 torneiras encherão o tanque de 600 com 750litros em uma hora (3*250), o que ultrapassa a medida a capacidade do tanque.
    No entanto, desse tanque são despejados 2,5 litros /min. Em uma hora temos 60 min, então em uma hora são despejados 150 litros de água (2,5*60). Então, se em 60 min (1 hora) encho o tanque com 750 e despejo 150, basta subtrair 750-150=600.
    Tenho então que em 1hora as três torneiras podem encher um tanque de 600.

    Simplificando:

    uma torneira enche 500 litros em duas horas
    Logo, uma torneira enche 250 litros em uma hora
    três torneiras enchem 750 litros em uma hora.
    Quero 600 litros.
    150 litros são despejados em 1 hora.
    Se três enchem 750 em uma hora, e despejo 150 também em uma hora... tenho 600 litros em uma hora enchidos pelas 3 torneiras!

    :)

  • Vamos lá! Sabemos que 500l --- 2h --- 120min, o que significa que em 1min teremos aproximadamente: 4,17litros.
    Como são três torneiras de mesma vazão? 3x 4,17 =12,50 litros(aproximado).
    Ou seja, 3torneiras --12,5 litros por minuto.Como perde-se 2,5 litro por minuto, na verdade as três torneiras enchem 10litros por minuto. Logo, 1min ---10l
    x ----600
    resolvendo a regra de três, x= 60 (JC)
  • (1T) (500L) (2H)
    (1T) (250L) (1H) Logo (3T) (750L) (1H) logo (375L) (1/2H) logo (187,5L) (15m) logo (12,5L) (1m)

    se em um minuto "vaza" 2,5L entao as 3 colocam 10L por min, e se sao 600L , (60 minutos)  B de bola!
  • 1 torneira - 500 litros - 120 minutos (2 horas)

    Logo: 1T enche 4,16 litros por minuto. 

    3 torneiras = 12,5 litros/minuto

    12,5 litros/minuto - 2,5 litros/minuto = 600

    10,0 litros/minuto = 600 litros

    minuto = 600 / 10 = 60
  • Para mim, o melhor jeito de chegar na resposta certa (letra "b") foi o seguinte: 

    -1 torneira despeja 250 litros de água por hora (em duas horas são 500 litros). 

    Seguindo essa vazão, três torneiras despejarão 750 litros de água por hora. 

    - É importante lembrar que o exercício cita o uso de uma bomba que retira 2,5 de água por minuto. 

    Já que são 750 litros de água por hora, com as três torneiras abertas, teremos: 

    - 375 litros em 30 minutos; 

    - 187,5 litros em 15 minutos. 

    Assim, em 45 minutos são 562,5 de água (3 X 187,5). 

    O exercício pede o tempo total para 600 litros de água, certo? Então, agora, colocaremos a bomba de água nos cálculos. 

    Descontando a vazão de 562,5 litros das torneiras, que acontece em 45 minutos, teremos a necessidade de mais 37,5 litros de água. 

    A partir do cálculo: 37,5/ 2,5 (2,5 litros por minuto) teremos 15. Ou seja, mais 15 minutos para que a bomba retire os 37,5 litros restantes e os 600 litros fiquem completos. 

    * 45 + 15 = 60 


  • 1º 2 horas equivalem a 120 minutos, que corresponde a 500 litros,  então, 1 hora(60 minutos) equivale à metade, que será 250

    2º As tres torneiras encheriam um tanque de 750 litros em 60 minutos No entanto, o tanque possui apenas 600, sobrando 150 litros

    3º ao mesmo tempo, uma bomba retira 2,5 litros por minuto. Nos mesmos 60 minutos,(pois a torneira e a bomba trabalham simultaneamente), a bomba retira 150 litros de agua do tanque (2,5x60), que corresponde à mesma medida que sobra dos 750 litros.(750-600=150)

    Assim, bastam 60 minutos para encher um tanque de 600 litros

    Letra B

     

  • Utilizei a técnica ensinada no vídeo abaixo para calcular o tempo gasto para encher o tanque de 500 litros e obtive os seguintes resultados:

    https://www.youtube.com/watch?v=2Bl7frW_fbU

    1/120 + 1/120 +1/120 - 1/200 esse "200" é o resultado de 500/2,5, ou seja 200 minutos para a bomba esvaziar 500 litros.

    200 + 200 + 200 - 120 / (120 * 200) Observação fazer a multiplicação 120 * 200 (que equivale ao MMC) é perda de tempo.

    480 / (120 * 200)

    4/200

    1/50 do tanque por minuto, ou seja, 50 minutos para encher o tanque.

     

    Aplicando regra de 3 encontramos 60 minutos pra encher o tanque de 600 litros

  • Eu cheguei na resposta mais demorei muito.

    Para mim, a maneira mais simples e rápida é da Jaciara Costa.

  • torneiras    litros          horas          bomba retira 2,5l/min
     1            500            120
     3            600+2,5x        x       
    120/x=3/1 * 500/600+2,5x
    120/x = 1500/600+2,5x
    1500x = 72000 + 300x
    1200x = 72000
    x = 720/12 = 60      

  • 500 L -- 120 min

    500 L / 120 min = 4,16

    1T = 4,16 L / min

     

    4,16 x 3 = 12,5

    3T = 12,5 L / min

     

    12,5 L / min - 2,5 L / min = 600 L

    10 L / min = 600 L

    min = 600 L / 10 L = 60


ID
51025
Banca
FCC
Órgão
TJ-SE
Ano
2009
Provas
Disciplina
Raciocínio Lógico
Assuntos

Se, para numerar todas as páginas de um texto, forem usados 225 algarismos do sistema decimal de numeração, quantas vezes o algarismo 3 aparecerá na numeração dessas páginas?

Alternativas
Comentários
  • É preciso estar atento para não efetuar o cálculo tendo em mente que são 225 páginas, mas ALGARISMOS. Assim sendo, temosdo 1 ao 9 -> 9 algarismosdo 10 ao 99 -> 180 algarismos (9 x 2) -----------------Sub-total 189 (- 225 = 36)36 algarismos divididos por 3, já que trata-se de números de centenas, teremos 12. Assim, teremos que calcular quantas vezes o dígito 3 haverá de 1 a 112.de 1 a 99 exceto a dezena de 30 a 39 -> 09de 30 a 39 ----------------> 11de 100 a 112 ----------------> 01 ---- TOTAL 21
  • Devemos simplificar. Lógica deve ser simples. Então vejamos:Imagine a seguinte tabela. Já preenchi as colunas e as linas. Basta imaginar seu preenchimento, ok:As colunas são: 10, 20, 30 .....110, 110 ...Ao unir as linhas com as colunas respectivas teremos por exemplo:coluna 1: 0,1,2,3,4,5..coluna 2 e 3: 10,11,12,13,14...coluna 4 e 5: 20,21,22,23,24...0 1 0 2 0 3 0 4 0 5 0 6 0 7 0 8 0 9 0 1 0 0 1 1 0123456789Bom, agora fica fácil de imaginar e calcular de cabeça. Por exemplo:1) de 0 à 9, foram utilizados 10 algarismos2) de 0 à 19, foram utilizados 3x10 algarismos3) de 0 à 29, foram utilizados 5x10 algarismos.4) de 0 à 99, foram utilizados 19x10 algarismos.4) de 0 à 109, foram utilizados 22x10 algarimos.Agora ficou fácil.Resolução:- de 1 à 109 - utilizamos 219 algarismos(22x10 -1)- faltam 6 (225-219)- então: 110 e 111 -> teremos os 6 restantes- então a sequência e: de 1 à 111, ok.- agora basta retornar a referida tabela e veremos que temos o número '3' na linha e na coluna. É fácil ver que o '3' aparece '21' vezes na tabela.Fácil, não. Questão resolvida sem efetuar nenhum cálculo. Tudo de cabeça.Bons estudos.
  • Primeiro devemos contar os algarismos, e depois os "3".A "tabela" abaixa foi estruturada conforme o descrito:Contagem - número de algarismos na contagem - quantidade de "3" na contagem - onde os números 3 aparecem1 a 9 - 9 algarismos - números "3": 1 (3)10 ao 19 - 20 algarismos - números "3": 1 (13)20 ao 29 - 20 algarismos - números "3": 1 (23)30 ao 39 - 20 algarismos - números "3": 11 (30, 31, 32, 33, 34, 35, 36, 37, 38, 39)40 ao 49 - 20 algarismos - números "3": 1 (43)50 ao 59 - 20 algarismos - números "3": 1 (53)60 ao 69 - 20 algarismos - números "3": 1 (63)70 ao 79 - 20 algarismos - números "3": 1 (73)80 ao 89 - 20 algarismos - números "3": 1 (83)90 ao 99 - 20 algarismos (total 189) - números "3": 1 (93)100 ao 109 - 30 algarismos (total 219) - números "3": 1 (103)110 - 3 algarismos (total 222) - números "3": 0111 - 3 algarismos (total 225) - números "3": 0Somando-se todos os "3" chegamos ao resultado: 21
  • Não entendi! Fiz toda a sequência e contei 42 números 3!
    30, 31, 32, 33, 34, 35, 36, 37, 38, 39 = 11 vezes que o némero 3 aparece
    130, 131, 132, 133, 134, 135, 136, 137, 138, 139 = + 11 vezes

    Só com essas sequências já 22 vezes que o número 3 aparece, fora as outras... Não sei como chegou a 21
  • Vamos lá gente

    Primeiro vamos saber quantas páginas aproximadamente este livro possui:
    0-9 paginas possui - 10 algarismos - confiram, pags 0,1,2,3,4,5,6,7,8,9 - contem
    9-99  paginas - 99-9=90 x 2 visto que a partir da página 90 passarão a ser dois algarismos por número- 10-11-12-13(....) =180 algarismos


    LOGO = Sao 225 algarismos ok? E até a pagina 99 temos 10+180 algarismos(0-9 + 9-99) que é igual a 190 algarismos.
    225-190 = 35 sendo que essas 35 paginas correspondem às paginas que estão após a página 99, logo devem ter 3 algarismos = 100,101,102,103,104,105.
    35\3=11 páginas que sobram. 

    logo: ao todo são 119 páginas. pois 9+99+11=119 apróximadamente.
    agora vamos lá

    0-9 - temos o algarismos 3 =1

    da página 10 até à página 99 temos 13,23,30,31,32,33,34,35,36,37,38,39,43,53,63,73,83,93. =18

    100 até a página 119 temos 103,113 =2

    Gente, agora é somente somar: 2+18+1
  • Meninos;


    Video que explica a questão:

    http://www.youtube.com/watch?v=xYX_76EsW24

    Solução do OPUS PI;

    Atenção:  A QUESTÃO FALA DE ALGARISMOS. EXEMPLOS: 9 = 1 ALGARISMO; 22 = DOIS ALGARISMOS 2 E 2)

    Primeiro, precisamos determinar quantas páginas foram numeradas. Considerando que a numeração começou na página 1, temos:



    Páginas de 1 a 9: usados 9 algarismos; (restam 225 - 9 = 216) ( DO 1 AO 9 SÃO NÚMEROS COM UM ALGARISMO)

    Páginas de 10 a 99: usados 180 algarismos ( 180, pois são 90 números com 2 algarimos); (restam 216 - 180 = 36)

    Com os 36 restantes, numeram-se somente páginas de três algarismos, de forma que apenas mais 36/3 = 12 páginas foram escritas.

    Portanto, tivemos 9 + 180/2 + 12 = 111 páginas. Ou seja, as páginas numeradas foram as 1, 2, 3, 4, ..., 109, 110, 111.

    Deseja-se saber quantos algarismos "3" temos na sequencia 1, 2, 3, 4, ..., 109, 110, 111.

    Total de algarismos "3" nas páginas de 1 a 9: 1
    Total de algarismos "3" nas páginas de 10 a 99: 19
    Total de algarismos "3" nas páginas de 100 a 111: 1

    Para determinar a quantidade 19 acima, repare que em cada uma das dezenas de "10", "20", "30", "40", "50", "60", "70", "80" e "90", o algarismo "3" aparece uma vez em cada uma na posição das unidades, totalizando 9 algarismos "3". (13, 23, 33, 43, 43, 63, 73, 83, 93)

    Além disso, na dezena de "30" o "3" aparece 10 vezes , todas elas no algarismo das dezenas. Assim, de 10 a 99, aparece 9 + 10 = 19.

    Portanto, o algarismo "3" foi escrito 1 + 19 + 1 = 21 vezes.

    FONTE; http://www.forumconcurseiros.com/forum/showthread.php?t=305637
  • fala-se em algarismos,sendo assim os numeros 13 e 14 são 4 algarismos como exemplo.Voltando à questão sabemos que 0 não conta  por isso são 9 algarismos(1 a 9) deduzindo dos 225 encontramos 216,a partir do 9 são 2 algarismos por número logo 216/2=108 +9=117 foi a numeração de páginas agora basta saber qnts 3 existe de 1 a 117.De 30 a 39 são 11 e 1 a cada 10 numeros que são 10 logo= 11+10=21 perdoe-me a falta de coesão.

  • Só esclarecendo vai da página 1 a 111 (e não 112) como no comentário top curtido


    Pq? do número 100 ao 111 temos 12 números

    Se fizer de 100 a 112 daria 113 números

    No caso coincidiu de bater o resultado, mas cuidado!


ID
76798
Banca
CESGRANRIO
Órgão
BACEN
Ano
2010
Provas
Disciplina
Raciocínio Lógico
Assuntos

Analise as frases abaixo e assinale:

S: caso a declaração contenha um equívoco do ponto de vista da lógica verbal;

N: em caso contrário.

( ) Pretendendo acabar com as baratas que havia em sua casa, comprou remédio para insetos.

( ) De acordo com o calendário de datas festivas do Brasil, em novembro há um feriado.

( ) Sua vida mudou radicalmente; pode-se dizer que deu um giro de 360°.

A sequência correta das letras, de cima para baixo, é

Alternativas
Comentários
  • Em relação a declaração 1: o equívoco está no fato de comprar REMÉDIO pra matar baratas. Remédio é pra curar e não matar.Na declaração 2 não há nenhum equívocoNa declaração 3, a vida da suposta pessoa deu um giro de 3600. Um giro completo é de 360 graus, levando ao pé da letra a pessoa deu 10 giros de 360. E sabemos que quem dá um giro completo, volta ao mesmo lugar.essa foi minha interpretação!!
  • Remédio para matar? Giro de 3600? Só o Tony Hawk! Letra b)
  • Em Novembro não há dois feriados?
  • Pessoal, dois esclarecimentos:
    - A afirmativa "Sua vida mudou radicalmente; pode-se dizer que deu um giro de 3600" foi colocada erradamente. O enunciado original diz que mudou 360º (360 graus) e não 3600. Logo, incorreto porque como sabemos esse giro conduz ao mesmo lugar.
    - Quanto ao comentário do Josivan, em Novembro há mais feriados, porém o dia 2/11 é Finados, e o enunciado fala em data FESTIVA. Finados é o dia que homenageia os mortos, portanto, considerá-lo data festiva contraria a lógica.
    Abçs a todos.
  • GABARITO:  LETRA B

    A primeira frase possui um equívoco, pois para matar as baratas ele deveria comprar veneno para insetos. A segunda frase não possui nenhum equívoco, pois 15 de novembro é de fato feriado nacional. Finalmente, a terceira frase não faz sentido, pois com um giro de 360 graus e vida voltaria exatamente ao mesmo lugar. Logo, temos S - N - S.
  • Acho essas afirmativas de lógica verbal muito perigosas.

    I - OK, unanimidade. - errada

    II - Essa assertiva achei estranha, já que em novembro há 2 feriados. Finados, que não seja uma data "festiva"... e a Proclamação da república... é uma data festiva?

    III - Ok tb - errada.

    Se tivesse uma alternativa S - S - S eu assinalaria.

    Como não teve, optei pela S - N - S e acertei
  • "...acabar com as baratas que HAVIA..."

  • Pelo que vi dos comentários, é preciso tomar cuidado. Não importa o número de feriados que de fato temos em Novembro, porque ele não pergunta se as premissas são falsas, mas só se o argumento é válido. A questão pergunta se há equívoco do ponto de vista da lógica verbal. Ora, para olhar os feriados de um mês, ele se utilizou de um calendário de datas festivas. Não há equívoco nenhum.

  • O gabarito é "B" mesmo?

    O meu deu "D"


ID
84382
Banca
ACEP
Órgão
BNB
Ano
2006
Provas
Disciplina
Raciocínio Lógico
Assuntos

As três seqüências de números a seguir obedecem a uma lei de formação lógica. Determine o número ausente da última seqüência, indicado por um sinal de interrogação.

(12, 4, 15) (36, 9, 20) (21, ?, 35)

Alternativas
Comentários
  • primeira sequencia:(12, 4, 15)12:4=33x5=15O resultado da divisão do primeiro pelo segundo é multiplicado por 5 e encontramos o terceiro termo:segunda sequencia:(36, 9, 20)36:9=44x5=20terceira sequencia:(21, ?, 35)qual número que multiplicado por 5 da 35? é o 7, logo o segundo termo deve ser um número que ao dividir o primeiro dê 7 como resultado.21:3=77x5=35Logo a resposta é 3
  • Dada a sequência (a, b, c), a lei de formação é: (a/b).5 = c

    Isolando b, temos: b = (a/c).5 = (21/35).5 = 3 [Item A]


ID
93094
Banca
FCC
Órgão
DNOCS
Ano
2010
Provas
Disciplina
Raciocínio Lógico
Assuntos

Na sentença abaixo falta a última palavra. Procure nas alternativas a palavra que melhor completa essa sentença.

Padecia de mal conhecido e de tratamento relativamente fácil. Como era imprudente e não se cercava dos devidos cuidados, tornava impossível qualquer

Alternativas
Comentários
  • a) Diagnóstico - Conhecimento ou determinação de uma doença pelo(s) sintoma(s), sinal ou sinais e/ou mediante exames diversos (radiológicos, laboratoriais, etc).b) Observação - Ato ou efeito de observar(-se); Exame, análise.c) Consulta - Ato de consultar, procedimento para obtenção de informação.d) (alternativa correta) Prognóstico - Juízo médico, baseado no diagnóstico e nas possibilidades terapêuticas, acerca da duração, evolução e termo de uma doença.e) Conjetura - Juízo ou opinião sem fundamento preciso; suposição, hipótese.
  • O mal é conhecido, seu tratamento também, assim o diagnóstico já existe e a consulta também.Ele era imprudente e não se cercava de cuidados, logo não é possível saber o resultado, os efeitos da doença nele.Torna impossível qualquer PROGNÓSTICO.
  • segundo o Aurélio:
    prognóstico

    [Do gr. prognostikón, pelo lat. prognosticu.]
    Substantivo masculino
    1.Conjetura sobre o desenvolvimento de um negócio, de uma situação, etc.; predição, agouro, presságio, profecia:
    “adotamos o sistema da crítica, fenômeno literário, se lhe posso chamar, que era em Portugal espantoso prognóstico de desastres” (Correia Garção, Obras Poéticas e Oratórias, p. 553).

    2.Med. Juízo médico, baseado no diagnóstico e nas possibilidades terapêuticas, acerca da duração, evolução e termo de uma doença. 


    Acredito que a questão deveria ser anulada, pois o conceito de prognóstico traz em si: Conjetura e diagnóstico
    AA 
  • letra d. 

    Definições de diagnóstico e prognóstico de acordo com área médica:

    Diagnóstico: é o conhecimento ou juízo ao momento, feito pelo profissional da saúde, acerca das características de uma doença ou de um quadro clínico, que comumente suscita um prognóstico, com base nas possibilidades terapêuticas, segundo o estado da arte, acerca da duração, da evolução e do eventual termo da doença ou do quadro clínico sob seu cuidado ou orientação.

       

    Prognóstico: é conhecimento ou juízo antecipado, prévio, baseado necessariamente no diagnóstico e nas possibilidades terapêuticas, segundo o estado da arte, acerca da duração, da evolução e do eventual termo de uma doença ou quadro clínico sob seu cuidado ou orientação. É predição do médico de como a doença do paciente irá evoluir, e se há e quais são as chances de cura.

    Fonte: PORTAL EDUCAÇÃO - Cursos Online : Mais de 1000 cursos online com certificado 

    http://www.portaleducacao.com.br/psicologia/artigos/12268/diagnostico-e-prognostico#ixzz3ZNgPL8f0

  • Diagnóstico é o resultado atual de uma avaliação no momento presente. É o que se tem no momento.

    Prognóstico é um resultado prévio, algo que pode acontecer no futuro. O prognóstico é obtido a partir de interpretações feitas com base no diagnóstico.

    www.galirows.com.br/meublog/blogdasdiferencas/2017/12/11/diferenca-diagnostico-prognostico/


ID
93118
Banca
FCC
Órgão
DNOCS
Ano
2010
Provas
Disciplina
Raciocínio Lógico
Assuntos

Os termos da sequência (12, 15, 9, 18, 21, 15, 30, 33, 27, 54, 57, . . .) são sucessivamente obtidos através de uma lei de formação. Se x e y são, respectivamente, o décimo terceiro e o décimo quarto termos dessa sequência, então:

Alternativas
Comentários
  • A lógica é a seguinte:Soma o primeiro com 3 que dará o segundo (12+3=15); subtrai o segundo em 6 e dará o terceiro (15-6=9); o quarto é o terceiro em dobro (2x9=18) e assim sucessivamente até chegar no 12º que será 51, o 13º será 102 e o 14º será 105.A única resposta válida é a B que diz que y=x+3 105=102+3.
  • Devemos colocar as razões entre os números para descobrir a lógica : 12 15 9 18 21 15 30 33 27 54 57 51 13º 14º +3 -6 x2 +3 -6 x2 +3 -6 x2 +3 -6 x2 +3 13º= 51 x2 = 102 14º= 13º +3 = 102 +3 = 105 Então y=105 e x=102 sendo y=x+3 letra B)
  • Seria possivel alguém explicar a lei de recorrencia da questão com maior clareza??
  • a sequência é a seguinte:

    -primeiro soma 3

    -resultado diminui 6

    -resultado multiplica por 2

    -e assim sucessivamente..

  • Eu fiz por um método mais complicado, mas consegui chegar ao resultado.

    sequência: 12, 15, 9, 18, 21, 15, 30, 33, 27, 54, 57...

    1° - dividir todos por 3 = 4, 5, 3, 6 , 7, 5, 10, 11, 9, 18, 19...

    2° entre os números faz-se as seguintes operações na seguinte ordem:+1, -2, x2...)

    fica assim= 4 (primeiro número da sequencia) + 1 = 5 (segundo número da sequência)
                        5  (segundo número da sequência) - 2 = 3 (terceiro número da sequência)
                        3 (terceiro número da sequencia)  x2 = 6 (quarto número da sequência)

                        6 (quarto número da sequencia) +1 = 7 (quinto número da sequencia)
                        7 (quinto número da sequencia) -2 = 5 (sexto número da sequencia)
                        5 (sexto número da sequencia) x2 = 10 (sétimo número da sequencia)

    Continua assim: 4,5,3,6,7,5,10,11,9,18,19,17,34,35...                    Logo: o décimo terceiro e décimo quarto são respectivamente: 34 e 35.
                        Para voltar à sequencia original, deve-se x3 novamente. Os número 13° e 14° são 102 e 105.
                        Dessa forma, a única resposta correta é a B (105=102+3).
  • Questão covarde, para quem precisa administrar o tempo de prova...
  •     +3  - 6    x2   +3  - 6   x2  +3 - 6   x2   +3  - 6    x2      +3
         ^    ^    ^    ^    ^    ^      ^    ^     ^      ^    ^     ^      ^
    12   15   9   18   21   15   30   33   27   54  57   51   102   105
                


    Acho que assim fica mais fácil de enxergar a lógica com que os números são apresentados ( esse editor deixa as flechas meio desencontradas, mas dá pra entender, né?).  Além de vc ter que enxergar isso,  vc tinha que estar atento para o fato de que ele não quer os dois próximos números...  e sim, o  13o  e o 14o...    e  a relação entre eles é:   y = x + 3.   Letra B    OBS>  Se  vc não atentasse para esse detalhe, vc marcaria a D...  e aí... ferrô-se...
  • GABARITO: B

    Este é uma típica questão de "adivinhação lógica". Faz-se necessário treinar o olhar e resolver muitas questões desse tipo, aí garanto que você não perderá mais nenhuma questão parecida! :)

    Aqui a primeira coisa que temos que fazer é descobrir qual a lei de formação que a sequência segue. Pois bem, depois de analisar com cuidado chegamos à seguinte conclusão:

    Lei de formação: + 3 , - 6 , x 2. Ou seja, a sequência ficou assim:
    12,15,9,18,21,15,30,33,27, 54 (x2), 57(+3), 51(-6), 102(x2), 105(+3)......


    Onde o 13o.termo é o: 102 (X)
    Onde o 14o.termo é o: 105 (Y)


    Ou seja: 105 = 102 + 3
  • o padrão é: mais 3 menos 6 vezes 2

    12 + 3= 15

    15-6= 9

    9x2= 18 


     

  • Segue um padrão de +3 -6 x 2 ou seja

    12+3= 15

    15-6 = 9

    9x2 = 18

    na lógica teremos o valor do décimo terceiro numero que sera X e o décimo quarto que é y

    ou seja y=x+3 Gab:B


ID
119125
Banca
FCC
Órgão
TRF - 4ª REGIÃO
Ano
2010
Provas
Disciplina
Raciocínio Lógico
Assuntos

Uma propriedade comum caracteriza o conjunto de palavras seguinte:

MARCA - BARBUDO - CRUCIAL - ADIDO - FRENTE ?

De acordo com tal propriedade, a palavra que, em sequência, substituiria corretamente o ponto de interrogação é

Alternativas
Comentários
  • Observe:MARCA - duas letras A.BARBUDO - duas letras B.CRUCIAL - duas letras C.ADIDO - duas letras D.FRENTE - duas letras E.Pela lei, a próxima palavra tem que ter duas letras F: FOFURA.Letra D.Opus Pi.
  • Caro colega "Opus Pi",PARABÉNS!!! Confesso que estava resolvendo essa prova, acertei todas as outras questões de RLM e matemática, mas essa não saia de jeito nenhum! Não conseguia entender que relação havia com "fofura"...Cheguei a mostrar pro meu marido, e ficamos um tempão tentando decifrar o mistério da propriedade comum... rs!!Você é um gênio!!!
  • Parabens Opus Pi, também não estava conseguindo enxergar essa solução.
  • AFFF!!!
    onde eu perdi minha esperteza?
    essa não deu pra resolver, tive que vir olhar os comentários pra saber alguém tinha decifrado
  • Outra forma de resolver:

    MARCA - BARBUDO - CRUCIAL - ADIDO - FRENTE 
     5 letras     7 letras        7 letras      5 letras     6 letras

    Isso significa que a próxima palavra deveria ter seis letras.  

    Letra c 
  • Jr.

    Nada a ver!
    Forçar algum tipo de coincidência não significa "outra forma de resolver" :P

    A ideia é realmente a sequência alfabética das letras repetidas
  • Desculpe caro colega mais eu consegui resolver da mesma forma!!!!!! 
  • tambem não consegui resolver e tive que vir ver os comentarios mas acho que a forma correta é a de combinação de letras, porem da outra forma tambem se chega ao resultado!
  • O cara das 6 letras acerta na pura sorte e ainda não dá o braço a torcer... 
  • Ótima linha de raciocínio do opus pi, mas percebam que também há outra forma de interpretação:

    de trás pra frente se se contarem 4 letras se chega a uma sequência alfabética...

    MARCA   ACR..A

    BARBUDO  ODU...B

    CRUCIAL  LAI...C

    ADIDO  ODI...D

    FRENTE  ENT...E

    FOFURA  ARU...F


  • O ponto de interrogação? Qual dos seis pontos de interrogação? Desse jeito não dá pra resolver mesmo...

  • Há várias formas de resolver esta questão. Ao meu ver, uma questão fácil. Eu pensei da seguinte forma:

    MARCA = 2 letras A

    BARBUDO = 2 letras B

    CRUCIAL = 2 letras C

    ADIDO = 2 letras D

    FRENTE = 2 letras E

    Próxima palavra, deve conter 2 letras F. Neste caso, FOFURA - Letra C.

  • Quantidade de letras. 5 - 7 - 7; 5 - 6 - ( a proxima tem que ter 6)

  • 2 A, 2 B, 2C, 2D, ... Logo 2 F. Letra C: FOFURA

  • Acho que essas questões de raciocínio são pura sorte.. olha a minha lógica

     

    MARCA - BARBUDO - CRUCIAL - ADIDO - FRENTE 

                     DO                          DO                          O próximo teria de ser terminado com DO, logo,  ILIBADO

     

    Mas, enfim....

  • Meu Deux!!!!!

  • A maneira mais simples que encontrei foi considerar o número de letras de cada palavra e sequência lógica... 5 letras, 7 letras, 7 letras, 5 letras, 6 letras... próxima: 6 letras!

    Resposta: FOFURA

    Não é sorte como disseram...é lógica! Caso tivesse dado outro resultado, caberia recurso.

    AVANTE!

  • A maneira mais simples que encontrei foi considerar o número de letras de cada palavra e sequência lógica... 5 letras, 7 letras, 7 letras, 5 letras, 6 letras... próxima: 6 letras!

    Resposta: FOFURA

    Não é sorte como disseram...é lógica! Caso tivesse dado outro resultado, caberia recurso.

    AVANTE!

  • Resposta: Veja que as letras das palavras que se repetem seguem a lógica do "ABC" sendo elas repetidas: A em MARCA, B em BARBUDO, C em CRUCIAL, D em ADIDO, E em FRENTE;  Resposta: (C) F em FOFURA. 

  • Na sequência apresentada na questão, a palavra “marca” tem 2 a, “barbudo” tem 2 b e assim por diante.

    Logo, a palavra da alternativa correta deve ter 2 f. 


ID
122926
Banca
FCC
Órgão
SEFAZ-SP
Ano
2009
Provas
Disciplina
Raciocínio Lógico
Assuntos

Considere a sequência:

(P, 3, S, 4, W, 5, B, 4, F, 3, ......)

De acordo com a lógica observada nos primeiros elementos da sequência, o elemento, dentre os apresentados, que a completa corretamente é

Alternativas
Comentários
  • Como pode-se observar, o próximo termo da sequência é uma letra.(P, 3, S, 4, W, 5, B, 4, F, 3, ......)E a ordem da sequência é que a letra que vem após o número é igual a posição no alfabeto da letra anterior menos o número que a sucede.Então, se B - 4 = F (pule C,D,E)=> F - 3 = I (pule G e H)Não há fórmula mágica. É raciocíno lógico-dedutivo mesmo.Letra (C)
  • (P+3= S+ 4= W+ 5= B+ 4= F+ 3=I ......)
  • Pela sequência percebe-se que é uma letra.
    Vamos à definição dessa letra.
    Acompanhando a lógica da sequência vi que:
    1 - (P, 3, S, 4, W, 5, B, 4, F, 3, ......)

    Obs. Perceba que a letra seguinte é a quantidade de letras determinada por: você Ignora a primeira e inclui a última.
    Assim,
    1º de "P" para "S" existem duas letras (q, r). Ignora o P (primeira) e inclui o S (última), ou seja, totaliza o três que está entre o "P" e o "S";
    O pensamento é o mesmo para a seguencia:
    2º de "S" para "W" existem três letras (t,u,v e W);
    3º de "W" para "B" existem quatro letras (x, y, z, a e B);
    4º de "B" para "F" existem três letras (c, d, e e F);
    5 de "F" para "?" existem duas letras (g, h e "?= i)
  • Letra"C"

    Pessoal da para resumir bem mais esse raciocínio.

    (P, 3, S, 4, W, 5, B, 4, F, 3, ......)

    Andou 2  chegou na 3 depois andou 4 chegou ha 5 quando chegar no B ando 3 letras e por ultimo andar 2 chego na letra
    I

    P         S             W                   B                F             I
       Q R     T U V         X Y Z A          C D E         G H
  • ALTERNATIVA C

    (P, 3, S, 4, W, 5, B, 4, F, 3, ......)

    A, B, C, D, E, F, G, H, I, J, K, L, M, N, O, P, Q, R, S, T, U, V, W, X, Y, Z

    P+3= S( Q,R,S)

    S+4=W(T,U,V,W)

    W+5= B (X,Y,Z,A,B)

    B+4= F ( C,D,E,F)

    F+3= (G,H,I)

  • RESOLVIDO:

    https://youtu.be/mYxlQfPvzwM


ID
125572
Banca
ESAF
Órgão
Prefeitura de Natal - RN
Ano
2008
Provas
Disciplina
Raciocínio Lógico
Assuntos

Os seis primeiros termos de uma seqüência de 1500 números são iguais a 2, x, 8, y, p, q, r, s... . Esta seqüência possui uma propriedade bastante interessante, a saber: cada termo, a partir do terceiro (inclusive), é a média aritmética de todos os termos anteriores. Com isso, o último termo dessa seqüência é igual a:

Alternativas
Comentários
  • Esta questão deve ter sido anulada. A resposta correta seria 8.

  • Tirando a média dos termos, temos x=14  y=8  p=8  q=8  r=8  s=8

    com isso r= 0

    an=2+(1500-1)0

    an=2

    resposta letra d

  • Sabemos que 8 é a média aritmética de x e 2. Portanto:

    2+x / 2 =8  =>  x=14

    Veja que o enunciado nos diz que cada elemento, a partir do terceiro, é a média aritmética de todos os seus antecessores.
    A partir daí acontece uma coisa interessante. A resposta para todos os elementos seguintes será 8. Por quê? Porque quando acrescento um elemento em um conjunto que é exatamente a média aritmética daquele conjunto, a média continua sendo a mesma. Vamos calcular y para mostrar:

    2+14+8 / 3 = y => y=8

    Se formos calcular, p, q, etc, sempre resultará em 8, porque estamos acrescentando no conjunto um número que coincide com a média. Dessa maneira a média não se altera.
    Não temos resposta. Por isso a questão foi anulada.
     
    Juci Melim Junior do Ponto dos Concursos
  • Olá, pessoal!

    Essa questão foi anulada pela organizadora.


    Bons estudos!
  • Questão interessante que envolve, também, as propriedades da média aritmética, creio que o digitador tenha errado na letra B, trocou o "8" por "0"

    Quanto ao comentário da colega Ívna é necessário uma pequena retificação, pois a razão da PA é r = 0 a partir do 3º termo. 
    Dai, a fórmula do An = A1 + (n-1)*r, será a partir do A3. Assim: A1500 = A3 + 1.497 *r => A1500 = A3 = 8 (sem gabarito)

ID
131950
Banca
FGV
Órgão
CAERN
Ano
2010
Provas
Disciplina
Raciocínio Lógico
Assuntos

Considere a sequência numérica (1, 4, 5, 9, 14, 23, ...). O primeiro número dessa sequência a ter 3 algarismos é

Alternativas
Comentários
  • É SÓ FAZER A SEQUENCIA (1,4,5,9,14,23,37,60,97,157)RESPOSTA a=157.
  • Mary, ótimo comentário. Obrigada, que bom que você explica detalhadamente :)
  • 1+4=55+4=9, 9+5=14, 14+9=23, 23+14=37, 37+23=60, 60+37= 97, 97+60= 157

    157 é o primeiro número de 3 algarismos

    Espero ter ajudado.


  • An = 10°

  • Basta fazer a sequência de fibonacci.

  • o terceiro número é sempre a soma dos dois anteriores.

    1+4=5

    4+5=9

    5+9=14

    9+14=23

    14+23=37

    23+37=60

    37+60=97

    60+97=157


ID
143986
Banca
FCC
Órgão
TCE-GO
Ano
2009
Provas
Disciplina
Raciocínio Lógico
Assuntos

Considere que os números que compõem a sequência seguinte obedecem a uma lei de formação.

(120; 120; 113; 113; 105; 105; 96; 96; 86; 86; . . .)

A soma do décimo quarto e décimo quinto termos dessa sequência é um número

Alternativas
Comentários
  • Só ir subtraindo:
    120 - 113 = 7
    113 - 105 = 8
    105 - 96 = 9
    96 - 86 = 10

    Repare que o resultando sempre aumenta 1..continuando as subtrações
    86 - x = 11 .... x  = 75
    75 - x = 12 .... x = 63
    63 - x = 13 .... x = 50

    A sequência ficou: (120;120;113;113;105;105;96;96;86;86;75;75;63;63;50;50)
    14º = 63
    15º = 50

    63 + 50 = 113 número ímpar.
  • Simples assim....

    120; 120; 113; 113; 105; 105; 96; 96; 86; 86; 75;75;63;63;50....

    63+50=231

    portanto ...numero ímpar
  • Letra"B"




  • Os números são diminuídos na sequência 7, 8, 9, 10, 11, 12..., ou seja, 120-7= 113; 113-8= 105; 105-9= 96; 96-10= 86; 86- 11= 75; 75-12= 63; 63-13= 50... Assim, obedecendo a sequência (120; 120; 113; 113; 105; 105; 96; 96; 86; 86; 75; 75; 63; 63; 50; 50), verificamos que o 14º e 15º são o 63 e 50. A soma deles 113.



ID
148066
Banca
FCC
Órgão
TRT - 16ª REGIÃO (MA)
Ano
2009
Provas
Disciplina
Raciocínio Lógico
Assuntos

Considere p = FALSE e q = TRUE. Os resultados booleanos de p AND q, p OR q e NOT p serão, respectivamente,

Alternativas
Comentários
  • p = Falso
    q = Verdadeiro

    p AND q
    F AND V = Falso (conectivo AND para ser verdadeiro os 2 valores devem ser verdadeiro, caso contrario o resultado é falso);

    p OR q
    F OR V = Verdadeiro (para conectivo OR bastar um dos valores ser verdadeiro, só resulta em falso se os 2 valores serem falsos);

    NOT p
    NOT F = Verdadeiro (conectivo NOT inverte o valor sendo Falso passa a ser verdadeiro e vice-versa)

    Portanto auternativa "D" é a correta.

    Espero ter ajudado.

ID
158665
Banca
CESGRANRIO
Órgão
TJ-RO
Ano
2008
Provas
Disciplina
Raciocínio Lógico
Assuntos

Em uma seqüência de números, o primeiro termo é 61 e todos os outros termos correspondem à soma dos quadrados dos algarismos do termo anterior. O número que ocupa a 81ª posição desta seqüência é

Alternativas
Comentários
  • A sequência dos 9 primeiros números é:  61 - 37 - 58 - 89 - 145 - 42 - 20 - 4 - 16

     Como 61 e 16 geram o mesmo resultado (37) os próximos números se repetirão até voltar ao 16. Projetando essas repetições achamos a letra B.
     61 - 37 - 58 - 89 - 145 - 42 - 20 - 4 - 16 (proposições de 1 até 9)
     37 - 58 - 89 - 145 - 42 - 20 - 4 - 16 (proposições de 10 até 17)
     37 - 58 - 89 - 145 - 42 - 20 - 4 - 16 (proposições de 18 até 25)
     37 - 58 - 89 - 145 - 42 - 20 - 4 - 16 (proposições de 26 até 33)
     37 - 58 - 89 - 145 - 42 - 20 - 4 - 16 (proposições de 34 até 41)
     37 - 58 - 89 - 145 - 42 - 20 - 4 - 16  (proposições 42 de  até 49)
     37 - 58 - 89 - 145 - 42 - 20 - 4 - 16 (proposições de  50 até 57)
     37 - 58 - 89 - 145 - 42 - 20 - 4 - 16  (proposições de 58 até 65)
     37 - 58 - 89 - 145 - 42 - 20 - 4 - 16  (proposições de 66 até 73)
     37 - 58 - 89 - 145 - 42 - 20 - 4 - 16  (81ª posição)
  • Este é o tipo de problema que se faz por eliminação; senão vejamos:

    a1 = 61------>está na letra e)

    a2 = 37----->está na letra c)

    a3 = 58

    a4 = 89

    a5 = 145

    a6 = 42----->está na letra d)

    a7 = 20

    a8 = 4------>está na letra a), logo só restou a letra b).

  • 1º termo = 61
    2º termo = 6^2 + 1^2 = 36 + 1 = 37
    3º termo = 3^2 + 7^2 = 9 + 49 = 58
    4º termo = 5^2 + 8^2 = 25 + 64 = 89
    5º termo = 8^2 + 9^2 = 64 + 81 = 145
    6º termo = 1^2 + 4^2 + 5^2 = 1 + 16 + 25 = 42
    7º termo = 4^2 + 2^2 = 16 + 4 = 20
    8º termo = 2^2 + 0^2 = 4
    9º termo = 4^2 = 16
    10º termo = 1^2 + 6^2 = 1 + 36 = 37

    Pronto! Houve a repetição! O 10º termo é o mesmo que o 2º!

    Agora, o pulo do gato: o 9º termo é 16 e o 1º é 61. São diferentes? Sim e não! Sim, porque (lógico!) são números diferentes. Porém, são iguais porque vão gerar o número 37.

    Então, a sequência é formada por 8 números. A partir do 9º, é repetição!

    Mais um detalhe: o número 61 só aparece no 1º termo. Nos outros, o número que aparecerá é o 16.

    Agora, vamos dividir 81 (termo que a questão pede) por 8. Encontraremos quociente 10 e resto 1. É como se déssemos 10 voltas na sequência e parássemos no 1º termo.

    E quem é o 1º termo? 61, correto? Então, é a resposta, correto?

    ERRADO! Como já explicamos acima (em negrito), o número que sempre aparecerá na sequência é o 16.

    Fonte: http://beijonopapaienamamae.blogspot.com.br/2010/01/dia-06-de-janeiro-questao-06.html

  • comentem esta questão professores

  • LETRA B


ID
161089
Banca
FCC
Órgão
TRF - 1ª REGIÃO
Ano
2006
Provas
Disciplina
Raciocínio Lógico
Assuntos

Assinale a alternativa que completa a série seguinte: C3, 6G, L10,...

Alternativas
Comentários
  • A alternativa (D) é a resposta. Os números devem alternar, ora antes, ora depois da letra;Os números também seguem uma seqüência: do 3 para 6: +3; do 6 para o 10: +4; logo, do 10 para o 15: +5.A lógica da seqüência das letras é em função da seqüência dos números, da seguinte forma:3C: O número é 3 e a letra é a 3ª (3 = 3 + 0)6G: O número é 6 e a letra é a 7ª (7 = 6 + 1)L10: O número é o 10 e a letra é a 12ª (12 = 10 + 2)15R: O número é o 15 e a letra é a 18ª (18 = 15 + 3).
  • Obs. O raciocínio abaixo só dá certo se considerarmos o alfabeto com K, W e Y, o que é estranho, pois em 2006 essas letras ainda não faziam parte do nosso alfabeto e não é feita nenhuma resalva sobre isso na questão. Mas o raciocínio da colega está perfeito!;)
  • Segui o seguinte raciocínio:
    1º=> Há uma sequência entre LN NL LN ( L=Letra e N=Número).
    2º=>Nos números temos: de 3 para 6( soma 3) ; de 6 para 10 (soma 4); então 10 para o próximo soma 5, logo o próximo número é 15.
    3º=>Considerando o alfabeto completo, temos: A letra C é a 3ª letra do alfabeto; a letra G é a 7ª letra do alfabeto; a letra L é a 12ª letra do alfabeto, logo da 3ª letra para a 7ª letra soma-se 4, e da 7ª letra para 12ª soma-se 5, então soma-se 6 a 12ª letra, chegando na 18ª letra do alfabeto ,que é a letra R.

    Portanto Letra D
  • Letra"D"

    Pessoal tentar da uma resumida nesta questão

    (C3, 6G, L10...)

    A Sequencia foi  3,4 a proxima é 5


    3        6          10                    15
      4 5      7 8 9      11 12 13 14

    C         G            L                      R
       D E F   H I J K    M N O P  Q

ID
161095
Banca
FCC
Órgão
TRF - 1ª REGIÃO
Ano
2006
Provas
Disciplina
Raciocínio Lógico
Assuntos

Assinale a alternativa que completa a série seguinte: 9, 16,25, 36,...

Alternativas
Comentários
  • observe que a sequencia de números quadráticos:3^2=9, 4^2=16, 5^2=25, 6^2 = 36, 7^2 = 49
  • Questão muito fácil, vejam :

    9,16,25,36,... eu vou explicar qual é o critério.

    9 pra chegar a 16 = 7

    16 pra chegar a 25 = 9

    25 pra chegar a 36 = 11

    36 pra chegar em 49 = 13

    o critério e de aumentar a diferença em 2 e 2, como conseguimos ver 7 pra 9 = 2, 9 pra 11 = 2, 11 pra 13 = 2

    Se perguntassem o numero que vem depois do 49 seria 64, porque 49 + 15 = 64

    Bons Estudos !!!

  • A resolução do Everton está correta! o sinal demonstrado por ele por " ^  "  equivale a elevação de uma potência

    A série 9, 16, 25, 36  é igual a série  32;  42;  52;  62   então o proximo seria 72  que é igual a 49

    Letra b)
  • Fiz igual ao Pedro Henrique e deu certo também.
  • Letra B.

    Essa questão de lógica tem 2 teorias que dão certo. Os colegas já apresentaram ambas, resta saber qual a banca utilizou de fato.
  • A banca geralmente utiliza as duas teorias, por isso que da o mesmo resultado.
  • 9+7= 16                +9=25                 +11= 36                        +13=49

    Saiba analisar!

     

  • 9 16 25 36 49

    3x3 4x4 5x5 6x6 7x7

  • Quadrados perfeitos, letra B.

  • 9,16,25,36

    Do 9 pro 16 aumenta 7

    do 16 pro 25 aumenta 9

    do 25 pro 36 aumenta 11

    Antes de ir pro próximo, veja o padrão de quanto aumentou, pois ai que está a pegadinha.

    7-9-11 então o próximo é 13

    ou seja

    36 + 13 = 49

    Gabarito letra B


ID
161251
Banca
FCC
Órgão
TRF - 5ª REGIÃO
Ano
2008
Provas
Disciplina
Raciocínio Lógico
Assuntos

Observando a seqüência (2, 5, 11, 23, 47, 95, ...) verificase que, do segundo termo em diante, cada número é obtido a partir do anterior, de acordo com uma certa regra. Nessas condições, o sétimo elemento dessa seqüência é

Alternativas
Comentários
  • 2+2 = 4 + 1 = 55+5 = 10 + 1 = 1111 + 11 = 22 + 1 = 2323 + 23 = 46 + 1 = 4747 + 47 = 94 + 1 = 9595 + 95 = 190 + 1 = 191
  • 2 + 3 = 5 5 + 6 (dobro 3 da conta anterior e assim por diante) = 11 11 + 12 (dobro de 6)= 23 23 + 24 (dobro de 12)= 47 47 + 48 (dobro de 24) = 95 95 + 96 (dobro de 48) = 191 - Resposta b
  • 2x5+1=11
    2x11+1=23
    2x23+1+47
    2x47+1=95
    2x95+1=191
  • a diferença entre os primeiros numeros.... 5-2=3 .. 3+3+55=11.............11-5=6...6+6+11=23.... com o dobro da difereça, mais o último numero somado, sempre se chega ao número posterior..... até 48 x2 = 96 + 95 = 191
  • Meu raciocínio foi: O número que segue surge a partir do dobro do anterior somado +1unidade:
         2,                   5,                   11,                        23,            47,              95, ...
    2x2=4+1=5    2x5=10+1=11    2x11=22+1=23  2x23=46+1=47  2x47=94+1=95  2x95=190+1=191
    Gabarito: Letra B
  • somar com o dobro

    2(+3), 5 (+6), 11(+12), 23(+24),47(+48),95(+96), 191

  • 2, 5, 11, 23, 47, 95,

    +3 +6+12+24+48+96

    Cada casa aumenta de acordo com a multiplicação

    3x2 = 6 6x2 = 12 12x 2 =24  24x2 = 48 48x2=96

    95+96 = 191


ID
162346
Banca
FCC
Órgão
TRT - 6ª Região (PE)
Ano
2006
Provas
Disciplina
Raciocínio Lógico
Assuntos

Observe que há uma relação entre os dois primeiros grupos de letras apresentados abaixo. A mesma relação deve existir entre o terceiro e quarto grupo, que está faltando.

DFGJ : HJLO :: MOPS : ?

Considerando que as letras K, Y e W não pertencem ao alfabeto oficial usado, o grupo de letras que substituiria corretamente o ponto de interrogação é

Alternativas
Comentários
  • Vou tentar "explicar" visualmente:D E F G H I JH I J L M N OM N O P Q R SQ R S T U V XPs. As que estão entre parênteses e mais claras são as letras que devemos "pular". As que estão sublinhadas são as que iniciam a próxima sequência de letras.:)
  • De outra forma:

    Entre o primeiro e o segundo grupo, a relação existente é:

    D + 4 letras= H

    F + 4 letras= J

    G + 4 letras= L

    J + 4 letras= O

    Logo, seguindo tal regra, o quarto grupo de letras será:

    M+ 4 letras= Q

    O+ 4 letras= S

    P+ 4 letras= T

    S + 4 letras= X

  • Observamos que o padrão existente é:
    • Os grupos ordenam-se da esquerda para a direita.
    • Uma letra inicia o grupo, em seguida pula-se uma letra e as duas seguintes
    ocupam a segunda e terceira posição. Pulam-se outras duas letras, e a letra seguinte ocupa a quarta posição.
    • O grupo seguinte inicia na letra do alfabeto seguinte à terceira letra do grupo anterior.
    Logo, a sequência formada é Q S T X.
  • Resolvo essa e outras questões similares aqui nesse vídeo

    https://youtu.be/_59ETIxrzXE

    Ou procure por "Professor em Casa - Felipe Cardoso" no YouTube =D


ID
162355
Banca
FCC
Órgão
TRT - 6ª Região (PE)
Ano
2006
Provas
Disciplina
Raciocínio Lógico
Assuntos

Sabe-se que os pontos marcados nas faces opostas de um dado devem somar 7 pontos. Assim sendo, qual das figuras seguintes NÃO pode ser a planificação de um dado?

Alternativas
Comentários
  • Somando os lados tem sempre que dar 7.
    Na letra B os lados errados são 2 e 1 = 3, quando deveria dar 7

    Resposta correta: Letra "b"
  • Desculpe mas não entendi, alguém poderia me ajudar????????
  • é só imaginar como se tivesse fechando o dado... ai a soma do numero com o do lado oposto, tem q ser 7.
  • SIMPLES


    É SÓ SOMAR DA SEGUINTE FORMA COMO SE FOSSE FECHAR O DADO


    SOMA  O 1º QUADRADO SUPERIOR COM O ULTIMO INFERIOR
    SOMA O 1 QUADRADO DA SEGUNDA FILEIRA COM O 3º QUADRADO DA MESMA FILEIRA
    SOME O 2º QUADRADO DA SEGUNDA FILEIRA COM O 4º QUADRADO DA MESMA FILEIRA




    VERÁ QUE A LETRA B TAIS SOMAS DARÃO


    SOMA  O 1º QUADRADO SUPERIOR COM O ULTIMO INFERIOR   ( 3+4 = 7)
    SOMA O 1 QUADRADO DA SEGUNDA FILEIRA COM O 3º QUADRADO DA MESMA FILEIRA (1+2=3)
    SOME O 2º QUADRADO DA SEGUNDA FILEIRA COM O 4º QUADRADO DA MESMA FILEIRAC (5+6 =11)
  • ACHEI O CONTRÁRIO! Q A ÚNICA QUE FORMARIA O DADO É A LETRA B POIS O DADO FECHADO, OS NÚMEROS EM SEQUÊNCIA FICARIAM EM LADOS OPOSTOS.

  • Resolvo essa questão aqui nesse vídeo

    https://youtu.be/hGSL09ggzZo

    Ou procure por "Professor em Casa - Felipe Cardoso" no YouTube =D

  • Resolvido:

    https://youtu.be/1dSWzlnJogs


ID
162358
Banca
FCC
Órgão
TRT - 6ª Região (PE)
Ano
2006
Provas
Disciplina
Raciocínio Lógico
Assuntos

Os termos da seqüência (2, 5, 8, 4, 8, 12, 6, 11, 16, ...) são obtidos através de uma lei de formação. A soma do décimo e do décimo segundo termos dessa seqüência, obtidos segundo essa lei, é

Alternativas
Comentários
  • Resposta : Letra a)

    1°  2°   3°   4°  5°  6°   7°    8°    9°   10°   11°    12°         

    2    5    8    4    8   12      11    16     x       y       z

       3    3          4    4          5      5              6        6

                   2                2                     2    

    x =  8        y  =  14      z  =  20

  • De outra forma:

    A regra da sequência é:

     +3 +3 metade  +4  +4  metade +5  +5  metade (...)

    O próximos termos da sequência serão:

    10° -> (metade de 16) 

    11° -> 14 (+6)

    12°->  20 (+6)

    Somando o 10° e o 12° = 8 + 20 = 28

  • 2 5 8     4 8 12       6 11 16      (+2)        8 (décimo)

    2 5 8     4 8 12       6 11 16      (+3)        14 ( décimo primeiro)

    2 5 8     4 8 12       6 11 16      (+4)        20  ( décimo segundo)

    Décimo + décimo segundo=

    8 + 20 = 28

  • 1º  2º  3º  4º  5º  6º   7º   8º    9º    10º         11º         12º
    2,  5,   8,  4,  8,  12,  6,  11, 16, __8__, __14_ , __20__

    Termo:
    1) 2+3 = 5
    2) 5+3 =8
    3) 8 - 4 = 4
    4) 4+4=8
    5) 8+4=12
    6) 12-6=6
    7) 6+5=11
    8) 11+5=16
    9) 11-8=8
    10) 8+6=14
    11) 14+6=20
    12) 20-10=10
    Nota-se que a seqüência é = +, -
    Na soma, soma-se + 1
    Na Subtração, soma + 2

    3 + 3; ( - 4) 4 + 4; ( - 6 ); 5 + 5; ( - 8 ); 6 + 6; ( - 8 )

    A soma dos termos:
    10º = 8
    +
    12º = 20
    Total = 28.
     

  • 2 MAIS 3 É 5
    5 MAIS 3 É 8
    8 DIVIDIDO POR 2 É 4

    4 MAIS 4 É 8
    8 MAIS 4 É 12
    12 DIVIDIDO POR 2 É 6

    6 MAIS 5 É 11
    11 MAIS 5 É 16
    16 DIVIDIDO POR DOIS É 8 ( DECIMO NUMERO)

    8 MAIS 6 É 14
    14 MAIS 6 É 20( DECIMO SEGUNDO NUMERO)

    20 MAIS 8 É 28 QUE É A RESPOSTA!


  •  A minha lógica foi a seguinte:

    Duas sequências de números adicionados (começando por 3) e uma de subtração (começando por quatro). A adição aumentava uma unidade em relação a outra e a subtração duas unidades, ou seja:

    +3, +3, -4, +4, +4, -6, +5, +5...

    A seuqência lógica então seria -8, +6 e +6.

    pode ser assim também não? Ou foi sorte? :P
  •  Marina... 

    Eu fiz do seu jeito também ! A soma, está subindo com os números em sequencia direta.  +3, +4, +5 ,=6

    A subtração, em seq dos números pares . -4, -6, -8, -10
  • Eu dividi em grupos de 3. O primeiro número de cada grupo é sempre duas unidades acima do 1º número do grupo anterior:

    1 º) soma-se 3 ao nº anterior
    2,5,8 

    2º) soma-se 4 ao nº anterior  (1º número: 2 + 2 = 4)
    4,8,12

    3º) soma-se 5 ao nº anterior (1º número: 4 + 2 = 6)
    6,11,16

    4º) soma-se 6 ao nº anterior (1º número: 6 + 2 = 8)
    8,14,20

    Assim: 10º = 8 e 12º = 20


  • (2, 5, 8, 4, 8, 12, 6, 11, 16...) 

    Vamos lá: 

    2 + 3 = 5 

    5 + 3 = 8 

    8 - 4 = 4 

    4 + 4 = 8 

    8 + 4 = 12 

    12 - 6 = 6 

    6 + 5 = 11

    11 + 5 = 16 

    Chegando até aqui já podemos perceber que as somas são feitas em sequência (algum número + 3, 4, 5, 6, 7, 8...) e as subtrações sempre de 2 em 2 (-4, -6, -8, -10...). Somamos duas vezes e subtraímos uma, sempre desse modo. Por isso: 

    16 - 8 = 8 (10º termo)

    8 + 6 = 14 

    14 + 6 = 20 (12º termo) 

    20 + 8 = 28

    Bons estudos e boa sorte!


  • 2, 5, 8, 4, 8, 12, 6, 11, 16...     814, 20... 

     

    2+2=4...        4+2=6...        6+2=8...      (soma sempre 2)

    5+3=8...       8+3=11...   11+3=14...      (soma sempre 3)

    8+4=12...   12+4=16...   16+4=20...      (soma sempre 4) 

     

    10º Termo: 8

    12º Termo: 20

     

    8 + 20 = 28

     

    GABARITO LETRA A: 28

  • Resolvido:

    https://youtu.be/fnQ97mGoyUM


ID
164299
Banca
FGV
Órgão
CODESP-SP
Ano
2010
Provas
Disciplina
Raciocínio Lógico
Assuntos

Observe a sequência numérica a seguir: "13527911413151761921238...".
Mantida a lei de formação, os dois próximos algarismos na sequência serão

Alternativas
Comentários
  • 1,3,5__(2)__7,8,11__(4)__13,15,17__(6)__19,21,23__(8)__25,...(25)Gabarito (a)
  • 1 3 5 2 7 9 11 4 13 15 17 6 19 21 23 8 ...                    (+2)

    1 3 5     7 9 11    13 15 17    19 21 23                           ( +2)

    Resposta:

     23 + 2 =25

  • A questão precisa muito mais de OLHO do que de CUCA.

    1 3 5
    2 7 9 11 4 13 15 17 6 19 21 23 8
     
    Observem que a ‘lei de formação’ é:
    1) colocar 3 números ímpares consecutivos;
    2) o próximo número será o 1º número par (no caso, o 2);
    3) colocar os próximos 3 números ímpares consecutivos, continuando os do item 1;
    4) colocar o próximo número par (repete o passo 3 e 4)
     
    Como, no final da seqüência, temos 3 números ímpares e 1 número par, o próximo número será ímpar! Como o último número ímpar foi o 23, o próximo será o 25!
  • O difícil é enxergar isso a tempo.


ID
164512
Banca
FGV
Órgão
BADESC
Ano
2010
Provas
Disciplina
Raciocínio Lógico
Assuntos

Em uma fila, denominamos extremos o primeiro e o último elementos e equidistantes os elementos que estão à mesma distância dos extremos.
A distância entre dois elementos consecutivos dessa fila é sempre a mesma, quaisquer que sejam esses dois elementos.
Sabendo que essa fila é formada por 52 elementos, o 8º elemento é equidistante ao:

Alternativas
Comentários
  • 1,2,3,4,5,6,7,8,..............,45,46,47,48,49,50,51,52
    ___________________,8_,7_,6_,5_,4_,3_,2_,_1

    Gabarito (b)
  • Outra maneira:
    Termos eqüidistantes: Ap + Aq = A1 + An
    Logo: 8 + X = 1 + 52
    X = 45
    Letra B
  • Uma dúvida  Gustavo, é pq quando eu fiz a primeira vez entendi que o a1=0. Eu tenho que começar a contar do a0 então ? Para poder a1=1?
  • Numa PA finita a soma dos termos distantes equidistantes dos extremos é igual a dos extremos, portanto:

    a1- 52
    a2- 51
    a3-50
    a4- 49
    a5- 48
    a6- 47
    a7-46
    a8- 45
  • A distância entre o termo dado (8) e o termo inicial (1) tem que ser a mesma entre o termo final (52) e o termo correspondente (x), logo:
    8-1 = 52-x
    x = 52 - 7
    x = 45
  • Não compliquem!

    Extremos: 1 e 52

    8-1=7

    52- x = 7 , logo, x = 45.

    Afinal, o número que se procura é tão equidistante do extremo correspondente quanto o  8. Só!

    Bons estudos!
  • 1º - Elemento: 1

    52º - Elemento: 52

    ----

    Fila:

    1,2,3,4,5,6,7,8...X...52

    A distância entre o 8º e o primeiro elemento é 7, pois 8-1 = 7.

    Logo, a distância entre o elemento equidistante é também 7, então: 52-7 = 45!


  • 1,2,3,4,5.......52

    pegando 51 + 1 que é o 1° e o ultimo numero =53

    8+x = 53

    x =53-8 = 45

  • Eu fiz assim, depois eu so chutei antes.

    1,2,3,4,5,6,7,8= 7 espaços entre eles: então conclui que era pra subtrair de 52. Resultado 52-7=45 Ta certo tmb?


ID
175258
Banca
FCC
Órgão
TRT - 9ª REGIÃO (PR)
Ano
2010
Provas
Disciplina
Raciocínio Lógico
Assuntos

Considere o conjunto:

X = {trem, subtropical, findar, fim, preguiça, enxoval, chaveiro, ...}, em que todos os elementos têm uma característica comum. Das palavras seguintes, a única que poderia pertencer a X é:

Alternativas
Comentários
  • o elemento em comum entre todos os elementos de x é que não tem letras repetidas. Dentre as alternativas, o único elemento que não repete letras é PELICANO. Portanto, a letra "a" é a alternativa correta.

  • A característica comum dos elementos é "não ter letra repetida", portanto, a única palavra que não tem letra repetida é PELICANO.

    LETRA (A)

  • É difícil enxergar isso na hora da prova.
  • Qual ponto de partida para fazer questões desse tipo? eu geralmente vejo ordem das palavras, letras iniciais, silabas, quantidade de letras, significado, material, imaterial... poucas são as vezes que encontro o resultado.

  • Para quem mesmo após os comentários ficou boiando(como eu) veja a resoluçao..

    http://www.youtube.com/watch?v=ZGFhJpUDNXk

  • AGORA TA EXPLICADO

  • Um enunciado mais apropriado para essa questão é "Desvende, por supostos meios sobrenaturais, o que eu, elaborador, estou pensando"

  • Uma pena que não tem a palavra MISERICÓRDIA. :/

  • Letras que não se repetem.

     

    Na palavra PELICANO há letras distintas, sem repetição.

     

    b) FORMOSURA.

     c) SOBRENATURAL.

     d) OVO.

     e) ARREBOL.

  • Questão um tanto prejudicial, eu mesmo estava observando a possibilidade de existir o DITONGO, TRITONGO em comum kkkkkkk MEU PAI AMADO!

  • Prova viva que o examinador coloca as coisas mais malucas e temo que adivinhar. Vou providenciar uma bola de cristal. quem sabe não consigo uma aprovação 100%?

  • Colegas, isso é tão viajado que eu acertei por outros motivos:

    eu percebi a letra “ i ” em todas as palavras, ainda que oculta, como em: treim; einxoval - o que acontece quando uma vogal é nasalizada por “m” ou “n”.

    Pelicano é a única palavra com “ i ”.

    Que eu tenha essa sorte na prova.

  • Tem que dar um trago antes de responder, igual quem fez a questão.


ID
175261
Banca
FCC
Órgão
TRT - 9ª REGIÃO (PR)
Ano
2010
Provas
Disciplina
Raciocínio Lógico
Assuntos

Em um ambulatório há um armário fechado com um cadeado cujo segredo é um número composto de 6 dígitos. Necessitando abrir tal armário, um funcionário não conseguia lembrar a sequência de dígitos que o abriria; lembrava apenas que a soma dos dígitos que ocupavam as posições pares era igual à soma dos dígitos nas posições ímpares.

As alternativas que seguem apresentam sequências de seis dígitos, em cada uma das quais estão faltando dois dígitos. A única dessas sequências que pode ser completada de modo a resultar em um possível segredo para o cadeado é:

Alternativas
Comentários
  • Esta questão pode ser resolvida por eliminação. É correta a alternativa "e", pois 2 + 4 + 8 = 14 (posiçoes impares); se a soma das posições pares deve ser igual a das posições impares, então, 14 - 6 (nº da posição 2) = 8; sendo os números diferentes, usamos 5 + 3; portanto o número é 2 6 4 5 8 3.

  •  Minha forma de resolver a questão, que deu certo:

    9 2 _ _ 6 2               ->        9+?+6 (mínimo de 15) = 2+?+2 (máximo de 13): Não pode ser

    7 _ 7 _ 7 1               ->        7+7+7 (21) = ?+?+1 (máximo de 19): Não pode ser

    6 _ 9 0 _ 5               ->        6+9+? (mínimo de 15) = ?+0+5 (máximo de 14): Não pode ser

    4 8 _ 9 _ 7               ->        4+?+? (máximo de 22) = 8+9+7 (23): Não pode ser

    2 6 4 _ 8 _               ->        2+4+8 (14) = 6+?+? (mínimo de 6 e máximo até 24): Pode ser compatível

     

    Não tem fórmula, mas foi o jeito que deu certo para encontrar a resposta. Quando atribuído o mínimo possível considerava-se o 0 e ao máximo o 9.

    Bons estudos!

  • Somando-se os valores das posições ímpares e igualando-se à soma dos valores das posições pares, temos:

    a) 9 2 x y 6 2  =>    9+x+6 = 2+y+2, logo: y = x+11 => y seria um número com 2 algarismos!!!! não pode! (Tem que ser de 0 a 9)

    b) 7 x 7 y 7 1  =>    7+7+7 = x+y+1, logo: x+y = 20 => pelo menos um dos valores terá 2 algarimos!!!! não pode!

    c) 6 x 9 0 y 5  =>     6+9+x = y+0+5, logo: y = x+10 => y seria um número com 2 algarismos!!!! não pode!

    d) 4 8 x 9 y 7  =>     4+x+y = 8+9+7, logo:  x+y = 20 => pelo menos um dos valores terá 2 algarimos!!!! não pode!

    e) 2 6 4 x 8 y  =>     2+4+8 = 6+x+y, logo: x+y = 8 => os dois valores estarão entre 0 e 9! Logo, esta é a única sequência que pode ser completada
  • Se denotarmos por X e Y os algarismos faltantes, em cada alternativa temos:

    (lembrar que X e Y pertencem a {0, 1, 2, 3, 4, 5, 6, 7, 8, 9)}

    a) 92XY62 => 9 + X + 6 = 2 + Y + 2 => Y - X = 11. Não há X e Y que satisfazem a essa igualdade.

    Alternativa incorreta.

    b) 7X7Y71 => 7 + 7 + 7 = X + Y + 1 => X + Y = 20. Não há X e Y que satisfazem a essa igualdade.

    Alternativa incorreta.

    c) 6X90Y5 => 6 + 9 + Y = X + 0 + 5 => X - Y = 11. Não há X e Y que satisfazem a essa igualdade.

    Alternativa incorreta.

    d) 48X9Y7 => 4 + X + Y = 8 + 9 + 7 => X + Y = 20. Não há X e Y que satisfazem a essa igualdade.

    Alternativa incorreta.

    e) 264X8Y => 2 + 4 + 8 = 6 + X + Y => X + Y = 8. É possível ter X e Y satisfazendo a essa igualdade. Tome, por exemplo, X = 0 e Y = 8, ou X = 3 e Y = 5, dentre outras combinações.

    Alternativa correta.

    Resposta: e.

    Opus Pi.

     

  • ..."lembrava apenas que a soma dos dígitos que ocupavam as posições pares era igual à soma dos dígitos nas posições ímpares."

    Ou Seja, o enunciado quis dizer: vermelho com vermelho, azul com azul (soma de posições pares com pares e impares com impares). Após, temos que igualar as duas cores:

    a) 9 2 y 6 2 =>   9+x+6 = 2+y+2, logo: y = x+11 => y seria um número com 2 algarismos. ERRADO.

    b) xy 7 1  =>   7+7+7 = x+y+1, logo: x+y = 20 => pelo menos um dos valores terá 2 algarimos. ERRADO.

    c) x 9 0 y 5  =>    6+9+x = y+0+5, logo: y = x+10 => y seria um número com 2 algarismos. ERRADO.

    d) 4 8 x 9 y 7 =>    4+x+y = 8+9+7, logo: x+y = 20 => pelo menos um dos valores terá 2 algarimos. ERRADO.

    e) 2 6xy =>    2+4+8 = 6+x+y, logo: x+y = 8 => os dois valores se encaixam exatamente entre 0 e 9! CORRETO.

    E correr pro abraço!


ID
204397
Banca
FEC
Órgão
MPA
Ano
2010
Provas
Disciplina
Raciocínio Lógico
Assuntos

A sequência de números: 0, 3, 8, 15, 24, ..., obedece a uma certa regra lógica. Continuando essa sequência, a soma dos três próximos termos será igual a:

Alternativas
Comentários
  •      

          Observando atentamente os números vê-se que eles obedecem o quadrado menos um, baseado na formula (X² -1)

          Portanto o números seguintes serão:       6² - 1 =  35

                                                                                7² - 1 =  48

                                                                                8² - 1 =  63

                                                                                           ______

                                                                                                                          146

  • pode ser tambem 

    0

    0 +3= 3

    3 +5= 8

    8 +7= 15

    15+9=24

    24+11=35

    35+13=48

    48+15=63  

    é só ir somando cada numero com o proximo numero impar

    soma-se os ultimos tres termos 35+48+63=146

  • 24+11=35

    35+13=48

    48+15=63

    Vai aumentando em cada sentenças 2 números

    ex : 3,5,7,9 ...

     

    Resposta Letra B

    Bons Estudos Pessoal !!

     

    Paulo.

     

  • Meu raciocínio, foi o mesmo do Alexandre.

ID
219625
Banca
FCC
Órgão
BAHIAGÁS
Ano
2010
Provas
Disciplina
Raciocínio Lógico
Assuntos

Observe a sequência que foi criada com uma lógica matemática:

7; 29; quarenta;
8; 11; vinte;
3; 31; trinta;
5; 73; oitenta;
6; 52; .......

A palavra que completa o espaço é:

Alternativas
Comentários
  • Essa foi a explicação de um colega. Não me convenceu muito, mas como não achei outra melhor...

    Somam-se os 2 números e o resultado vc arredonda para a dezena mais próxima (arredondamento):

    7+29 = 36 - arrendonda p/ 40 (dezena mais próxima)
    8+11 = 19 -->arredonda p/ 20
    3+31 = 34 --> arredonda p/ 30
    5+73 = 74 --> arredonda p/ 80
    6+52 = 58 --> arredonda p/ 60

    Logo, a resposta seria sessenta (letra b)

  • Eu fiz essa questão assim :

    quarenta pra vinte são vinte de diferença

    então oitenta diminuindo vinte é sessenta

     

    Resposta Letra B

    Bons Estudos Pessoal !!

    Paulo.

  • Eu acredito q os números da frente sejam só pra causar confusão;;;

    fui pela lógica tipo: 40 30 20 põe o menor no meio nesse caso 20 ...nessa lógica depois de  80 70 60 pondo o menor no meio 60...

    pelo menos assim deu certo rsrs!!!!

  • Também fui pelo raciocínio de arredondar para a dezena mais próxima da soma dos números. Isso levando em conta o número  abaixo ou acima de 5 (cinco).

    7+29 = 36 - arrendonda p/ 40 (dezena mais próxima)
    8+11 = 19 -->arredonda p/ 20
    3+31 = 34 --> arredonda p/ 30
    5+73 = 74 --> arredonda p/ 80
    6+52 = 58 --> arredonda p/ 60
    GABARITO "B"
    Creio eu que seja desse jeito, é a maneira mais lógica possível.
    Bons estudos!
  • Observe que:

    A dezena mais próxima de 7+ 29 é quarenta;
    A dezena mais próxima de 8+11 é vinte;
    A dezena mais próxima de 3+ 31 é trinta;
    A dezena mais próxima de 5+ 73 é oitenta;
    A dezena mais próxima de 6 + 52; sessenta.

  • No meu caso, encontrei a resposta dessa maneira:

    7; 29; quarenta; - A palavra "quarenta" possui 08 Letras;
    8; 11; vinte; - A palavra "vinte" possui 05 letras;
    3; 31; trinta; - "Trinta" possui 06 letras;
    5; 73; oitenta; - "Oitenta" possui 07 letras;
    6; 52; ....... - E no caso da proxima palavra seria uma com 08 letras que no caso é "sessenta".

                          E se o raciocinio estiver certo, as proximas  sequencias recomeçaria o ciclo de 05, 06, 07 e 08 letras.
     
  • não entendi essa questão alguém  pode por favor explicar detalhadamente!!! Obrigada
  • Questão mais sem lógica !

    Uma hora vcs arredondam para cima, outra hora para baixo !!

    Kd a lógica ?!

  • GABARITO: B

    Aqui temos que perceber (= adivinhar) a lógica que o examinador usou. Então podemos notar o seguinte:
    7 + 29 = 36, arredondando: quarenta;
    8 + 11 = 19, arredondando: vinte;
    3 + 31 = 34, arredondando: trinta (valor menor que 35);
    5 + 73 = 78, arredondando: oitenta
    Portanto: 6 + 52 = 58, arredondando: sessenta


    OBSERVAÇÃO: Esta questão foi explicada pelo professor Jairo Teixeira no Curso de Questões de Raciocínio Lógico para a banca FCC no site Eu Vou Passar (vídeo 46, na parte que trata de sequências lógicas).

    E lembrem-se: "A exaustão faz o Samurai".
  • Questão ridícula

  • Gente que questão mais tosca  O.O passei um tempão tentando adivinhar essa bendita lógica.

  • que mongoloide essa fundação copia e cola

  • Questão ridícula, não há padrão nenhum nesses arrendondamentos

  • VALEU GALERA VOU VIRAR UM SAMURAI.

  • Arredondei tudo e já foi

  • Recomendo um curso de Clarividência. O humano comum no nervosismo da prova e outras coisinhas, não consegue ver a lógica da questão. EXplicaram, mas nada convence dessa bendita lógica. Loucura, loucura, loucura.

  • Isaías 8;11-20

    11 O Senhor falou comigo com veemência, advertindo-me a não seguir o caminho desse povo. Ele disse:

    12 "Não chamem conspiração a tudo o que esse povo chama conspiração­; não temam aquilo que eles temem, nem se apavorem com a FCC.

    Só assim pra acertar isso.

  • Resolvo essa questão aqui nesse vídeo

    https://youtu.be/pTvyACz_dLo

    Ou procure por "Professor em Casa - Felipe Cardoso" no YouTube =D


ID
233671
Banca
FCC
Órgão
MRE
Ano
2009
Provas
Disciplina
Raciocínio Lógico
Assuntos

Considere a sucessão dos números naturais múltiplos de 3, dispostos na seguinte forma:

0 3 6 9 1 2 1 5 1 8 2 1 2 4 2 7 3 0 3 3 3 6 3 9 . . .

Nessa sucessão, o algarismo que deve ocupar a 126ª posição é

Alternativas
Comentários
  • De 0 até 9 temos quarto algarismos.

    De 12 até 99, temos quantos multiplos de 3? Aplicando o termo geral de PA, temos: 99 = 12 + (n-1).3; n = 30. Logo temos 60 algarismos aqui.

    Já temos 64 (4 + 60) algarismos, faltam 62 (126-64) para chegar a resposta, todos esses agora, agrupados três a três, continuam multiplos de 3.

    62 algarismos geram 20 (60:3, com resto 2) números de três algarismos, sobrando DOIS algarismos para o 21º número de três digitos. Então, quem seria esse 21º número? Outra vez, usando o termo geral de PA: An = 102 + (21-1).3; An = 162.

    Assim, o algarismo procurado é o 162.

  • 0369121518212427303336394245485154576063666972757881848790939699102105108111114117120123126129132135138141144147150153156159162

  • 126 dividido por 3 é igual a 42,que 4 mais 2 é igual a 6.

  • Mais uma resolução para nos ajudar:

    Quantidade de múltiplos de 3 com um algarismos: 4 (= (9 - 0)/3 + 1)
    Quantidade de múltiplos de 3 com dois algarismos: 30 (= (99 - 12)/3 + 1)
    Quantidade de múltiplos de 3 com três algarismos: 300 (= (999 - 102)/3 + 1)

    Até o 34.° múltiplo de 3 temos 64 (=4 + 30*2) algarimos. Faltam 62. Mas 62 = 20*3 + 2. Isso significa que o algarismo desejado é o segundo algarismo do 21.° múltiplo de 3 com três algarismos. E esse múltiplo (vou chamar de X) é o 21.° termo da seqüência (uma PA de razão 3) 102, 105, 108, ..., 999, tal que 21 = (X - 102)/3 + 1. Logo, X = 162, cujo segundo algarismo é o 6.

    Resposta: 6 (letra A)


    Professor Opus Pi

    http://www.forumconcurseiros.com/forum/showthread.php?t=240757  
  • Caros, não foi possível entendê-los. Isso porque a questão quer saber qual número estará na 126ª posição. Se na 2ª posição temos o 3; Na 3ª posição o 6; e sucessivamente, quando chegarmos à posição 126 teremos o número 375. O gabarito nem chegou perto. Vejo que, em suas respostas, conta o nº 6, que está sendo escolhido aleatoriamente de um número maior. Qual é a lógica nisso???
  • Jéssica, a questão busca o qual Algarismo, não o número.
    Como pode ver, a sequência dos múltiplos de 3 foi considerando, após o número 9, cada número como 2 algarismo
    Veja:
    0, 3, 6, 9, 1, 2, 1, 5, 1, 8 (...)
                      (12)  (15) (18)

    Dessa forma, o ideal é fazer como no primeiro comentário:
    Primeiro vemos quantos números de 1 algarismos são (4 números - 0, 3, 6, 9)
    Depois quantos de 2 algarismos (12 - 99) --> Perceba que o número de múltiplos que existir aqui, você deve multiplicar por 2 (pois busca-se os algarismos, não os números)
    Depois quantos de 3 algarismos (102 - X) --> Aqui faça o mesmo, mas por três

    Usando a formula da PA

    An = A1 + (n-1) * r
    99 = 12 + (n-1) * 3
    n= 30 Números... Como queremos os algarismos, multiplica-se por 2 = 60

    Desta forma ja temos 64 algarismos... Queremos achar o 126
    126 - 64 = 62 algarismos faltam
    62/3 (já que são números de três algarismos) = 20 ( com 2 de resto) --> Ou seja, passaremos por + 20 números completos e andaremos mais dois algarismos do 21o número

    Usando a PA
    An = 102 + (20-1) * 3 --> (Obs: 102 é o primeiro dos múltiplos de 3 com 3 algarismos)

    An = 159, Ou seja, o Vigésimo número completo é o 159
    Mas lembra que andamos + dois (sobrou 2 de resto)?
    Então --> 159, 162 --> O segundo algarismo do próximo número é "6"


    Méritos aos que responderam antes... Eu apenas tentei colocar de uma forma mais explicadinha

    Abs.
  • Reescrevendo a sequência da seguinte maneira, temos:

    0             (1ª posição)   
    3             (2ª posição)     
    6                .
    9                .
    12           (Algarismos 1 e 2 ocupando respectivamente a 5º e 6º posições)
    15              .
    18              .
    21              .
    24              .
    .                 .
    .                 .
    .                 .
    Assim por diante.
    Para não fazer tudo na mão, reparem que:
    De 0 → 9, temos 4 x 1 algarismos = 4
    De 12 → 99, podemos usar uma PA para fazer este cálculo:
    PA→ 99 = 12 + (n - 1) x 3 → 87 / 3 = N - 1→ n = 30
    Logo, 30 x 2 algarismos = 60
    De 102 → 99, temos:
    Y x 3 algarismos = 62
    pois 4 + 60 + 62  = 126 algarismos, então:
    Y x 3 = 62 → Y = 62 / 3
    Como Y não é um número exato, basta acrescentarmos 1 algarismo em 62, assim:
    Y = 63 / 3 = 21. 
    Substituindo esse valor na PA, onde a1 = 102 (primeiro número múltiplo de 3 com três algarismos) e n = 21, temos:

    a21 = 102 + (21-1) x 3
    a21 = 162

    O número seria dois, mas lembrem-se que adicionamos 1 unidade para que a divisão acima desse exata, logo a posição 126ª será o número 6.



    Resposta: Alternativa A.
  • eu fiz assim: 4 numeros de 1 digito, PA pra descobrir quantos de 2 digitos: 99=12+(n-1)x3; n=30; 60 algarismos. daí ja sao 64 algarismos, subtraídos de 126, dá 62 de 3 digitos, dividindo por 3= 20 sobra 2. 

    beleza, entao sao 54 números, e ele quer saber o 2o algarismo do número 55

    entao precisamos descobrir qual o numero 55 e ver qual o algarismo do meio. 

    PA: x=0+(55-1)x3; 54x3=162. esse eh o 55º numero, seu algarismo do meio é o 6. 

  • 03691215182124273033 (20 algarismos) 36394245475053565962 (40 algarismos) 65687174778083868992 (60 algarismos)


    95981011041071101131 (80 algarismos) 16119122125128131134 (100 algarismos) 13714014314715015315 (120 algarismos)


    615916 (126 algarismos)


    levei menos tempo na mao que esse povo fazendo conta....


ID
240424
Banca
FCC
Órgão
TRT - 22ª Região (PI)
Ano
2010
Provas
Disciplina
Raciocínio Lógico
Assuntos

No esquema abaixo, considere a relação existente entre o primeiro e o segundo grupos de letras, a contar da esquerda. A mesma relação deve existir entre o terceiro grupo e o quarto, que está faltando.

A C E B : D F H E :: L N P M : ?

O grupo de letras que substitui corretamente o ponto de interrogação é

Alternativas
Comentários
  • Alfabeto:

    A B C D E F G H I J K L M N O P Q R S T U V W X Y Z

    A C E B
    D F H E

    Repare que ele pega a primeira primeira posição A e adicionais "+3" para achar o resultado, ou seja, A + 3 (B, C, D) = D; e assim sucessivamente, vejamos:

    A + 3 = D
    C + 3 = F
    e + 3 = H
    b + 3 = E

    Agora é só fazer isso com a segunda parte:


    L + 3 = O
    N + 3= Q
    P+ 3= S
    M +3= P

    Ficando: O Q S P - Letra C


     

  • Ou então poderíamos perceber que os grupos são formados pelas letras do alfabeto na seguinte ordem:

    5ª 2ª :: 6ª 8ª 5ª

    então:

    L N P M :: O Q S P
  • A C E B - D F H E - L N P M
    A cada sequência anda-se três casas. Veja: A + 3 casinhas = 4. Qual a quarta letra do alfabeto contando com A? D ( A B C D). Logo, a próxima sequência irá começar com D.
    A partir desses quatro pontinhos é outra sequência, não tem nada a ver com a anterior, ok?!
    Vamos à segunda:
    L N P M: seguindo a mesma regra da anterior, L + 3 = 4. Qual é a quarta letra contando com L? O ( L M N O). Logo, a próxima sequência começará com a letra O.
    Resposta correta: letra C.
    Ai vocês me perguntam, mas tem duas alternativas com a letra O. Vejam agora como são as sequências ordenadas:
    ABC_E
    DEF_H
    LMN_O
    Viram que ele conta em ordem alfabética até a terceira, pula uma letra, e depois escreve a próxima letra. A mesma coisa será na próxima sequência.
    Letra C: OPQ_S
    Vejam que não pode ser a letra d: OPRT. Foge da regra, não é mesmo? Portanto, a correta mesmo é a letra C.












  • Solução: Vejam que de A para D, avançamos 3 casas. O mesmo vai acontecer de L para a próxima, ou seja, vai para a letra O. De C para F nas duas primeiras sequências, avança apenas três casas. Dessa forma, de N para a próxima, avança três também, logo vamos para a letra Q. A sequência será OQSP .

    Resposta: alternativa C
    http://www.folhape.com.br/blogdosconcursos/?p=4916

  • GABARITO: C

    Este é o tipo de questão em que temos de usar de toda a nossa percepção, é meio que uma "adivinhação lógica", um teste psicotécnico mesmo, rs....

    Como o colega acima explicou, ao analisarmos cuidadosamente as letras da primeiras sequência (A C E B ) e depois as da segunda sequência (D F H E ) percebemos que elas pulam de 3 em 3.

    O mesmo deverá acontecer com a sequência L N P M. A sequência seguinte será O Q S P. Respeitando a ordem do alfabeto brasileiro.

ID
245998
Banca
FMZ - AP
Órgão
SEAD-AP
Ano
2010
Provas
Disciplina
Raciocínio Lógico
Assuntos

O próximo número da sequência 125, 131, 167, 383 é

Alternativas
Comentários
  • 125 a 131 tem diferenca de 06 , de 131 a 167 difernca de 36. de 167 a 383 diferenca de 216 proximo numero obedece o padrao 6x216 +383 = 1679.OBS:  o padrao da seguencia e o seguinte: 6x6  = 36x6 =216x6 =1296 +383.

  • encontrei 1296, pois:
    131 - 125 = 6 (x6 = 36)
    167 - 131 = 36 (x6 = 216)
    383 - 167 = 216 (x6 = 1296)
      x    - 383 = 1296

    x = 1296 + 383
    x = 1679
  • Pensa assim:

    125   131   167   383

    a próxima sequência teria que ser 1 x x 9

    blz!
  • Soma-se sempre o 6 elevado ao próximo número..

    125 + 61 = 131
    131 + 62 = 167
    167 + 63 = 383
    383 + 64 = 1679

ID
247288
Banca
FCC
Órgão
TRT - 12ª Região (SC)
Ano
2010
Provas
Disciplina
Raciocínio Lógico
Assuntos

Considere os seguintes grupos de letras:

A B C A - J K L J - D E F D - N O Q N - T U V T

Desses grupos, o único que NÃO tem a mesma característica dos demais é:

Alternativas
Comentários
  • resposta letra D.
    se mantivesse o padrão seria: N O P N, e não N O Q N.
  • A questão cria grupos (ou conjuntos) formados pela sequencia alfabética com a repetição do primeiro elemento na quarta posição.
    Assim temos:

     A B C A
    J K L J
    D E F D
    N O Q N (deveria ser: N O P N)
    T U V T

    portanto, resposta é a letra D.
  • A lógica apresentada pelos colegas está corretíssima! Entretanto, entendo que a questão da margem a uma interpretação diferente e que leva a outra alternativa correta. O segundo grupo de letras é o único que não possui vogal. Portanto, é o único grupo que não tem como característica a presença de uma vogal.
  • A Banca coloca como referência para a resolução da questão as letras do anunciado e ao mesmo tempo coloca como opção de resposta. Pois a questão foi mal elaborada , deixando vago a que se pede para a resposta.

  • A B C A

    J K L J

    D E F D

    N O Q N [N O (D) N]

    T U V T


ID
275194
Banca
COMPERVE
Órgão
UFRN
Ano
2010
Provas
Disciplina
Raciocínio Lógico
Assuntos

A letra que ocupa a 1248ª posição na sequência A, B, C, D, E, A, B, C, D, E, A, B, C, D, E,... é:

Alternativas
Comentários
  • nesses casos deve-se dividir a posição pedida (1248) pelo numero da sequencia.... no caso percebe que ela se repete de 5 em 5 (a,b, c, d, e) ao fazer a divisão restará 3.... ai vc conta mais três casas que no caso é a letra C

  • Nesse caso quebrei a cabeça menos ainda..

    se a letra "E" e igual a 5° posição,por conseguinte 1245° é igual a "E" logo:

    1248° é igual a "c".

    gabarito letra D.

  • 1248/5 = 249 - restando 3 unidades. então letra C - resposta D.

  • ASSERTIVA D 


ID
284566
Banca
FCC
Órgão
TCE-SP
Ano
2010
Provas
Disciplina
Raciocínio Lógico
Assuntos

A seguinte sequência de palavras foi escrita obedecendo a um padrão lógico:

PATA - REALIDADE - TUCUPI - VOTO - ?

Considerando que o alfabeto é o oficial, a palavra que, de acordo com o padrão estabelecido, poderia substituir o ponto de interrogação é

Alternativas
Comentários
  • A ÚLTIMA LETRA DE CADA PALAVRA SEGUE A SEQUENCIA DAS VOGAIS DO ALFABETO:

    PATA - REALIDADE - TUCUPI - VOTO - XAMPU
  • PATA - QREALIDADE - S -  TUCUPI - U -  VOTO - W -  XAMPU



    E tem esse outro padrão...
  • Resolvi assim!

    Notei que a primeira letra de cada palavra segue o alfabeto na ordem de 3 em 3, vejamos:

    PATA -  P,Q,R  REALIDADE-  R,S,T  TUCUPI- T,U,V  VOTO V,W,X XAMPU letra d)




  • 1º JEITO DE RESOLVÊ-LA:
    PATAREALIDADETUCUPI VOTO − ?
    A primeira palavra tem 4 letras, começa com P e termina com A.
    A segunda palavra tem 9 letras, começa com R e termina com E.
    A terceira palavra tem 6 letras, começa com T e termina com I.
    A quarta palavra tem 4 letras, começa com V e termina com O.
    Vendo as alternativas, encontramos a palavra XAMPU terminando em U. XAMPU
    2º JEITO DE RESOLVÊ-LA:
    Percebam a primeira letra: P-R-T-V. O alfabeto é P-Q-R-S-T-U-V-W-X, a questão propõe P-Q-R-S-T-U-V-W-X. Viram que a questão pula uma letra?? Essa é outra maneira de resolvê-la. 
  • Eu vi uma sequência que, apesar de ser lógica, me levou a resposta errada. Eu numerei as letras e achei entre as palavras uma sequencia decadêncial crescente como podem ver abaixo onde represento em vermelhos os nº referêntes as letras e em azul a diferença entre a ultima letra de um para a primeira da palavra seguinte


     P16 A1 T20 A1   16  R17E5A1L11I9D4A1D4E15  T20U21C3U21P16I9   13  V22O15T20O15  

    16   -1    15   -2    13  -3    ?= 10

    Ou seja a proxima letra seria a da posição 25, logo seria Y

    Acabei marcando a E, mas como podem ver tem lógica o pensamento e tinha uma opção que corroborava com ele.
  • Meu erro foi colocar  Pata - Q - Realidade - S - Tucupi - U - Voto - X - Yesterday

  • De acordo com a Wikipedia o "V" vem sim antes do "W". Sei que não é uma fonte confiável, mas já é um indício. Lá é falado que o "W" foi incorporado ao alfabeto português em 2009.

    http://pt.wikipedia.org/wiki/Ordem_alfab%C3%A9tica
  • PATA - REALIDADE - TUCUPI - VOTO - XAMPU  ou seja  A E I O U ELTON CARVALHO

  • Eu vi outra sequência que também é lógica e alguns exercícios seguem este padrão:


    PATA - 2 consoantes - 2 vogais (A)

    REALIDADE - 4 consoantes  - 5 vogais (B)

    TUCUPI - 3 consoantes - 3 vogais (C) 

    VOTO - 2 consoantes - 2 vogais (A)

    QUALIDADE 4 consoantes - 5 vogais  , mesmo padrão de REALIDADE

    Questão passível de recurso?



  • P  Q  R  S  T  U  V  W  X  Y  Z

    As iniciais de cada palavra segue o padrão:  pula uma letra   na ordem alfabética.

  • Gabarito: D

    Sequencia das vogais:

    patA - realidadE - tucupI - votO - xampU

    A - E - I - O - U

  • Letra que inicia palavras Pata - Q - Realidade - S- Tucupi - U- Voto - W - (X de Xampu )....seguir ordem do alfabeto

  • Resolvo essa e outras questões similares aqui nesse vídeo

    https://youtu.be/_59ETIxrzXE

    Ou procure por "Professor em Casa - Felipe Cardoso" no YouTube =D


ID
346039
Banca
ESAF
Órgão
SMF-RJ
Ano
2010
Provas
Disciplina
Raciocínio Lógico
Assuntos

A partir da lei de formação da sequência 1, 1, 2, 3, 5, 8, 13, 21,..., calcule o valor mais próximo do quociente entre o 11° e o 10° termo.

Alternativas
Comentários
  • A sequencia sempre  o sucessor é somando com antecessor

     1, 1, 2, 3, 5 ,8, 13, 21, 34, 55, 89

    89/55= 1,618

  • Vamos lá;
    1, 1, 2, 3, 5, 8, 13, 21, 34, 55, 89

    O Terceiro termo é a soma dos dois antecessores e, assim por diante. Ou seja, 3°=2°+1°; 4º=3°+2°; 6°=5°+4°; 10°=9°+8°; 11°=10°+9º

    Logo, a divisão será; 89/55=~1,6181

    Letra C

    até mais!
    ;)
  • Para quem não sabe essa é a famosa sequência de FIBONACCI, basteante recorrente em provas.
  • 1, 1, 2, 3, 5, 8, 13, 21,....
    1+1=2
    1+2=3
    2+3=5
    3+5=8

    13+21=34
    21+34=55
    34+55=89
    89/55=1,618
  • Resolução no YouTube:

    http://www.youtube.com/watch?v=1rblpiFzBp4
  • Já reconhecendo que se trata da sequência de Fibonacci nem se faz necessário cálculo algum, visto que que a divisão entre um termo e seu sucessor converge para o número áureo, também conhecido como relação de ouro ou mesmo razão áurea: 1,618...
  • Pra agregar mais valor: essa é a razão áurea, o número de ouro da matemática.

  • Diêgo Lima

    Cara,muito obrigado pela explicação. Nunca pensei que seria desta forma a resolução. 
    O professor no pdf deu uma bosta de explicação. Muito lógica e pouco didático. 
    Valeu man!
  • Observe que cada termo desta sequência, a partir do terceiro (2), é igual à soma dos dois termos anteriores. Continuando a preencher a sequência, com esta mesma lógica, temos:

    1, 1, 2, 3, 5, 8, 13, 21, 34, 55, 89,...

                   O quociente entre o 11º e o 10º termos é:

    Quociente = 89 / 55 = 1,618

    Resposta: C


ID
348775
Banca
FGV
Órgão
CODESP-SP
Ano
2010
Provas
Disciplina
Raciocínio Lógico
Assuntos

Observe a sequência numérica a seguir: “13527911413151761921238...”.
Mantida a lei de formação, os dois próximos algarismos na sequência serão

Alternativas
Comentários
  • 2 – Lei de formação das seqüências 
     
    Termo em função da posição 
    Expressa an em função de n. 

    Exemplo:




    1 3(a1) 5(a2) 2 7(a3) 9(a4) 11(a5) 4 13(a6) 15(a7) 17(a8) 6 19(a9) 21(a10) 23(a11) 8 25(a12)
    resposta

    a12= 2.12+1 = 25
  • Tem duas sequências entrelaçadas nessa "sequência numérica". Observe:

    13527911413151761921238...

    A sequência de azul é fácil de visualizar. São os números pares a partir do 2.
    A sequência de vermelho, por sua vez, são os númeos ímpares a partir do 1.

    O próximo número ímpar, depos do 23, é o 25.

  • Complementando o excelente comentário do Douglas:

    A lógica é que a cada três números ímpares (com um ou mais algarismos cada número) aparece um número par.
    • 1
    • 3
    • 5
    • Daí vem o número par: 2
    • 7
    • 9
    • 11
    • Daí vem o par: 4
    • 13
    • 15
    • 17
    • Par: 6
    E assim vai...
  • Se eu tivesse feito essa prova, entraria com um recurso, pois a diagramação não permite o correto entendimento da sequencia... não fica claro se estamos diante, por exemplo, de 1, depois 3  ou de um 13.
  • Eu achei ótima a questão e a explicação do Douglas também.
    O lance é achar uma lógica, uma razão entre os números.
    Pares> razão 2
    Ímpares> razão 1
    Eles só intercalaram pares e ímpares para dificultar. :)
  • Acho assim mais fácil:

     

    http://sketchtoy.com/66824822

  • Questão ridícula, não avalia o conhecimento de ninguém

     

    Vou postar aqui o comentário do Prof. Guilherme Neves do Ponto dos Concursos:

    "A lei de formação é a seguinte: escreva 3 números ímpares, escreva um
    número par. Observe:
    1 3 5 2 7 9 11 4 13 15 17 6 19 21 23 8...
    O próximo número ímpar a ser escrito é 25."

  • 13527911413151761921238

    Ao olharmos temos que identificar os comuns. 

    Nota-se que os 3 primeiros números são ímpares é que o quarto seguinte é par

    Depois vem uma sequencia de quatro impares é o seguinte um par que é multiplo de 2 do primeiro número par entcontrado e que os números ímpares que estão na frente são a continuação dos 2 primeiros {135(impar)2(par)7911 (impar)...}... 

    Loga vamos para os últimos impares da sequencia 19 21 23 que aparece em seguida um par e logo depois a continuação dos impares que seria 25 (se continuasse 21 29...) 

    Então... 

    135 impar

    2 par

    7 9 11impar

    4par

    13 15 17impar

    6 par

    19 21 23impar

    8par


ID
357376
Banca
CESGRANRIO
Órgão
Petrobras
Ano
2010
Provas
Disciplina
Raciocínio Lógico
Assuntos

Ânderson, Benício, Cecílio, Demétrio e Emídio formam uma fila. Cecílio está na frente de Emídio, que é o 2o da fila. Demétrio não é o último e está atrás de Benício. O último da fila é

Alternativas
Comentários
  • 1º Cecílio

    2º Emídio

    3º  Benicio

    4º Demétrio

    5º Anderson

  • Anderson

  • LETRA A


ID
358456
Banca
FGV
Órgão
CODESP-SP
Ano
2010
Provas
Disciplina
Raciocínio Lógico
Assuntos

Observe a sequência de letras a seguir: “bcadfeghijkolmunpaq...”. Mantida a lei de formação, as duas próximas letras na sequência serão

Alternativas
Comentários
  • 2 consoantes e uma vogal essa é a regra.

    seguindo a sequencia... 

    BC

    DF

    GH

    JK

    LM

    NP, observe que depois do P a vogal A se repete agora tem que ter a sequencia alfabetica com duas consoantes e uma vogal( essa próxima vogal deve ser a letra E.

    ORDEM DAS VOGAIS  ( A,E,I,O,U) JA APARECEU O( A E I O U).... COMEÇA DE NOVO COM A LETRA A A PRÓXIMA DEVE SER A LETRA E)

    Portanto, deve ser LMUNPAQRE, 

    espero ter ajudado! bons estudos! 

  • bcadfeghijkolmunpaq

    Duas consoantes seguidas na ordem + as vogais na ordem

     

    Começou de novo na vogal A, então a próxima continuaria a consoante depois de Q "R" e depois novamente a vogal E


ID
361840
Banca
FUNRIO
Órgão
FURP-SP
Ano
2010
Provas
Disciplina
Raciocínio Lógico
Assuntos

O próximo termo da sequência 0, 3, 8, 15, 24, 35, 48, ... é

Alternativas
Comentários
  • Só no olho daria para resolver! 48+15=63

    de 0 até 3 temos uma diferença de 3 algarismos
    de 3 até 8 temos uma diferença de 5 algarismos
    de 8 até 15 temos uma diferença de 7 algarismos
    de 15 até 24 temos uma diferença de 9 algarismos
    de 24 até 35 temos uma diferença de 11 algarismos
    de 35 até 48 temos uma diferença de 13 algarimos....ENTÃO A PRÓXIMA DIFERENÇA SERÁ DE 15!

    até mais!

    ;)
  • Só  não é correto dizer que a diferença é de "tantos algarismos".
    Pelo menos não no sentido que você quis empregar.

    Números são uma espécie de idéia abstrata, os algarismos são as representações dessas idéias.
  • Obrigado pela retificação amigo! então lá vai....

    Só no olho daria para resolver! 48+15=63

    de 0 até 3 temos uma diferença de 3  números inteiros positivos
    de 3 até 8 temos uma diferença de números inteiros positivos
    de 8 até 15 temos uma diferença de 7 números inteiros positivos
    de 15 até 24 temos uma diferença de 9 números inteiros positivos
    de 24 até 35 temos uma diferença de 11 números inteiros positivos
    de 35 até 48 temos uma diferença de 13 números inteiros positivos....ENTÃO A PRÓXIMA DIFERENÇA SERÁ DE 15 NÚMEROS INTEIROS POSITIVOS!

    até mais!

    ;)


  • OLÁ.

    PRIMEIRAMENTE PODEMOS DESCOBRIR A DIFERENÇA ENTRE OS NÚMEROS:

    3    PARA  8  =5
    8    PARA 15 =7
    15 PARA 24 =9
    24 PARA 35=11
    35 PARA 48=13

    OU SEJA, O PRÓXIMO NÚMERO É A SOMA DO RESULTADO ATUAL, + A DIFERENÇA ENTRE O NÚMERO ANTERIOR E O ATUAL + 2
    LOGO --> (RESUTADO ATUAL)  48 + ( DIFERENÇA: 48-35=) 13 + 2 = 63

  • Eu Hoje= X
    Ela 
    Hoje=X-22

    Eu Daqui a 10 anos = X + 10
    Ela 
     Daqui a 10 anos = X-12

    X+10 = 2X -24   =====>  X = 34
  • Assim foi como fiz: 0,3,8,15,24,35,48...
     
                                    3+2,5+2,7+2,9+2,11+2 logo: 13+2= 15 assim, 48 + 15 = 63
     
     
  • É BEM MAS SIMPLES.Todo número é um quadrado perfeito menos 1

    quadrados:                     1,4,9,16,25,49,64
    o que pede na questão:   0, 3,8,15,24 ,48,63
  • Enxerguei outra coisa...

      0 ,    3 ,    8 ,    15 ,   24 ,  35 ,  48 ,  63 ,  80 ...

    0x2, 1x3, 2x4, 3x5, 4x6, 5x7, 6x8, 7x9, 8x10...
  • Explicação mais simples de Andressa Castro.

  • Com sua resposta ficou bem mais fácil  Diego Lima. Obrigada colega!

  • 0  + 3 =3

         +5= 8

         +7= 15

         +9=24

         +11= 35

         +13= 48

    Podemos ver que é uma progressão aritmética (P.A), que são somados as sequências ímpares... Logo, o próximo número ímpar depois do 13, é o número 15.. Devemos então somar 15+48=63..

    Gab: E


ID
361960
Banca
FUNRIO
Órgão
FURP-SP
Ano
2010
Provas
Disciplina
Raciocínio Lógico
Assuntos

O próximo termo da sequência 0, 4, 18, 48, 100, ... é

Alternativas
Comentários
  • Pensei assim:
    1² x 0, 2² x 1, 3² x 2, 4² x 3, 5² x4 e a sequência seria 6² x 5 = 180
  • 0 , 4 , 18 , 48 , 100 , x

      4  , 14 , 30 , 52,  y

        10 ,16, 22, z

            6   6    6 ....

    z = (22+ 6) = 28

    y = (52 + 28) = 80

    x = ( 100 + 80 ) = 180

  • 2x2x1= 4

    3x3x2=18

    4x4x3=48

    5x5x4=100

    6x6x5=180



ID
370612
Banca
FCC
Órgão
TCE-GO
Ano
2009
Provas
Disciplina
Raciocínio Lógico
Assuntos

Considere que os números que compõem a sequência seguinte obedecem a uma lei de formação.

(120; 120; 113; 113; 105; 105; 96; 96; 86; 86; . . .)

A soma do décimo quarto e décimo quinto termos dessa sequência é um número

Alternativas
Comentários
  • Sequencia é: 120 soma 0 e depois -7, na segunda 113 soma 0 e depois -8, na terceira 105 soma 0 e depois -9 e assim por diante até chegarmos no 14 termo 63 e o decimo quinto 50 somados : 113 IMpar Resposta A


ID
408361
Banca
FCC
Órgão
TJ-SE
Ano
2009
Provas
Disciplina
Raciocínio Lógico
Assuntos

Trocando a ordem das letras OEMTSIO obtém-se um adjetivo que é um sinônimo da palavra OBSTINADO. A letra central desse adjetivo é

Alternativas
Comentários

  • Para resolução da questão deve-se:

    1°)  Procurar os sinônimo da palavra OBSTINADO;


    obstinado
    obs.ti.na.do
    adj (part de obstinar) 1 Que se obstina.
     2 Firme, pertinaz, teimoso. 3 Inflexível. 
    4
     Feito com insistência, com pertinácia.
    fonte:
    http://michaelis.uol.com.br/moderno/portugues/index.php?lingua=portugues-portugues&palavra=obstinado

    2°) Trocando a ordem das letras OEMTSIO é possível formar um dos sinônimos de obstinado: teimoso;

    3°) O comando da questão pede a letra central do adjetivo formado, qual seja: teimoso
    por ter sete letras nesse adjetivo, a letra que fica exatamente no centro é a letra "m". Veja: 
    teimoso
     
    Diante do exposto, a resposta certa é a alternativa (c)
     

     
  • OEMTSIO- teimoso

    c)M
  • anagram- OEMTSIO = 7!

    5040 possibilidades com as letras de OEMTSIO. 1 delas é teimoso.

ID
449821
Banca
FGV
Órgão
MEC
Ano
2009
Provas
Disciplina
Raciocínio Lógico
Assuntos

Uma seqüência numérica (a1, a2, a3, a4,...) é construída de modo que, a partir do 3° termo, cada um dos termos corresponde à média aritmética dos termos anteriores. Sabendo-se que a1 = 2 e que a9 = 10, o valor do 2° termo é:

Alternativas
Comentários
  • Simples;

    a3=(a1+a2)/2

    a4=[(a1+a2)/2 +a1+a2)]/3=(3a1+3a2)/6=(a1+a2)/2

    a5=(a1+a2)/2

    a9=(a1+a2)/2

    10=(2+a2)/2

    20=2+a2

    a2=18



    qualquer dúvida deixem recado


    até mais!

    ;)
  • Só gostaria de complementar o comentário do colega acima com o desenvolvimento de a4.
    primeiro não tinha entendido como o colega acima havia chegado a (a1+a2) + (a1+a2) /3

    depois de ler o enunciado de novo "cada um dos termos corresponde à média aritmética dos termos anteriores"

    então a4 é a soma de a1 com a2 e a3 /3 (média aritmética)

    a3 = (a1 + a2) / 2

    a4 = (a1 + a2 + a3) 
             --------------------
                       3                         -> substitui-se a3 por (a1 + a2) / 2

    a4 =(a1 + a2 + (a1 + a2) / 2 ) 
             ------------------------------------
                              3           

    a4 =(a1 + a2 + (a1 + a2) / 2 ) .  1
                                                     ---
                                                     3                         -> passa o 3 multiplicando-se por 1/3

    a4 =(a1 + a2 + (a1 + a2)  )
             ----------       -----------                                       
                3                6                           -> mmc         

    a4 = (2a1 + 2a2 + (a1 + a2)  )
              --------------------------------                                      
                           6         

    a4 = 3a1 + 3a2    =>          a4 = ( a1 + a2 ) / 2
            ----------------
                    6
            



  • Se for resolver cada um dos termos conforme cita o enunciado da questão, chegaremos a conclusão de que a lei de formação dessa sequência é: 

    an = (a 2 + a1) / 2 
                
    Dessa forma: a9 = (a 2 + a1) / 2
                         10 = (a 2 + 2) / 2
                          a2 = 18

    A questão é simples, mas até chegar a essa conclusão se leva um tempo. 
  • Achei na net bem explicadinho:


    a3 = (a1 + a2)/2

    a4 = (a1 + a2 + a3)/3

    a5 = (a1 + a2 + a3 + a4)/4

    ...

    a9 = (a1 + a2 + ... + a7 + a8)/8 = 10 ---> a1 + a2 + a3 + a4 + a5 + a6 + a7 + a8 = 80

    Vamos fazer o seguinte:

    (a1 + a2) = k

    a3 = (a1 + a2)/2 = k/2

    a4 = (a1 + a2 + a3)/3 = (k + k/2)/3 = k/2

    a5 = (a1 + a2 + a3 + a4)/4 = (k + k/2 + k/2)/4 = k/2

    ...

    Aki a gente percebe que a4 = a5 = a6 = a7 = a8 = a9 = ... = k/2

    Mas a9 = k/2 = 10 ---> k = 20

    a1 + a2 = k

    2 + a2 = 20

    a2 = 18

    Letra A
  • Bons os comentários. Na hora da H, contudo, mais rápido é montar o esqueminha: 2, x, y , _ , _ , _ , _ , _ ,10...jotando x=18  já nos damos conta na hora que a média vai sempre dar 10 para todos, a sequencia seria 2, 18, 10, 10, 10, 10.....
  • Gente, tanta conta, quase pirei e não entendi.

    Eu preferi testar, diante do tamanho das contas e vai que a gente soma algo errado no meio do caminho???1!!!!

    Assim,

    Letra A = { 2, 18, 10 (2+18/2), 10 (2+18+10/3, 10 (2+18+10+ 10/4), 10, 10, 10 ,10 (a9)}, percebe-se que os próximos termos serão 10

    Letra B = { 2, 10, 6, 6, 6, 6, 6, 6, 6(a9)

    Letra C = { 2, 6, 4, 4, 4, 4 , 4, 4,  4(a9)

    Letra D = { 2, 5, 3,5, 3,5, 3,5, 3,5, 3,5, 3,5, 3,5 (a9)

    Letra E = { 2, 3, 2,5, 2,5,2,5,2,5,2,5,2,5,2,5(a9)

    Já que era para fazer tanta conta, testar me parece mais rápido.

    Espero ter ajudado!
  • Concordo com o que o Alexandre disse aí: na hora H ninguém tem tempo pra ficar montando esses esquemas MIRABOLANTES!!!!

    Nessa questão, o ideal mesmo (E MAIS RÁPIDO) é substituir o segundo termo pelas assertivas e montar as sequencias para perceber a lógica:

    Assim, teríamos RAPIDAMENTE:

    quando a2 = 18 (alternativa A) -> 2, 18, 10, 10, 10, 10, 10, 10, 10..... (sempre 10) - gabarito: LETRA A

    quando a2 = 10 (alternativa B) -> 2, 10, 6, 6, 6, 6, 6, 6, 6, 6, (sempre 6)

    Por esse método, conseguiriamos inclusive resolver se fosse pedido o a965483967959687 ...

ID
492517
Banca
CESGRANRIO
Órgão
Petrobras
Ano
2008
Provas
Disciplina
Raciocínio Lógico
Assuntos

Considere a seqüência numérica 1,2,1,2,3,2,1,2,3,4,3,2,1,2,3,4,5,4,3,2,1,2,3,4,5,6,5,4,3,2,1,2, ...

Nessa seqüência, qual a posição ocupada pelo número 50 quando este aparece pela primeira vez?

Alternativas
Comentários
  • -  ache a primeira vez que o numero 3 aparece na sequencia

      multiplique o seu antecedente e o seu consequente e some 1. >>>     2x2 +1= 5º posiçao


    1º-  ache a primeira vez que o numero 4 aparece na sequencia

    2º  multiplique o seu antecedente e o seu consequente e some 1. >>>     3x3 +1= 10º posiçao


    1º-  ache a primeira vez que o numero 5 aparece na sequencia

    2º  multiplique o seu antecedente e o seu consequente e some 1. >>>     4x4 +1= 17º posiçao


      ZZZZZzzzzzz


    3º-  ache a primeira vez que o numero 50 aparece na sequencia

    4º  multiplique o seu antecedente e o seu consequente e some 1. >>>     49x49 +1= 2402º posiçao






  • 1- grupo1

    2 - grupo 2

    12 321 - grupo 3

    234 321 - grupo 4

    234 (54) 321 - grupo 5 - 2 números entre 234 e 321

    234 (56 54) 321 - grupo 6 - 4 números entre 234 e 321

    234 (56 76 54) 321 - grupo 7 - 6 números entre 234 e 321 

    a partir do grupo 3 percebe-se que existe a repetição da sequência de números 321 em todos os seguintes.

    a partir do grupo 4 percebe-se que existe a repetição da sequência de números 234 em todos os seguintes.
    a partir do grupo 5 percebe-se que existe a aparição de sequências de 2 números em ordem múltipla de 2 (54...56,54...56,76,54....)

    Dessa forma, o número 50 irá aparecer pela primeira vez no grupo 50. 

    o grupo 1(1) e grupo 2(2) e parcialmente o grupo 3 (números 1 e 2) representam 4 posições

    A sequência 321 aparece pela primeira vez no grupo 3, logo irá se repetir 48 vezes, então 48 x 3(números) = 144 posições. 

    A sequência 234 aparece pela primeira vez no grupo 4, logo irá se repetir 47 vezes, então 47x3(números) = 141 posições

    Os grupos de 2 números de ordem múltipla de 2 aparecem pela primeira vez no grupo 5, logo serão presentes 46 vezes = 46² = 2116

    4+144+141+2116 = 2401 posições antes do número 50, que ocupará a 2.402ª posição.

  • LETRA C

  • Separei em linhas:

    L1 = 1,2

    L2 = 1,2,3,2

    L3 = 1,2,3,4,3,2

    L4 = 1,2,3,4,5,4,3,2

    ...

    Identifiquei a linha que encontrará o primeiro número 50:

    Se na L2 = número 3; L3 = nº 4; L4 = nº 5..... Para o nº50 a linha será 49.

    A fórmula encontrada é: para saber a soma de elementos da linha, basta multiplicar a linha atual pela linha anterior + a multiplicação da linha atual por 2. Resumo: Nº de elementos da L4 = (3 *4)+(4*2) = 20; L5 = (4*5)+(5*2)=30...

    Porém, o número 50 está no meio da lista, ao chegar nele a lista vai decrescendo, então temos que encontrar o somatório de elementos da linha anterior + 50.

    Logo a L48 = (47*48)+(48*2)= 2.352

    L48 + 50 = 2.352 + 50 = 2.402


ID
512887
Banca
CESGRANRIO
Órgão
Transpetro
Ano
2011
Provas
Disciplina
Raciocínio Lógico
Assuntos

Considere o conjunto de dados a seguir.

60 80 80 85 85 85 85 90 90 90 90 90 100 100 100 100 100 100

O box plot correspondente a esse conjunto de dados é

Alternativas
Comentários
  •  link: http://en.wikipedia.org/wiki/Box_plot
  • Gabarito letra E

    São 18 dados, como ja estao ordenados, facilmente encontramos a mediana, que é 90. Também é facil observar que o 60 é um ponto fora, inferior.
    E não há pontos fora no limite superior. Só com essas observações só ficamos entre as alternativas A, B e E.
    Entao se calcularmos o 3º quartil,verificamos que é 100, não podendo então serem corretas as alternativas A e B.

    Espero ter ajudado!

  • Q1 = 85

    Q3 = 100

    MÁX = 100

    MÍN = 60

    MEDIANA = 90

     

    Montando o boxplot, vemos que o vamor máximo e o Q3 são iguais a 100, logo, gabarito E

  • Sempre se lembrar que um box plot mostra 5 dados: mínimo, quartil inferior, mediana, quartil superior, máximo. Os três quartis dividem a amostra em quatro partes quase iguais.

    60 80 80 85 |Q1| 85 85 85 90 90 |Q2| 90 90 90 100 100 |Q3| 100 100 100 100


    O exercício é resolvido vendo que o máximo (100) coincide com o Q3 (representado pelo número a esquerda, também 100), então o box plot não tem a “perninha” de cima. Além disso, na figura, a mediana, 90, está mais próxima do quartil inferior, 85, que do quartil superior, 100, o que é condizente com os dados. Acho que o ponto embaixo não tem nada a ver e que as alternativas A, C e D são iguais, só muda a escala. Alt E.

     

    ver figura: http://engprodpetrobras.blogspot.com.br/2014/05/probabilidade-e-estatistica.html

    Bons estudos !

     

  • Nossa! Estudei e temos o seguinte:

    60 80 80 85 |Q1| 85 85 85 90 90 |Q2| 90 90 90 100 100 |Q3| 100 100 100 100. 

    Q1 = 85

    Q3 = 100

    AIQ (amplitude interquatílica) = Q3-Q1 = 15

    ÚLTIMO TERMO= 100

    PRIMEIRO TERMO= 60

    MEDIANA OU Q2 = 90

    O Limite superior é o MENOR valor entre:

    *o último termo e o valor de => Q3 + 1,5*(Q3-Q1), ou seja, 100 e 122,5. Logo o limite superior é 100

    O Limite inferior é o MAIOR valor entre:

    *o primeiro termo e o valor de=> Q1 - 1,5*(Q3-Q1), ou seja, 60 e 62,5. Logo o Limite inferior do box plot é 62,5.

    Porém, no box plot, deve-se plotar o menor valor da série (não necessariamente representa o limite inferior), o limite inferior, o 1º quartil (Q1), a mediana (Q2), o 3º quartil (Q3), o limite superior e o maior valor da série (não ncessariamente ser o limite superior).

    Dessa forma, teremos (na sequencia para formar o box plot):

    1º TERMO (pto fora do box plot e abaixo do limite inferior) = 60

    LI = 62,5

    Q1 = 85

    Q2 (MEDIANDA) = 90

    Q3 e limite superior = 100

    SOMENTE A LETRA "E" POSSUI ESSA REPRESENTAÇÃO!


ID
570082
Banca
FCC
Órgão
BACEN
Ano
2006
Provas
Disciplina
Raciocínio Lógico
Assuntos

Na figura abaixo, as letras foram dispostas em forma de um triângulo segundo determinado critério.

             P
          P    Q
         P  R  S
       Q  R  S  T
      Q R —  —  ?
Considerando que as letras K, W e Y não fazem parte do alfabeto oficial, então, de acordo com o critério estabelecido, a letra que deve substituir o ponto de interrogação é

Alternativas
Comentários

  • Resposta letra E. 

    Cada Letra é repetida 3 vezes...

    PPP QQQ RRR, no primeiro __ é S, no segundo _ é T e na ? também é T (terceira repetição)
  • Vejam neste sentido!!!


    http://desmond.imageshack.us/Himg543/scaled.php?server=543&filename=semttulorh.png&res=medium
  • Segue a imagem do Anderson:

    scaled.php.png

  • O site questões de concursos deveria atualizar suas perguntas, pois as letras k,w e y já fazem parte da ortografia oficial.
  • Erbenia,o site QC não pode modificar as questões pq são de concursos!Isto alteraria a finalidade do site!
    Ah!concordo plenamente com o raciocínio do Anderson,pensei assim tbm para resolver a questão!

    E por fim,realmente as letras K,W e Y não são consideradas no alfabeto brasileiro apesar de serem normalmente utilizadas ,principalmente em nomes.
    Ser considerada # ser utilizada
  • Matei a questão assim. Vi que elas seguem a sequencia de três letras

     

                 P                                 P
               P  Q                            P  Q
             P  R  S                        P  R   S
           Q  R  S  T                    Q  R  S  T
          Q R —  —  ?                Q  R  S  T  T


ID
570106
Banca
FCC
Órgão
BACEN
Ano
2006
Provas
Disciplina
Raciocínio Lógico
Assuntos

Na seqüência seguinte o número que aparece entre parênteses é obtido segundo uma lei de formação.

63(21)9; 186(18)31; 85( ? )17

O número que está faltando é

Alternativas
Comentários
  • Gabarito: A

    O número entre parênteses é formado pela divisão do primeiro numeral pelo terceiro, cujo resultado é multiplicado por 3.

    63 (21) 9 = 63/9 = 7 x 3 = 21


    186 (18) 31 = 186/31 = 6 x 3 = 18

    Logo:

    85 (?) 17 = 85/17 = 5 x 3 = 15

    Bons estudos a todos!
  • Também achei o resultado pelo mesmo raciocínio do colega acima.
  • Mas cuidado caro Samuel, não será sempre que esse resultado dar-se-a dessa forma!
  • Existe alguma orientação para questões desse tipo ou será sempre uma questão de advinhar o raciocínio, no caso uma fórmula?

    Respostas por favor!
  • Imaginem fazer isso na hora de uma prova. 
    O cara q elaborou essa questão era pra ter feito a mesma sob muita pressão pra vê se ele iventava essas alucinações. 

    Tava mofado!!
  • É...infelizmente não tem muita fórmula, na verdade quando vc aprende isso os próprios professores dizem: Qualquer coincidência é válida.
    Precisa testar todas as alternativas, esse é o caminho mais fácil .
    Espero ter ajudado!
    :)
  • Letra A

    Utilizei outro raciocínio.

    21 - 3 = 18
    18 - 3 = 15

  • Essas questões são complicadas, né?
    A gente tem que ter uma inspiração divina. Hehe.

    Eu achei uma outra lógica que daria a alternativa C.

    63(
    21)9
    6 + 3 + 9 =
    18

    186(18)31
    1 + 8 + 6 + 3 + 1 =
    19

    85(19)17
    8 + 5 + 1 + 7 =
    21
  • Olá amigos do QC, o número que está entre parênteses é resultado da seguinte operação:
    Divisão do primeiro pelo último número e multiplica por 3.
    63/9 = 7 . 3 = 21
    186/31 = 6 . 3 = 18
    85/17 = 5 . 3 = 15 que é o gabarito.
    Grande abraço.
  • 63/9=7       7X3=21

    186/31= 6  6X3=18

    85/17=5     5X3=15

  • Eu fiz de outro modo  63/9= 7 + 14 = 21 ; 186/31= 6 + 12 =18 ; 85/17= 5 + 10 = 15 ; ou seja na soma vai tintando de 2 em 2.

  • Gente, os numeros entre parenteses tem a diferença entre eles de 3 numeros.

    21 e 18. Logo, o numero faltante é 15!!!!!!!!!!!!!!!!!!! Eita!!

     

    21-3= 18

    18-3= 15

  • Resolvido:

    https://youtu.be/mobkXm4mj3w


ID
608026
Banca
CONSULPLAN
Órgão
Prefeitura de Congonhas - MG
Ano
2010
Provas
Disciplina
Raciocínio Lógico
Assuntos

Na sequência a seguir, quais letras correspondem às lacunas “ - ”, respectivamente:

A L ! B K @ C - # D I $ - H % F

Alternativas
Comentários
  • Questão super fácil! LETRA B

    A L ! B K @ C - # D I $ - H % F    -       sequência de letras da direita para esquerda (pula uma letra e um símbolo e tem-se a próxima letra - J)

    A L ! B K @ C - # D I $ - H % F    -      sequência de letras da esquerda para direita (pula uma letra e um símbolo e tem-se a próxima letra - E)

    Bom estudo para todos!
  • Acrescentando ao comentário acima...
    Fora os símbolos, foram apresentados pares de letras formados por letras de dois grupos:

    O primeiro grupo de letras segue a sequencia alfabética: A B C D E

    O segundo grupo a sequencia alfabética inverifa a partir da letra L: Com a última é a letra H, H I J K L.
  • e pra quem nao reparou a sequencia  "! @ # $ % "     A L ! B K @ C - # D I $ - H % F  é sequência no teclado!
  • A L !   B K @   C - #   D I $    - H %   F  

                                                     E

     

    A L !     B K @  C - #   D I $    E H %  F

                               J

     

    A  B  C  D  E  F  G  H  I  J  K  L  M  N  O  P  Q  R  S  T  U  V  W  X  Z

     

    RESPOSTA 

    b) J, E


ID
609817
Banca
PONTUA
Órgão
TRE-SC
Ano
2011
Provas
Disciplina
Raciocínio Lógico
Assuntos

Observe a sequência numérica abaixo e assinale a alternativa CORRETA, que corresponde ao próximo número da sequência:
77, 49, 36, 18,....

Alternativas
Comentários
  • 77, 49, 36, 18,...

    An = Produto dos algarismos de An-1.

    7*7=49
    4*9=36
    3*6=18
    1*8=8

    Resposta: 8
  • 77,49,36,18, ...

    Observando os números da sequência, concluímos que o números são a multiplicação dos algarismos de seus anteriores, ou seja:

    77, 49 que é 7x7, 36 que é 4x9,18 que é 3x6,...

    Então para o próximo número da sequência, temos:
    1x8=8

    RESPOSTA= 8.

  • Quebrei a minha cabeça e não consegui resolver... só entendi depois que li os comentários... tão simples e eu não descobri sozinha....

  • O pior não é não ter descoberto o porquê da sequência, mas olhar depois aqui e ver o quão besta era a questão...vlw galera...

    Foco sempre.

ID
616678
Banca
FCC
Órgão
MPE-SE
Ano
2010
Provas
Disciplina
Raciocínio Lógico
Assuntos

Considere que os termos da sucessão seguinte são obtidos segundo determinado padrão.

(112, 1 114, 11 118, 1 111 116, 11 111 132, . . . )

A soma dos dígitos que compõem o décimo termo dessa sequência é um número

Alternativas
Comentários
  • resposta 1 111 111 111 512 (10º TERMO) soma total = > 18 = >multiplo de 6 
    GAB C

  • O décimo termo é: 111.111.111.111.024 -> soma total=18 -> múltiplo de 6


    GAB C


ID
622237
Banca
FCC
Órgão
TRT - 2ª REGIÃO (SP)
Ano
2008
Provas
Disciplina
Raciocínio Lógico
Assuntos

Os dois primeiros pares de palavras abaixo foram escritos segundo determinado critério. Esse mesmo critério deve ser usado para descobrir qual a palavra que comporia corretamente o terceiro par.
ESTAGNAR – ANTA
PARAPEITO – TIRA
RENOVADO – ?

Assim sendo, a palavra que deverá substituir o ponto de interrogação é

Alternativas
Comentários
  • ESTAGNAR – ANTA
    PARAPEITO – TIRA
    RENOVADO – ?


    af..o negocio é pensar = maluco.
    ESTAGNAR – ANTA
    AN = estagNAr. antepenultima e penultima letras, ao inverso. NA = AN
    TA = esTAgnar. segunda silaba. 
    penultima + ultima letra(ao inverso) +  segunda silaba = ANTA.
    o msm p as demais.
    PARAPEITO – TIRA
    TI = ante penultima e penultima letras(ao inverso).IT = TI
    RA = segunda silaba
    RENOVADO – ?
    DA = ante penultima e penultima letras(ao inverso). AD = DA
    NO = segunda silaba.
  • LETRA D
    muito easy... PENÚLTIMA + ANTE-PENÚLTIMA + TERCEIRA + QUARTA (LETRAS)
  • ESTAGNA R – ANTA
    PA RA PE IT O – TIRA
    RE NO V AD O – ?

    DANO.

    O "determinado critério" é trocar a ordem da penúltima e antepenúltima letra e concatenar a 2° sílaba.

  • O critério seria pegar a penúltima letra + antepenúltima letra + terceira letra + quarta letra.
  • VSF!
    Que questãozinha.
    Já fiz várias e sou bom em RL, mas essa aí só louco ou drogado pra conseguir resolver kkk.
    Sem querer ofender ningúem viu!
    kkk
  • Agora postando o real comentário.
    Exclui a última letra e a primeira sílaba de cada palavra.
    Pega a segunda sílaba na ordem normal e usa na primeira sílaba da palavra a ser descoberta.
    Para segunda sílaba da palavra descoberta escreva as duas últimas letras ( depois de excluir a última da palavra) na ordem inversa.
    Legal?
    Não achei, mas o examinador deve ter se divertido.
  • ESTAGNAR - ANTA

        34    21

    PARAPEITO - TIRA

        34    21

    RENOVADO - DANO

        34   21

  • GABARITO ITEM D

     

     

    PERCEBA QUE 1º SÍLABA TEMOS QUE INVERTER A ORDEM DAS LETRAS

     

    ESTAGNAR -----> AN   TA 

     

    PARAPEITO ----> TI     RA

     

    RENOVADO -----> DA   NO

  • Esse tipo de questão dada é só para a pessoa não tirar zero, é só prestar atenção que a palavra é composta de 4 letras que ambas tem a leitura invertida de sua referência.


ID
622240
Banca
FCC
Órgão
TRT - 2ª REGIÃO (SP)
Ano
2008
Provas
Disciplina
Raciocínio Lógico
Assuntos

Considere a seguinte seqüência de cálculos:
11² = 121
111² = 12 321
1 111² = 1 234 321
11 111² = 123 454 321
·
·
·
A soma dos algarismos do número que se obtém calculan- do 111 111 111² é

Alternativas
Comentários
  • Fui por exclusão.

    111.111.111² = 12345678987654321

    A soma desses algarismos resulta em 81.

    Oitenta e um não é maior que cem, não é menor que setenta, não é divisível por cinco e não é um número primo.

    Portanto, só poderia ser letra A.
  • 11²  = 121
    111²  = 12 321
    1 111²  = 1 234 321
    11 111²  = 123 454 321

    repare que os numeros dos resultados sao como uma escada..se ha 2 numeros ao quadrado, o resultado tera 2 numeros em ordem crescente e depois vao caindo...
    11^2 = 2 numeros(11) = 12..agora ele cai..1 = 121
    111^2 = 3 n (111) = 123...caindo..21
    1111^2 = 4 n (1111) = 1234..caindo..321
    11111^2 = 5 n (11111) = 12345..caindo..4321
    repare que quando ele chega no auge da escada ele comeca a cair..
    entao se o examinador lhe dá 9 numeros...111 111 111²
    111.111.111^2 = 123456789..chegou no auge e cai..87654321
    a soma de 12345678987654321 é 81
    81 é quadrado perfeito..9^9 = 81
    nozes!
  • nossa esse comentário do colega não é bom não, é excelente...

    parabéns ótimo raciocínio!

  • Veja que todas as somas são quadeado perfeito:
                         11²  = 121 = 1 + 2 + 1 = 4
                      111²  = 12 321 = 9
                   1 111²  = 1 234 321 = 16
                 11 111²  = 123 454 321 = 25
    então: 111 111 111² -> 9²  = 81
  • 11²  = 121- 2 vezes n° 1 ao quadrado: 2 no meio
    111²  = 12321- 3 vezes n° 1 ao quadrado: 3 no meio
    1111²  = 1234321 4 vezes n° 1 ao quadrado: 4 no meio

    111111111^2= 12345678987654321- 9 vezes n° 1 ao quadrado: 9 no meio.

    somando os algarismo de 12345678987654321= 81

    a) um quadrado perfeito.
  • Pensei da seguinte forma:
    111 111 111²
    1+1+1+1+1+1+1+1+1=9 ao quadrado: 81
    Então...por eliminação...letra A
  • pode ser pensado tb assim:

    11² = 12 | 1
    111² = 123 | 21
    1111² = 1234 | 321
    11111² = 12345 | 4321

    Repare que forma-se uma sequencia com a quantidade de 1 que estão elevados ao quadrado e logo em seguinda começa-se uma sequencia decrescente a partir do ultimo numero -1

    Sendo assim o numero

    111.111.111² = 123456789 | 87654321 

    o somatório = 81 sendo um quadrado perfeito

    gabarito letra A
  • Eu somei os números de 1-8, multipliquei por 2 e depois somei 9. Deu 81.
  • é quadrado perfeito a soma da sequencia de 1+1+1+1+1+1+1+1+1=9 / 9*9=81
  • 2 algarismos  =>          1                    2                     1
    3 algarismos  =>          12                 3                     21
    4 algarismos  =>          123               4                    321
    5 algarismos  =>          1234                                4321

    Podemos dividir o número final em 3 partes:

    Na primeira parte ( números em verde), sempre é inserido um número na ordem crescente.
    O número central, em vermelho, corresponde a quantidade de algarismos 1.
    A parte final ( números em laranja), é a ordem inversa dos números da primeira parte.

    111111111  possui 9 algarismos 1, logo, sabendo das informações anteriores, podemos completar a sequência:


    6 algarismos  =>          12345             6                     54321
    7 algarismos  =>          123456           7                     654321
    8 algarismos  =>          1234567         8                     7654321
    9 algarismos  =>          12345678       9                     87654321

    Soma dos algarismos = 12345678       9                     87654321 = 36 + 8 +36 = 81

    81 é quadrado perfeito e não atende às afirmações das demais alternativas.

    Gabarito: Letra A
     
  • Resolvendo de um outro modo...

    112 = 121 (1+2+1 = 4 ) obs.: 4=22
    1112 = 12 321 (1+2+3+2+1 = 9) obs.: 9=32
    11112 = 1 234 321 (1+2+3+4+3+2+1 = 16) obs.: 16=42
    111112 = 123 454 321 (1+2+3+4+5+4+3+2+1 = 25) obs.: 25=52


    Só pelo enunciado já é possível perceber que as somas dos algarismos dos resultados são sempre quadrados perfeitos.
    Ainda é possível observar que a soma dos algarismos do resultado é igual à quantidade de algarismos "1" ao quadrado. 
    Ex.: 1112 -> como são 3 algarismos "1", a soma será igual a 32 = 9 


    Agora resolvendo o que foi pedido por esse método:

    A soma dos algarismos do número que se obtém calculan do 111 111 111² é???

    - Primeiro contamos quantos algarismos "1" temos. São 9
    - Agora é só descobrir quanto é 92 ( já sabemos que é um quadrado perfeito, mas vamos até o fim)
    - 92 = 81. Portanto  1+2+3+4+5+6+7+8+9+8+7+6+5+4+3+2+1= 81
  • se o numero está elevedo ao quadrado então ele vai resultar em um quadrado perfeito...
    estou certo ???

  • dois algarismos numero central vai ate 2 e decresce ate 1

    cinco algarismo numero central vai ate 5 e decresce ate 1

    111.111.111

    9 algarismo numero central vai ate 9

    12345678987654321

    1+2+3+4+5+6+7+8=36 

    36x2=72 + 9 =81


ID
637813
Banca
CONSULPLAN
Órgão
Prefeitura de Mossoró - RN
Ano
2010
Provas
Disciplina
Raciocínio Lógico
Assuntos

Qual das opções completa corretamente a lacuna na sequência NORRAM; _________; EDREV; OLERAMA?

Alternativas
Comentários
  • "MARRON". Parabéns por assassinarem a ortografia, Consulplan.

    Demorei pra resolver só por conta desse erro tosco de vocês.

  • O enunciado e as alternativas estão escritos ao contrário

    NORRAM - MARRON (a banca escreveu errado mesmo, acho q foi para dificultar ou ela não sabia mesmo..)

    EDREV - VERDE

    OLERAMA - AMARELO

    A única alternativa que trata de cor é a d)

    LUZA - AZUL

  • cai na minha prova por favor

  • kkkk só faltou mandarem um marrõ


ID
641299
Banca
FCC
Órgão
TRT - 2ª REGIÃO (SP)
Ano
2008
Provas
Disciplina
Raciocínio Lógico
Assuntos

Considere a sucessão dos números inteiros ímpares e positivos, dispostos da seguinte forma, sem que os algarismos sejam separados:
1 3 5 7 9 1 1 1 3 1 5 1 7 1 9 2 1 2 3 . . .
Nessa seqüência, o algarismo que deverá ocupar a 297a posição é

Alternativas
Comentários
  • Número de algarismos = 297

    Algarismos de UM número: 1,3,5,7,9 = 5 algarismos, Logo, 297 - 5 = 292

    Algarismos de DOIS números: 11 ao 99 = 90 algarismos, Logo, 292 - 90 = 202

    No caso dos números com 3 algarismos, a cada 5 números (101 ao 109; 111 ao 119; 121 ao 129; 131 ao 139; 141 ao 149...) tem-se 15 algarismos. Dessa forma, dividindo o número de algarismos restantes, por 15, saberemos em qual casa de 5 números, o 297º algarismo estará. 

    202/15 = 13, resto = 7

    Dessa forma, iniciando-se a contagem de casas de 5 números do número 101 ao 109, teremos 13 casas de 5 números completos/15 algarismos, nos números 221 ao 229. Observa-se que restam 7 algarismos. Os números seguintes serão: 231, 233, 235; sendo o 7º o algarismo 2.

    Espero que tenha conseguido ser clara. Se houver uma forma mais simples, gostaria de saber! =)



  • d-

    pegadinha: a questao esta 1 3 5 7 9 1 1 1 3 1 5 1 7 1 9 2 1 2 3

    mas na realidade é 1 3 5 7 9 11 13 15 17 19 21 23

    Contagem de digitos.:

    1 - 9  = 5 dig 

    11 - 99 = 90 dig

    101 - 199 = 135 dig

     

    Ate agora foram 135 + 90 + 5 = 230 dig. 297 - 230 = 67 dig faltam

    201 - 209 = 15 dig. 67 - 15 = 52 faltam

    211 - 219 = 15 dig. 52 - 15 = 37 faltam

    221 - 229 = 15 dig. 37 - 15 = 22 faltam

    231 - 239 = 15 dig. 22 - 15 = 7 faltam.

     

    Se restam 7 e o ultimo foi 239, continua-se a partir de 241:

    241 = 3 dig. 7 - 3 = 4 faltam.

    243 = 3 dig. 4 - 3= 1 falta

    O proximo numero da sequencia é 245. Porque 1 digito faltava, basta olhar o 1° digito do numero (2) e ignorar o resto. 

  • não entendi ainda

  • Fui pela ignorância:

    Montei um quadriculado de 30 de lado por 10 de altura (300 casas) e mandei bala :)

    Faltando 3 para completar, parei: deu 2!

    Demorou um pouco, mais acertei.


    Alternativa: D

  • Na hora da prova, primeira dica CALMA ! você vai ter que usar uns 10 minutos aqui não tem jeito.

    1º Passo: identificar a sequência: nesse caso é são todos os números ímpares.

    2 º Passo: diferenciar DIGITO de NUMERO .... ex. 123, 124 são dois números, mas são 6 dígitos ( não queremos o 297º numero, queremos o 297º digito)

    3º Passo: só podemos usar números ímpares na contagem, assim vamos identificar quantos números ímpares existem dentro de uma certa quantidade, segue explicação:

    de 0 a 100 temos 50 números ímpares.

    do nº 01 a 10 = teremos só 5 ímpares: são 1,3,5,7,9; aqui cada numero representa 1 dígito. logo temos 5 dígitos.

    do nº 11 até 100 = temos 90 números sendo a metade ímpar = 45 números, só os ímpares que estão na sequência lógica,

    nossos números estarão entre o 11 e 99 ( atenção, aqui cada numero sempre terá 2 digitos, logo basta fazer 45 números ímpares x 2 dígitos = + 90 dígitos)

    Até aqui você já encontrou 95 dígitos, mas a gente quer o 297º digito, vamos continuar.

    4º Passo: agora qualquer número terá 3 dígitos (ex 100, 530, 788, 966). significa que a cada 1 numero ímpar você vai avançar 3 dígitos.

    do nº 101 até 200 = igual acima explicado temos 45 nº ímpares x 3 DÍGITOS = + 135 dígitos

    quantidade de dígitos até aqui = 5 + 90 + 135 = 230 dígitos. ( faltam só 67 dígitos).

    Se preferir, você tem pode fazer manualmente ... até chegar no 297º

    Nosso número estará entre 200 e 299.

    XXX ,, ele com certeza será 2. ( ele será o numero 243)


ID
641308
Banca
FCC
Órgão
TRT - 2ª REGIÃO (SP)
Ano
2008
Provas
Disciplina
Raciocínio Lógico
Assuntos

A soma de todos os dígitos do décimo termo da seqüência ( 4² , 34² , 334² , 3 334² , ... ) é

Alternativas
Comentários
  • 1o) 4²=16 --> S = 7
    2o) 34²=1156 --> S = 13
    3o) 334²=111556 --> S = 19
    ... .... ... ... ... ... ... ...

     an = a1 + (n-1)*r 
     an=   7+(10-1)*6

    an=7+(9)*6

    an=7+54

    an=61


  • 10 - 1 111111111 555555555 6

  • Resolução:

    https://youtu.be/E42g4NzdT0U


ID
644635
Banca
FCC
Órgão
TJ-PE
Ano
2012
Provas
Disciplina
Raciocínio Lógico
Assuntos

Na sequência 1, 5, 8, 2, 6, 9, 3, 7, 10, 4, ... a lei de formação é uma adição, outra adição, uma subtração e repete a primeira adição, a segunda adição e a subtração, sempre da mesma maneira. Utilize exatamente a mesma lei de formação para criar uma sequência de números naturais a partir do número 7, e outra a partir do número 15. A diferença entre o décimo termo da segunda sequência criada e o décimo termo da primeira sequência criada é

Alternativas
Comentários
  • Essa questão foi muito bem elaborada. Quem prestou atenção ao enunciado e usou um pouco da lógica (a final, é uma questão de Raciocínio Lógico, e nao de matemática básica braçal) conseguiu responder ela em 15 segundos. Mas quem tava com tempo de sobra (acho que poucos que fizeram a prova) criou as duas sequências até o décimo termo.

    Basta analisar que ambas as sequências sofrem SEMPRE A MESMA formação (adições/subtrações). Ora, a diferença entre os termos das sequências será SEMPRE a mesma.

    Diferença entre o primeiro termo da segunda e o primeiro termo da primeira = 8
    Diferença entre o segundo termo da segunda e o segundo termo da primeira = 8
    Diferença entre o terceiro termo da segunda e o terceiro termo da primeira = 8
    .
    .
    .
    Diferença entre o décimo termo da segunda e o décimo termo da primeira= 8

    Logo, Resposta alternativa A
  • http://codabh.org/vou.pdf
  • Sequencia a partir do 7 .... 7 +1= 8...... 8+5=13..... 13-8=5..... 5+2=7......  7+6=13...... 13-9=4...... 4+3=7..... 7+7=14.... 14-10=4..... 4+4=8 este é 10º termo

    Sequencia a partir do 15...15+1=16...16+5=21..... 21-8=13...13+2=15...15+6=21.....21-9=12....12+3=15...15+7=22...22-10=12...12+4=16 este é 10º termo

    A diferença entre o décimo termo da segunda sequência criada e o décimo termo da primeira sequência criada é  16-8 = 8

    ALTERNATIVA  "A"
  • Olá, galera!

    Resolvi da seguinte maneira:

    Utilizando a primeira sequência perceba que a partir do 1 para 5, soma-se
    +4, do 5 para o 3, soma-se + 3 e do 8 para o 2 diminui - 6 e assim sucessivamente.

    É só aplicar
    +4, +3 e - 6. Adição, outra adição e uma subtração conforme o enunciado e, posteriormente, repete-as.

    Um abraço e que Deus nos abençoe
    !
  • Olá pessoal,

    resolvi através da fórmula;


    an = a1 + (n-1)r

     

    an - é o termo que queremos achar - a10  ;n = 10

    a1 - é o 1o. termo ;

    r - é a razão ; que não sabemos.


    Para a 1a. sequencia : 7,11,14,8,18,21...... a1 = 7

    para r = 3, pois é a razão de 11 para 14

    a 10 = 7 + 9 x 3 = 34

    Para a 2a. sequencia : 15,19,22,16,26,29......a1 = 15

    para r = 3 , pois é a razão de 19 para 22

    a10 = 15 + 9 x 3 = 42


    42 - 34 = 8


    Espero ter ajudado.

    Bons estudos !!!


    JMM


     

  • Colegas,

    Método rápido:
    Reparem no seguinte: Todas as sequências são feitas através de 2 adições e 1 subtração, quais sejam: +4 +3 - 6. Isso quer dizer que, de 3 em 3 termos verificaremos o termo inicial acrescido de 1. Este "1" vem da soma +4+3-6=1. 
    Vejam ainda que isso acontecerá no primeiro, no quarto, no sétimo e no décimo termo. Assim teremos
    Sequência do exemplo: 1, 5, 8, 2, 6, 9, 3, 7, 10, 4
    Sequência 1: 7, 11, 14, 8, X, X, 9, X, X, 10
    Reparem que quem entendeu a sistemática na sequência do exemplo não precisou fazer nenhum cálculo, pois sacou que o décimo termo da primeira sequência seria 7+1+1+1= 10 e da segunda 15+1+1+1 = 18.
    Resposta: 18-10=8 
    Gabarito: A
  • Fiz como o Douglas e o Edson:

  • Resolvi da seguinte forma, a seq. "1,5,8,2,6,9,3,7,10,4" nos dá o padrão somatório, de modo que a diferença entre um termo e outro sempre será igual. Logo, a diferença do 1º termo (1) e o 10º termo (4) sempre será igual, ou seja, sempre será +3.
    Partindo de 7:  --> 7+3=10
    Partindo de 15: --> 15+3=18
    Diferença: 18-10=8
    Resposta A
  • Olha só como as questões na FCC se repetem, caiu uma questão muito semelhante na prova do TRF 2ª Região em 2012 e para técnico...
  • GABARITO: A

    A sequência I ficará assim (aplicando sempre +4, +3, -6, e assim sucessivamente):
    7,11,14.8,12,15,9,13,16,10

    A sequência II ficará assim (aplicando sempre +4, +3, -6, e assim sucessivamente):
    15,19,22,16,20,23,17,21,24,18

    Então aplicando a diferença do décimo termo da sequência II para a sequência I: 18-10 = 8


    PULO DO GATO:
    Na verdade não precisava nem mesmo seguir este caminho todo. Voltemos à questão: ela diz o seguinte: a primeira sequência se inicia com o número 7. A segunda sequência se inicia com o número 15. Era só perceber que a diferença entre eles é de 15 - 7 = 8 e que se mantém constante, pois ambos pedem o décimo termo da sequência.
    Tempo de resolução da questão: 15 segundos!


    Bem-aventurados os concurseiros dedicados e disciplinados, porque deles é o Reino da Nomeação!
    Amém!
  • '1', 5, 8, '2', 6, 9, '3', 7, 10, '4',... = 1, 2, 3, ((4)) ← 10º termo está na primeira linha...
    1, '5', 8, 2, '6', 9, 3, '7', 10, 4,... = 5, 6, 7, 8...
    1, 5, '8', 2, 6, '9', 3, 7, '10', 4,... = 8, 9, 10, 11...

    Logo, só me interessa a primeira linha para resolver a questão, pois pede-se o 10º termo!

    Começando pelo 7, temos 7, 8, 9 e (10)
    Começando pelo 15, temos 15, 16, 17 e (18)

    18 - 10 = 8


ID
644809
Banca
FCC
Órgão
TJ-PE
Ano
2012
Provas
Disciplina
Raciocínio Lógico
Assuntos

Duas sequências são construídas conforme descrito abaixo:

Sequência 1: primeiro termo igual a 10 e qualquer outro termo, a partir do segundo, igual ao anterior acrescido de duas unidades.

Sequência 2: primeiro termo igual a 1 e qualquer outro termo, a partir do segundo, igual ao anterior acrescido do número de termos do primeiro até este termo anterior.

Um termo da sequência 1 que é igual a um termo da sequência 2 é

Alternativas
Comentários
  • 10 12 14 16 18 20 22
    1 2 4 7 11 16 22
  • Questão de sequências lógicas, onde devemos criar as duas. Vamos ver: Sequência 1 = 10, 12, 14, 16, 18... A sequência 2 é um pouquinho mais capciosa! Vamos colocar uma figura para facilitar a explicação:



    Notem que o número 16 já faz parte de ambas as sequências, porém não aparece nas alternativas!!!   Resposta: letra C.
  • Sequência/Termo          1        2         3         4          5          6          7       8       9
    [1]      ( 10 )  10 + 0 = 10  10 + 2= 12  12 + 2= 14   14 + 2= 16    16 + 2= 18    18 + 2= 20    20 + 2= 22    22 + 2= 24 24 ......
    SOMA [1]       10      12       14       16        18        20       [ 22 ]        24 .....
                       
    [2]       ( 1 )   1 + 0 = 1   1+ 1= 2   2+ 2= 4   4+ 3= 7   7+ 4= 11     11+ 5= 16     16+ 6= 22     22+ 7= 29 29  .....
                       
    SOMA [2]         1        2         4         7        11       16       [ 22 ]        29 ....
     
  • Colegas, a questão é simples feita manualmente. Tentei racionalizar demais aqui.. terminei por me confudir.
    Achei uma equação p/ a primeira sequência: f(x)= 10 + 2 (x - 1)
    Já para a segunda, não achei consegui enxergar nenhuma forma... será que tem como? hehe
    Se for possível, é só igualarmos f(x) com a equação da segunda sequência e encontraremos o termo que é comum a ambas as sequeências. abs!
     

  • A explicação do colega Djanilson Lopes foi categórica, não tem como não entender.!!!!
    Nota 10!!!
  • Pois é, frio na barriga na hora da prova. Pois o 16 também atende ao comando. Fiquei acreditando que não estava entendendo a questão.
  • Olá, não consigo visualizar a figura da resposta de Djanilson Lopes, alguém poderia explicar a formação lógica da sequência 2? Estou acompanhando a questão! Grato!

  • Sequência 1: primeiro termo igual a 10 e qualquer outro termo, a partir do segundo, igual ao anterior acrescido de duas unidades. 

    LOGO: PRIMEIRO TERMO 10; SEGUNDO TERMO 12; TERCEIRO TERMO 14; QUARTO TERMO 16; E ASSIM POR DIANTE.

    Sequência 2: primeiro termo igual a 1 e qualquer outro termo, a partir do segundo, igual ao anterior acrescido do número de termos do primeiro até este termo anterior. 

    LOGO A SEUNCIA 2 FICARÁ ASSIM:

    PRIMEIRO TERMO 1 (A QUESTÃO DEU),

    SEGUNDO TERMO (  igual ao anterior (1)  número de termos do primeiro até este termo anterior.) --> OBS: ELE NÃO PEDE A SOMA!!!,                                              |                                                                |                                               PEDE A QUANTIDADE DE TERMOS                              \ /                                                             \  /

    SEGUNDO TERMO                 1                             +                                  1                    =     2                    

    TERCEIRO TERMO    QUAL O ANTERIOR?    2      +    QUANTOS TERMOS ANTES DO TERCEIRO TERMO ?     2      =    terceiro termo 4

    QUARTO TERMO       QUAL O ANTERIOR?    4      +      QUANTOS TERMOS ANTES DO QUARTO TERMO ?       3     =    quarto termo   7

    QUINTO TERMO         QUAL O ANTERIOR?    7    +      QUANTOS TERMOS ANTES DO QUARTO TERMO ?          =    quarto termo   11

    SEXTO TERMO          QUAL O ANTERIOR?    11      +      QUANTOS TERMOS ANTES DO QUARTO TERMO ?        =    quarto termo   16

    SETIMO TERMO          QUAL O ANTERIOR?    16    +      QUANTOS TERMOS ANTES DO QUARTO TERMO ?       6     =  setimo termo  22

     

    RESUMINDO SEQUENCIA 1 :  10, 12 ,14, 16, 18, 20, 22    

                         SEQUENCIA 2 : 1 , 2, 4, 7,  11,  16,  22

     

    O valor 16 é igual , mas a questão não colocou como alternativa, logo 22 tem como alternativa. LOGO GABARITO :   C

     

     

  • Sequência 1, não há dúvidas, né?! 10 ; 12; 14 ; 16 ; 18 ; 20 ; 22 ; 24...
    Sequência 2 fica: 1 ; 2 (termo anterior:1 + posição do número anterior:1) ; 4 (termo anterior:2 + posição do número anterior:2) ; 7 (termo anterior:4 + posição do número anterior:3) ; 11 ; 16 ; 22 ; 29 ; 37...

     

    espero que tenha ficado claro!


ID
644977
Banca
FCC
Órgão
TJ-PE
Ano
2012
Provas
Disciplina
Raciocínio Lógico
Assuntos

As sequências de números naturais: 3, 7, 6, 10, 9, 13, 12, 16, 15, . . ., e 4, 8, 7, 11, 10, 14, 13, 17, 16, . . . foram criadas com uma regra que alterna uma mesma adição e uma mesma subtração ilimitadamente. São diferentes porque começaram com números diferentes. A soma entre o 12o termo de uma sequência, criada com essa mesma regra e cujo número inicial é 7, e o 13o termo de uma outra sequência, criada com essa mesma regra e cujo número inicial é 8, é

Alternativas
Comentários
  • (det 12o termo)   7, [+4], 11, [-1], 10, [+4], 14, [-1], 13, [+4], 17, [-1], 16, [+4], 20,

    (det 13o termo)   8, [+4], 12, [-1], 11, [+4], 15, [-1], 14, [+4], 18, [-1], 17, [+4], 21,
     
    19, 23, 22, 26,...
    20, 24, 23, 27, 26,...            26+26 = 52. Letra (D)
  • A regra da sequência é:
    1º termo + 4 = 
    2º termo - 1 =  
    3º termo + 4 = 
    4º termo - 1 = 
    e assim se repete...

    7, 11, 10, 14, 13, 17, 16, 20, 19, 23, 27, 26 (12º).
    8, 7, 11, 10, 14, 13, 17, 16, 20, 19, 23, 27, 26 (13º).

    Nessa regra o 12º termo de uma sequência é 26 e o 13º da outra também é 26. 26 + 26 = 52. Alternativa "D".
  • comecei a 1ª sequencia assim:
    1)   7+4
    2)   11-1
    3)   10+4
    .
    .
    .
    11)  22+4
    12)  26
     e depois fiz a 2ª sequencia da msm forma:
    1)  8+4
    2) 12-1
    3) 11+4
    .
    .
    .
    12) 27-1
    13) 26

    agora é somar os termos: 12º + 13º= 26+26=52

    facin, facin...
  • Mais fácil: não precisa somar e subtrair as duas sequências!!!
    1) Monte a primeira sequência até o 13º termo: 7-11-10-14-13-17-16-20-19-23-22-26-25
    2) Identifique o 12º dessa sequência (que é pedido pelo enunciado): 26
    3) A segunda sequência segue a mesma regra de soma e subtração da primeira, portanto cada termo se diferenciará apenas pela diferença do 1º termo, que é +1 (8-7=1)... Se o 13º termo da primeira sequência é 25, o 13º termo da segunda sequência será 26 (1 a mais). É uma questão de raciocínio lógico.
    4) Somando, 26+26=52
  • Essa questão era para ser anulada porque há dois tipos de respostas!

    veja bem!

     7, 6, 10, 9, 13, 12, 16, 15

    7+3=10

    10+3=13

    13+3=16

    16+3=19............e assim vai até chegar à 12° posição

    Seguindo essa logica a resposta erá outra!

    Portando, o gabarito deve ser anulado!

    Questão mal elaborada!


ID
646510
Banca
FCC
Órgão
TJ-PE
Ano
2012
Provas
Disciplina
Raciocínio Lógico
Assuntos

Em uma enquete dez pessoas apreciam simultaneamente as praias J, M e N. Doze outras pessoas apreciam apenas a praia N. O número de pessoas que apreciam apenas a praia M é 4 unidades a mais que as pessoas que apreciam apenas e simultaneamente as praias J e N. E uma pessoa a mais que o dobro daquelas que apreciam apenas a praia M são as que apreciam apenas e simultaneamente as praias J e M. Nenhuma outra prefe- rência foi manifestada nessa enquete realizada com 51 pessoas. A sequência de praias em ordem decrescente de votação nessa enquete é

Alternativas
Comentários
  • O gabarito está errado. O certo seria a letra c.
    Fica mais fácil resolver se montarmos um Diagrama de Venn-Euler.

  • Concordo com o gabarito letra C, incluse entrei com recurso na Fcc para essa questão
  • Também utilizei o diagrama e achei letra C.

    M:35
    J:31
    N:26


    Bons Estudos!
  • Alguém pode explicar como montar o diagrama de Venn-Euler para essa questão?
  • Unanimidade, gabarito letra C. A FCC se equivocou.

    n(N) = 10 + 12 + x

    n(M) = 10 + (x+4) + y

    n(J) = 10 + x + y

    n(M) + n(N) + n(J) = 51


    Tendo 2(x + 4) + 1 = y
    , resolvemos e encontramos x = 4

    Substituindo, temos n(N) = 26, n(M) = 35 e n(J) = 31

    Logo, gabarito é a letra C >> M; J; N
  • Resolvendo a questão:

    Montando o diagrama, são três círculos com intersecçao entre J e M; J e N; M e N; e entre os três simultaneamente. Comecemos por este, que fica no centro do diagrama, e corresponde às 10 pessoas que simultaneamente apreciam as três praias. 

    O segundo dado fornecido, isto é, as 12 pessoas que apreciam apenas a praia N, ficará no espaço exclusivo desta praia. 

    Supondo que X é o número de pessoas que apreciam simultaneamente as praias J e N, então:

    X + 4   será o número de pessoas que apreciam exclusivamente a praia M;

    2 (X + 4) + 1   será o número de pessoas que apreciam simultaneamente as praias J e M. 

    Pronto, como a questão diz que mais nenhuma preferencia foi manifestada na enquete, é só pegar tudo o que foi inserido no diagrama, e igualar a 51, que é o numero total de entrevistados. Assim:


    10 + 12 + X + X + 4 + 2 (X + 4) + 1 = 51

    26 + 2X + 2X + 8 + 1 = 51

    4X = 51 - 35

    4X = 16

    X = 4

    Agora é só substituir nas expressões que utilizamos:  

    Só praia M:   X + 4 = 8

    Praias J e N: o próprio X = 4

    Praias J e M:  2 (X + 4) + 1 = 17


    Somando o que corresponde a cada praia, fica:

    J = 10 + 4 + 17 = 31

    M = 10 + 8 + 17 = 35

    N = 10 + 4 + 12 = 26


    Portanto, em ordem decrescente:   M > J > N  


    Como foi dito, LETRA C. 

  • O Gabarito definitivo sairá amanhã dia 29/02/12. Com certeza letra C.
  • A FCC faz tanta volta em uma questão que ela mesma se confunde, Temos tanto trabalho para elaborar uma sequencia de raciocínio para depois vir essa banca chinfrim e fazer uma m.... desta.
       
         é óbvio que a alternativa certa é a "C"
  • Essa questão teve o gabarito alterado para letra C, está no DJE que foi publicado ontem.

    TRIBUNAL DE JUSTIÇA DO ESTADO DE PERNAMBUCO
    CONCURSO PÚBLICO
    EDITAL Nº 05/2012
    EDITAL DE DIVULGAÇÃO DOS RESULTADOS DAS PROVAS OBJETIVAS
    APÓS ANÁLISE DE RECURSOS
    O DESEMBARGADOR JOVALDO NUNES GOMES, PRESIDENTE DO TRIBUNAL DE JUSTIÇA DO ESTADO DE PERNAMBUCO, no uso de suas atribuições legais e tendo em vista o Concurso Público destinado ao provimento de cargos vagos e à formação de cadastro reserva para cargos efetivos do Quadro de Pessoal do Tribunal, regido pelo Edital de Abertura de Inscrições publicado no Diário da Justiça Eletrônico do Estado de Pernambuco, Edição nº 184/2011, de 04/10/2011, RESOLVE:

    II - INFORMAR a alteração de gabarito e reprocessamento do resultado, para o cargo de Analista Judiciário - Todas as Especialidades, em virtude de ter-se identificado que eram procedentes os recursos da questão de Raciocínio Lógico da Prova de Conhecimentos Gerais , quando haviam sido considerados como improcedentes.
    ALTERAÇÃO DE GABARITO CÓDIGO DE OPÇÃO: AA - Analista Judiciário - APJ - Judiciária e Administrativa
    Questão 19 tipo 1 C
    Questão 19 tipo 2 D
    Questão 18 tipo 3 D
    Questão 18 tipo 4 E
    Questão 19 tipo 5 E


    Recife/PE, 08 de março de 2012
    Desembargador Jovaldo Nunes Gomes
    Presidente

  • Olá, pessoal!
    O gabarito foi atualizado para "C", conforme edital publicado pela banca e postado no site.
    Justificativa da banca:  ALTERAÇÃO DE GABARITO CÓDIGO DE OPÇÃO: AB - Analista Judiciário - APJ -Educador Físico, AC - Analista Judiciário -
    APJ - Assistente Social, AD - Analista Judiciário - APJ – Psicólogo, AF -Analista Judiciário - APJ – Pedagogo, AG- Analista
    Judiciário - APJ – Bibliotecário, AH - Analista Judiciário - APJ – Fisioterapeuta, AI -Analista Judiciário - APJ – Nutricionista, AM
    - Analista Judiciário - APJ – Odontólogo, NA -Analista Judiciário - APJ - Médico Cardiologista, AO - Analista Judiciário - APJ -
    Médico Clínico Geral, AP -Analista Judiciário - APJ - Médico Ginecologista, AQ -Analista Judiciário - APJ - Médico Neurologista,
    AR -Analista Judiciário - APJ - Médico Psiquiatra, AS - Analista Judiciário - APJ - Médico Reumatologista, AT - Analista
    Judiciário - APJ - Médico Traumatologista, AU -Analista Judiciário - APJ - Médico Oftalmologista e AX - Analista Judiciário -
    APJ - Contador
    Questão 16 tipo 1 C
    Questão 15 tipo 2 E
    Questão 16 tipo 3 A
    Questão 16 tipo 4 E
    Questão 15 tipo 5 A
    Bons estudos!


  • Questão resolvida:

    http://www.espacojuridico.com/blog/possivel-recurso-de-logica/

  • Para essa questão vamos usa o diagrama de venn, que após análise ficou assim:

     

    Infelizmente na formatação do site qconcursos não aparece o diagrama.Você pode conferir a resposta na íntegra através do link:

    http://rlmparaconcursos.blogspot.com.br/2017/05/fcc-2012-tjpe.html

     

    No caso dez pessoas apreciam as três praias, por isso colocamos o valor “10” no conjunto que abrange as três praias, meio do diagrama;

    Doze pessoas apreciam somente a praia N, valor inserido na parte referente somente ao conjunto N;

    “O número de pessoas que apreciam apenas a praia M é 4 unidades a mais que as pessoas que apreciam apenas e simultaneamente as praias J e N.” Então no conjunto JN, podemos considerar como “x” e no conjunto M “x + 4”, pois não nos foi dado quantidade exata.

    “E uma pessoa a mais que o dobro daquelas que apreciam apenas a praia M são as que apreciam apenas e simultaneamente as praias J e M.” Como M é “x + 4”. O dobro será 2(x + 4); e uma pessoa a mais seria 2(x + 4) + 1, valor inserido no conjunto J M.

    Como o total de pessoas a realizarem a enquete foi de 51, a soma de todos esses valores deverá ser 51. Partindo daí podemos encontrar o valor de “x”.

    2(x+4)+1+10+x+12+x+4=51

    2x+8+1+10+x+12+x+4=51

    4x+35=51

    4x=51-35

    4x=16

    X=16/4

    X=4

    Como encontramos o valor de “x”, podemos substitui-lo no diagrama para saber o valor numérico de cada conjunto.

     

    Infelizmente na formatação do site qconcursos não aparece o diagrama.Você pode conferir a resposta na íntegra através do link:

    http://rlmparaconcursos.blogspot.com.br/2017/05/fcc-2012-tjpe.html

     

    Agora podemos somar os votos recebidos por cada praia:

    J: 17 + 10 + 04 = 31

    M: 08 + 17 + 10 = 35

    N: 12 + 04 + 10 = 26

    Como o exercício pede em ordem decrescente (do maior para o menor) temos a sequencia: M; J; N.

    Gabarito: Letra C

    Informamos que a questão foi objeto de recurso no concurso em questão, que ao final da análise da banca alterou a resposta para letra C.

    Fonte: http://www.concursosfcc.com.br/concursos/tjupe111/edital_resultado_apos_recurso_analista.pdf

    http://rlmparaconcursos.blogspot.com.br/


ID
655114
Banca
VUNESP
Órgão
UNIFESP
Ano
2007
Provas
Disciplina
Raciocínio Lógico
Assuntos

“Números triangulares” são números que podem ser representados por pontos arranjados na forma de triângulos eqüiláteros. É conveniente definir 1 como o primeiro número triangular.

Apresentamos a seguir os primeiros números triangulares.

                                                                            •           
                                                                      •     • 
              •                       •                          •    •   •       
          •  •                   •   •   •                   •   •   •   •
1            3                           6                              10

Se Tn representa o n-ésimo número triangular, então T1 = l, T2 = 3, T3 = 6, T4 = 10, e assim por diante. Dado que Tn satisfaz a relação Tn = Tn–1 + n, para n = 2,3,4,..., pode-se deduzir que T100 é igual a

Alternativas
Comentários
  • Técnica TWI:
    a1=1
    a2=1+2
    a3=1+2+3
    a4=1+2+3+4
    a100=1+2+3+4+5+....+100
    representa uma PA de razão 1:
    soma PA=(1+100)/2  x100 = 101x50 = 50,5x100 = 5050
  • Amigo, o que significa esse "TWI"  ?   Muito obrigado.
  • PA de segunda ordem (uma sequência de números em que as diferenças entre os termos consecutivos segue uma progressão aritmética).

    Achei um vídeo (curtinho, 4 min)  no youtube que explica como resolver:
    http://www.youtube.com/watch?v=_Ys8DsbwK6k
  • Trata-se de uma PA de segunda ordem, na qual a taxa r tambem obedece uma sequência, que vou chamar de R.
    R = 2, 3, 4..., sendo R(1) = 2;  R(2)=3;     R(3) = 4...; e a razão = 1.


    Fórmula da PA de segunda ordem:  (substituí An por Tn e A1 por T1, só para ficar nas mesmas variáveis que o exercício)

    T(n)  =   T(1)   +   [ R(1) +  R(n-1) ]   .   (n - 1)
                               -----------------
                                         2

    No caso, queremos encontrar T(100)
    Temos:  n = 100, portanto n-1=99
                    T(1) = 1
                 e R(1) = 2
    Devemos encontrar R(99)
    Como R também obedece uma progressão aritmética, podemos encontrar seu 99º termo pela fórmula:
    R(99) = R(1) + (n-1). r
    R(99) = 2    + (99 - 1).1
    R(99) = 2 + 98
    R(99) = 100

    Substituindo todos os termos na fórmula da PA de segunda ordem:

    T(n) = T(1)   +    [R(1) + R(n-1)]  *  (n-1)  =                        T(100) = 1 + ( 2 + 100) * 99
                                 ------------------                                                                   ------------
                                          2                                                                                      2

    T(100) = 1 + 102 (99)/2
    T(100) = 1 + 5049
    T(100) = 5050
  • Essa é fácil... basta pegarmos o a colocação do N-ésimo termo e somarmos com o primeiro... Assim: A100 vai ser o A99 + 100, então A100= 101 (a1+100 termos) * 50 (pois A1+A100 = A2+A99 = A49 + A51)...

    101*50 = 5050.


ID
655654
Banca
FCC
Órgão
TRF - 3ª REGIÃO
Ano
2007
Provas
Disciplina
Raciocínio Lógico
Assuntos

Em relação à disposição numérica seguinte, assinale a alternativa que preenche a vaga assinalada pela interrogação:

2 8 5 6 8 ? 11

Alternativas
Comentários
  • 2+8=10
    5+6=11
    8+x=12
    x=4
    11 + 2=13
  • 2 8 5 6 8 ? 11
    a relação do 2 com o 5 e do 5 com 8 e do 8 com 11 é de soma :
      logo: razão=3 .:
    2+ 3 = 5
    5 +3 = 8
    8 +3 = 11

    quanto a relação do 8 com o 6 e do 6 com ? é de subtração :
    logo: razão=
    2.:
    8-
    2= 6
    6-
    2= 4 => ( ?)
  • Não seria 14?

    2 8 5 6 8 ? 11

    Dividindo a sequência em duas partes "iguais" e excluindo o algarismo 6, temos:
    I) 2 8 5 (a1, a2 e a3, respectivamente)
    II) 8 ? 11 (b1, b2 e b3, respectivamente)

    Se correspondermos a1 com b1, a2 com b2 e a3 com b3, temos uma sequência lógica:
    2 >> 8 (de 2 para 8 foram acrescidas 6 unidades)
    8 >> ?
    5 >> 11 (de 5 para 11 também foram acrescidas 6 unidades)
    Pela lógica, 8 + 6 = 14
    Ou seja, somando-se 6 unidades aos números da primeira sequência, obtemos os valores correspondentes na segunda sequência.
    Na minha opinião, a resposta é 14.
  • 28+28=56
    56+28=84
    logo o nº que falta é 4     ----->   2 8 5 6 8__11

  • Mesmo com as explicações ainda estou "viajando" o.O
  • Devemos considerar que as seguintes dezenas, multiplas de 28, estão escritas, mesmo que de forma incompleta:

    28, 56, 84, 112

    Portanto: 2 8 5 6 8 4 11

    A resposta é 4
  • 2  8  5  6  8  __  11

    Se considerarmos os termos de dois em dois temos:

    28  56  8__  11

    onde: 28 = 7x4

             56 = 7x8

             84 = 7x12

             112 = 7x16 (2 deve ser o proximo termo no fim da sequencia apresentada)

    logo o numero que procuramos só pode ser 4.

  • Eu separei em duas sequências:

    Uma das posições impares (I) e outra das posições pares (II) e reparei:

    I - 2, 5, 8, 11 (soma 3)

    II- 8, 6, ? (subtrai 2)
     

    Espero ter ajudado. Bons estudos!

  • 2 8 5 6 8 ? 11

    28 + 28 = 56

    56 + 28 = 84

    84 + 28 = 112

    2 8 5 6 8 4 1 1 2

    Resposta: Letra (B) 4

  • Interessante como, apesar das lógicas apresentadas aqui serem diferentes, o resultado bate.

    Considerei a mesma coisa que o colega Helton, até pq os algarismos estão separados e não existe comando para assumí-los dois a dois, ou seja, para mim cada algarismo é um objeto diferente na sequência.


ID
669226
Banca
FCC
Órgão
TCE-SP
Ano
2012
Provas
Disciplina
Raciocínio Lógico
Assuntos

Observe as sequências de letras obtidas com uma mesma ideia.

I. A; B; D; G; K; P.

II. B; C; E; H; L; Q.

III. C; D; F; I ; M; R.

IV. D; E; ___; J; ___; S.

Utilizando a mesma ideia, a sequência IV. deverá ser completada, respectivamente, com as letras

Alternativas
Comentários
  • GABARITO LETRA C

    Resolvi da seguinte maneira.
    Li a sequencia na vertical, ou seja:

    (A,B,C,D)
     (B,C,D,E)

     (D,E,F,G)
     (G,H,I,J)
     (K,L,M,N)


     I.   A;          B;        D;      G;    K;        P.

    II.  B;          C;        E;       H;    L;       Q.

    III. C;          D;        F;        I ;    M;      R.

    IV. D;          E;       ___;    J;    ___;     S.


  • I   A B _ D _ _ G _ _ _ K _ _ _ _ P
    II  B C _ E _ _ H _ _ _ L _ _ _ _ Q
    III C D _ F _ _ I   _ _ _ M_ _ _ _R
    IV D E _ G _ _ J _ _ _ N _ _ _ _ S
  • tirando a primeira sequencia que nao segue um padrão 1,2,2,5,5 as outras segue o mesmo padrão de diferença de letra para letra (1,2,3,4,5) daí só aplicar na ultima sequencia a diferença de uma, duas, tres, quatro e cinco letras a amais da anterior!
    bons estudos!
  • Não sei se é mais fácil dessa forma, mas eu atribuí às letras a numeração correspondente ao alfabeto oficial.

    A = 1
    B = 2
    C = 3 
    D = 4
    E = 5
    F = 6
    G = 7
    H = 8
    I = 9
    J = 10
    K =11
    L = 12
    M = 13
    N = 14
    O = 15
    P = 16
    Q = 17
    R = 18
    S = 19

    Percebam que a sequencia é sempre o número da letra anterior + 1, + 2, + 3, + 4, + 5

    D > 4
    E > 5    ( 4 + 1)
    G > 7     (5 + 2)
    J > 10    (7 + 3) 
    N > 14  (10 + 4)
    S > 19  (14 + 5)


  • A B C D E F G H I J K L M N O P
    B C
    D E F C H I J K L M N O P Q
    C D
    E F G H I J K L M N O P Q R
    D E
    F G H I J K L M N O P Q R S
  • Bem, eu notei que a letra que some é de uma contagem crescente, vejamos:

    a 1º que some começa a contar como uma.
    depois a que some é duas
    depois somem 3 letras seguintes e assim sucessivamente....

    Vejam como fica....

    I. A; B; D; G; K; P.  
    letra c some (conta como uma), depois vem o D e some duas letras (E,F), coloca o G e some agora 3 letras (H,I,J), coloca o k depois some 4 e assim continua...
    II. B; C; E; H; L; Q. 
    III. C; D; F; I ; M; R. 
    IV. D; E; ___; J; ___; S. 
    Fiz a mesma coisa para o item IV:
    D;E (some uma letra "F") e coloca o G; (Agora somem 2 letras H,I); e depois do J somem 3 letras (K,L,M); Coloca o N (somem 4 letras) e chega ao S.

    1 letra desaparece, depois duas, depois três e depois 4.
  • De acordo com o enunciado e usando as posições das letras do alfabeto, tem-se:

    A B D G K P   1 , 2 , 4 , 7 , 11 , 16

    B C E H L Q  2 , 3 , 5 , 8 , 12 , 17

    C D F I M R  3 , 4 , 6 , 9 , 13 , 18

    D E ? J ? S  4 , 5 , ? , 10 , ? , 19

    Verifica-se que a terceira posição é a soma da segunda mais duas unidades e a quinta posição é a soma da quarta mais quatro unidades.

    Assim, a sequência se completa com os números:

    5 + 2 = 7  letra G do alfabeto

    10 + 4 = 14  letra N do alfabeto

    Resposta C



ID
671953
Banca
CESPE / CEBRASPE
Órgão
PC-ES
Ano
2009
Provas
Disciplina
Raciocínio Lógico
Assuntos

Julgue o  item a seguir, acerca de raciocínio lógico. 

Na sequência numérica 23, 32, 27, 36, 31, 40, 35, 44, X, Y, Z, ..., o valor de Z é igual a 43.

Alternativas
Comentários
  • CERTO.
    na sequência:
    23+9=32.
    32-5=27.

    27+9=36.
    36-5=31.
    31+9=40.
    e assim por diante. sempre soma 9 e diminui 5.
    Então Z terá valor 43.
  • Separe as posições ímpares das pares e some 4:

    Todos: 23, 32, 27, 36, 31, 40, 35, 44, X, Y, Z
    Posição ímpar: 23, 27, 31, 35, 39(X), 43(Z)
    Posição par: 32, 36, 40, 44, 48(Y).
  • 23+4=27

    32+4=36

    27+4=31

    36+4=40

    31+4=35

    40+4=44

    35+4=39 corresponde ao valor de X

    44+4=48 corresponde ao valor de y

    39+4=43 corresponde ao valor de z.

     

     

     

     

     

  • Temos dua sequencias de números IMPARES e PARES mesclados em uma sequencia:

    23, 32, 27, 36, 31, 40, 35, 44, X, Y, Z.

    Para facilitar retiramos da sequencia os impares: 23, 27, 31, 35, X, Z.

    Agora fica mais fácil perceber a sequencia logica. Observer que os números CRESCEM de 4 em 4.

    27 - 23 = 4

    31 - 27 = 4

    35 -31 = 4

    X-35 = 4

    Z- X = 4

    Então, 35 + 4 = X.   X é igual a 39.

    39 + 4 = Z.  então Z é igual a 43. RESPOSTA DA QUESTÃO ( CERTO)

     

  • Gabarito: CERTO.

     

    A sequência é +9, -5, +9, -5...

  • SOMA POR 9 e SUBTRAI POR 5, vá assim até chegar na letra Z, a qual será 43...

    Rumo à PCDF...

  • Vamos analisar as posições ímpares e as posições pares separadamente:

    23, 32, 27, 36, 31, 40, 35, 44, X, Y, Z

    Veja que, em ambas, do termo anterior para o termo seguinte é somado 4. Logo:

    35 + 4 = X à X = 39

    39 + 4 = Z à Z = 43

    Item CORRETO.

    Resposta: C

  • Vai achando que vai cair assim na PC-DF kkkk

    Pode ir resolvendo questões do BNB 2018.

  • GAb C

     23, 32, 27, 36, 31, 40, 35, 44, X(39), Y, Z (43).

    Tá somando de 4.

  • Certo

    A sequencia é d e 4 em 4 números

    40,35,44,x,y,z

    40,35,44,39,48,43

  • soma 9, subtrai 5 , vai embora até chegar em Z

  • Questão dada...

    +9 e-5

    X= 39

    Y= 48

    Z= 43

  • GAB C

    SOMA 9

    DIMINUI 5

  • Lógica da Questão:

    Número Par = -5

    Número Ímpar = +9

    23, 32, 27, 36, 31, 40, ...

  • 23,32,27,36,31,40,35,44,x,y,z

    Soma-se de 4 em 4 os números em vermelho.

    x=39

    y=48

    z=43

  • Olá pessoal,

     

    Vejam o vídeo com a resolução dessa questão no link abaixo

    https://youtu.be/yVC1ioG-CVA

     

    Professor Ivan Chagas

    www.youtube.com/professorivanchagas

  • (C)

    Questão igual com gabarito errado que caiu na PRF 2019

    (PRF-2019)Uma unidade da PRF interceptou, durante vários meses, lotes de mercadorias vendidas por uma empresa com a emissão de notas fiscais falsas. A sequência dos números das notas fiscais apreendidas, ordenados pela data de interceptação, é a seguinte: 25, 75, 50, 150, 100, 300, 200, 600, 400, 1.200, 800, ....

    Tendo como referência essa situação hipotética, julgue o item seguinte, considerando que a sequência dos números das notas fiscais apreendidas segue o padrão apresentado.

    A partir do padrão da sequência, infere-se que o 12.º termo é o número 1.600.(E)

    O Correto seria o numeral 2,400

  • GAB C

    X= 39

    Y= 48

    Z=43

    SEQUENCIA ( +9 ,-5 )

  • Certo!

    23,32,27,36,31,40,35,44,x,y,z

    ---------+4------+4-------+4------+4---+4

    Ou seja, X= 39 e Z= 43

  • É uma sequência intercalada na outra.

    Começando com 23 será 23 pula uma e 23 + 4 e segue nessa ordem.

    Começando com 32 será 32 pula uma e 32 + 4 e segue nessa ordem.

    23,32,27,36,31,40,35,44,39,48,43.

  • An = An-2 + 4

    A11 = A9 + 4

    A11 = X + 4

    O X, como diz na questão, é o A9, então tem que encontrar ele

    A9 = A7 + 4

    A9 = 35+4 = 39

    Tendo o A9 dá para encontrar o A11

    A11 = A9 + 4

    A11 = 39 + 4 = 43

    Item certo

  • Resolvido:

    https://youtu.be/pCU4cYDEtc8

  • 23 -----> 32 ----> 27 ----> 36 ----> 31 ----> 40 ---> 35 ----> 44 ---> X ---- Y ----> Z

    23+4= 27

    27+4= 31

    31+4= 35

    35+4= 39 ( X= 39 )

    39+4= 43 ( Z= 43 )

    _____________________________________________________________

    32+4= 36

    36+4= 40

    40+4= 44

    44+4= 48 ( Y= 48)


ID
694033
Banca
FCC
Órgão
Prefeitura de São Paulo - SP
Ano
2012
Provas
Disciplina
Raciocínio Lógico
Assuntos

Para a prova final de um concurso de televisão, serão colocadas 20 caixas no palco, numeradas de 1 a 20. Em cada caixa, haverá uma pista diferente, que ajudará a desvendar o enigma da noite. Um a um, os 20 concorrentes serão sorteados para ter acesso às pistas, de acordo com a seguinte regra:

- o 1o sorteado lerá as pistas das caixas 1, 2, 3, 4, 5, 6, 7, 8, 9, 10, 11, 12, 13, 14, 15, 16, 17, 18, 19 e 20,

- o 2o sorteado lerá apenas as pistas das caixas 2, 4, 6, 8, 10, 12, 14, 16, 18 e 20,

- o 3o sorteado lerá apenas as pistas das caixas 3, 6, 9, 12, 15 e 18,

- o 4º sorteado lerá apenas as pistas das caixas 4, 8, 12, 16 e 20,

- o 5o sorteado lerá apenas as pistas das caixas 5, 10, 15 e 20,

- o 6o sorteado lerá apenas as pistas das caixas 6, 12 e 18,

e assim sucessivamente, até o 20o sorteado, que só lerá a pista da caixa 20.

Algumas pistas serão lidas por um número par de concorrentes e as demais serão lidas por um número ímpar de concorrentes. A quantidade de pistas lidas por um número ímpar de concorrentes é

Alternativas
Comentários
  • Demorei pra fazer. resposta letra A
    Para resolver esta questão é bom observar que a leitura de uma pista não excluirá a possibilidade de outro ler a mesma pista.

    Assim, verifica-se que a possibilidade de cada candidato está ligada ao múltiplo de sua ordem, ou seja, 0 2° candidato vai ler as pistas múltiplas de 2, e assim vai.
    O último enunciado é confuso ("Algumas pistas serão lidas por um número par de concorrentes e as demais serão lidas por um número ímpar de concorrentes.")

    Avaliando as possibilidades de leituras de pista, podemos ver que o 1° lerá todas, e somente ele lerá a pista 1 (1° número ímpar), portanto:
    A pista 1 será lida 1x (pelo 1°)
    A pista 4 será lida 3x (pelo 1°, 2° e 4°)
    A pista 9 será lida 3x (pelo 1°, 3° e 9°)
    A pista 16 será lida 5x (pelo 1°, 2°, 4°, 8°, 16°)

    todas as outras serão lidas por um número par de candidatos.
  • Padrão:
    o número de pistas lidas pelo 1° candidato  é múltiplo de 1 {1,2,3,4...}
    o número de pistas lidas pelo 2° candidato  é múltiplo de 2{2,4,6,8...}
    o número de pistas lidas pelo 3° candidato  é múltiplo de 3
    o número de pistas lidas pelo 4° candidato  é múltiplo de 4
    E assim vai até o 20° candidato, que lerá um número múltiplo de 20 (somente 1 pista:a 20°)

    O 1° candidato leu todas as pistas, então devemos comparar com ele as pistas lidas pelos outros. Números primos serão lidos somente 1 vez, totalizando 2 (par), ficando fora da contagem.

    pista-cand

    -1 I  
    2 II
    3 II
    -4 III
    5 II
    6 IIII
    7 II
    8 IIII
    -9 III
    10IIII
    11II
    12IIIIII
    13II
    -14III
    15IIII
    -16IIIII
    17II
    18IIIIII
    19II
    -20IIIIII

    pistas lidas n° impar= 4
  • As pistas que cada sorteado leu
    01o sorteado: {1, 2, 3, 4, 5, 6, 7, 8, 9, 10, 11, 12, 13, 14, 15, 16, 17, 18, 19 e 20}
    02o sorteado: {X, 2, X, 4, X, 6, X, 8, X, 10, XX, 12, XX, 14, XX, 16, XX, 18, XX e 20}
    03o sorteado: {X, X, 3, X, X, 6, X, X, 9, XX, XX, 12, XX, XX, 15, XX, XX, 18, XX e XX}
    04o sorteado: {X, X, X, 4, X, X, X, 8, X, XX, XX, 12, XX, XX, XX, 16, XX, XX, XX e 20}
    05o sorteado: {X, X, X, X, 5, X, X, X, X, 10, XX, XX, XX, XX, 15, XX, XX, XX, XX e 20}
    06o sorteado: {X, X, X, X, X, 6, X, X, X, XX, XX, 12, XX, XX, XX, XX, XX, 18, XX e XX}
    07o sorteado: {X, X, X, X, X, X, 7, X, X, XX, XX, XX, XX, 14, XX, XX, XX, XX, XX e XX}
    08o sorteado: {X, X, X, X, X, X, X, 8, X, XX, XX, XX, XX, XX, XX, 16, XX, XX, XX e XX}
    09o sorteado: {X, X, X, X, X, X, X, X, 9, XX, XX, XX, XX, XX, XX, XX, XX, 18, XX e XX}
    10o sorteado: {X, X, X, X, X, X, X, X, X, 10, XX, XX, XX, XX, XX, XX, XX, XX, XX e 20}
    11o sorteado: {X, X, X, X, X, X, X, X, X, XX, 11, XX, XX, XX, XX, XX, XX, XX, XX e XX}
    12o sorteado: {X, X, X, X, X, X, X, X, X, XX, XX, 12, XX, XX, XX, XX, XX, XX, XX e XX}
    13o sorteado: {X, X, X, X, X, X, X, X, X, XX, XX, XX, 13, XX, XX, XX, XX, XX, XX e XX}
    14o sorteado: {X, X, X, X, X, X, X, X, X, XX, XX, XX, XX, 14, XX, XX, XX, XX, XX e XX}
    15o sorteado: {X, X, X, X, X, X, X, X, X, XX, XX, XX, XX, XX, 15, XX, XX, XX, XX e XX}
    16o sorteado: {X, X, X, X, X, X, X, X, X, XX, XX, XX, XX, XX, XX, 16, XX, XX, XX e XX}
    17o sorteado: {X, X, X, X, X, X, X, X, X, XX, XX, XX, XX, XX, XX, XX, 17, XX, XX e XX}
    18o sorteado: {X, X, X, X, X, X, X, X, X, XX, XX, XX, XX, XX, XX, XX, XX, 18, XX e XX}
    19o sorteado: {X, X, X, X, X, X, X, X, X, XX, XX, XX, XX, XX, XX, XX, XX, XX, 19 e XX}
    20o sorteado: {X, X, X, X, X, X, X, X, X, XX, XX, XX, XX, XX, XX, XX, XX, XX, XX e 20}

    A quantidade de pistas lidas por um número ímpar de concorrentes = 04
    A pista 1: lida 
    1 vez (1º sorteado)
    A pista 4: lida 
    3 vezes (1º, 2º e 4º sorteados)
    A pista 9: lida 
    3 vezes (1º, 3º e 9º sorteados)
    A pista 16: lida 
    5 vezes (1º, 2º, 4º, 8º e 16º sorteados)
    O restante, como pode ser observado acima, foram lidas 2, 4 ou 6 vezes (
    número par de concorrentes)

    Espero ter ajudado. Bons estudos!!
  • Na boa, até endenter o enunciado acabou a prova...

    Tem que estar inspirado para abstrair que "A quantidade de pistas lidas por um número ímpar de concorrentes"  que dizer  "A quantidate de pistas lidas de um mesmo baú  que o total é impar.

    Muita fé...
  • Aproveitando os comentários acima, só acrescento um detalhe: reparem que o número de concorrentes que leem as pistas corresponde ao número de divisores do número que representa a caixa.
    Ex.: caixa 1 - Quais são os divisores de 1? R: Ele mesmo. Portanto, um divisor. - um concorrente lerá essa pista
    caixa 2- Quais são os divisores de 2? R: 1 e 2. Portanto, 2 divisores. - dois concorrentes lerão essa pista
    caixa 3 - Quais são os divisores de 3? R: 1 e 3. Portanto, 2 divisores. - dois concorrentes lerão essa pista
    caixa 4- Quais são os divisores de 4? R: 1, 2 e 4. Portanto, 3 divisores. - três concorrentes lerão essa pista
    E assim por diante.
    Lembrete:
    1) Como encontrar os divisores de 6 - Pense nas multiplicações que dão 6: 1x6, 2x3. Logo, os divisores de 6 são: D(6) = {1,2,3,6}
    D(9) = ? Pense nas multiplicações que dão 9 - 1x9, 3x3. Logo, D(9) = {1,3,9}

    2) Números primos só têm 2 divisores: o um e ele mesmo. Por quê? Pq só há uma multiplicação que tem como resultado esse nº primo.
    Ex.: D(7) = {1,7} Pense nas multiplicações que dão 7 - só há 1x7. Portanto, seus divisores são 1 e 7.
    Quais são os números primos de 1 a 20? 2,3,5,7,11,13,17,19. Todas as caixas representadas por esses números serão lidas por 2 concorrentes. Logo, vou excluí-las, pq a questão pede a  quantidade de pistas lidas por um número ÍMPAR de concorrentes.
    Com isso, fico com as pistas das caixas 1, 4, 9 e 16. Portanto, 4. Resp. A
    Provando:
    D(1) = {1} - 1 divisor
    D(4) = {1,2,4} - 3 divisores
    D(9) = {1,3,9} - 3 divisores
    D(16) = {1,2,4,8,16} - 5 divisores
    Reparem que os quadrados perfeitos de 1 a 20 são: 1,4,9 e 16. Exatamente os números que representavam as caixas que eram a solução da questão.
    Espero ter ajudado.
     
  • Pessoal veja  que iinteressante!!!!!!!!!

    números primos
    1, 3 ,5; 7; .....................

    se eu começar pelo primeiro:
     1.......

    ao segundo primo, soma-se  a conta anterior:
    1 + 3 = 4

    ao terceiro primo, soma-se a conta anterior:
    5 + 4 = 9

    ao quarto primo, soma-se a conta anterior:
    7 + 9 = 16

    Comparando a tabelas apresentadas, a quantidade de dicas que são lidos por pessoas impares (ou seja, 1 ou 3 ou 5 ou 7........), são esses apresentados. 1, 4, 9, 16.

    Se observarem a progressão dos primos:

    1-3-5-7. pegue o primeiro e vá somando os outros.

  • Gente, demorou mas eu fiz assim:
    1 concorrente vai ler as caixas 1,2,3....20
    2 concorrente as caixas 2,4,6,8,10,12,14,16,18,20
    3 concorrente caixas 3,6,9,12,15,18
    4 concorrente caixas 4,8,12,16,20
    5 concorrente caixas 5,10,15,20
    6 concorrente caixas 6,12,18
    Notei o seguinte: o número posterior é a soma do anterior. Se o anterior é 5, 5+5 = 10. 15 = 10+5. 20 = 15=10.
    Aí fui montando,
    7 concorrente = 7, 14 (7+7)
    Assim por diante até o 20.
    Vi quantas candidatos veriam a pista 1, a pista 2, a pista 3, até a pista 20; e isso me mostrou que a pista 1 só ia ser mostrada para um concorrente, a pista 4 só seria mostrada para 3 concorrentes, a pista 9 para 3 concorrentes e a pista 15 para 5 concorrentes. O restaante das pistas seriam vistas por candidatos 'par'.... então fechou 4! =)
  • Depois de muito ler os comentários a cima, entendi a o que questão estava pedindo. Muito mal feito o enunciado.
  • Pessoal, apesar de todas as explicações anteriores somente entendi com esta explicação:

    1. Para descobrir quais pistas cada sorteado leu, basta observar que o número da 1ª pista/caixa lida é igual ao número do sorteado. Os números das outras caixas são múltiplos da 1ª. Ex.:

    3º sorteado: caixa 3, caixa 3x2=6, caixa 3x3=9, caixa 3x4=12, caixa 3x5=15, caixa 3x6=18 = 3, 6, 9, 12, 15, 18
    5º sorteado: caixa 5, caixa 5x2=10, caixa 5x3=15, caixa 5x4=20 = 5, 10, 15, 20

    Logo, a lista de caixa/pistas ficaria assim:
    1º sorteado: 1, 2, 3, 4, 5, 6, 7, 8, 9, 10, 11, 12, 13, 14, 15, 16, 17, 18, 19, 20
    2º sorteado: 2, 4, 6, 8, 10, 12, 14, 16, 18, 20
    3º sorteado: 3, 6, 9, 12, 15, 18
    4º sorteado: 4, 8, 12, 16, 20
    5º sorteado: 5, 10, 15, 20
    6º sorteado: 6, 12, 18
    7º sorteado: 7, 14
    8º sorteado: 8, 16
    9º sorteado: 9, 18
    10º sorteado: 10, 20
    11º sorteado: 11
    12º sorteado: 12
    13º sorteado: 13
    14º sorteado: 14
    15º sorteado: 15
    16º sorteado: 16
    17º sorteado: 17
    18º sorteado: 18
    19º sorteado: 19
    20º sorteado: 20

    2. Agora é só contar quantas vezes pista/caixa aparece na lista:
    pista/caixa 1: 1x
    pista/caixa 2: 2x
    pista/caixa 3: 2x
    pista/caixa 4: 3x
    pista/caixa 5: 2x
    pista/caixa 6: 4x
    pista/caixa 7: 2x
    pista/caixa 8: 4x
    pista/caixa 9: 3x
    pista/caixa 10: 4x
    pista/caixa 11: 2x
    pista/caixa 12: 6x
    pista/caixa 13: 2x
    pista/caixa 14: 4x
    pista/caixa 15: 4x
    pista/caixa 16: 5x
    pista/caixa 17: 2x
    pista/caixa 18: 6x
    pista/caixa 19: 2x
    pista/caixa 20: 6x

    3. Temos, portanto, 4 pistas (1, 4, 9 e 16) que são lidas um número ímpar de vezes.

    fonte: http://www.forumconcurseiros.com/forum/showthread.php?t=316092
  • Todos os quadrados perfeitos abaixo de 20.
  • Percebam que as pistas lidas por vezes ímpar são 1,4,9,16. Nesse raciocínio, seriam também lidas ímpar vezes as pistas (caso houvessem) 25, 36, 49...

  • Vamos escrever quais pistas cada concorrente irá ler e depois contamos.

    Observamos que cada concorrente tem acesso às pistas referente aos múltiplos do número de sua ordem de sorteio.

    1º: 1, 2, 3, 4, 5, 6, 7, 8, 9, 10, 11, 12, 13, 14, 15, 16, 17, 18, 19, 20;

    2º: 2, 4, 6, 8, 10, 12, 14, 16, 18, 20;

    3º: 3, 6, 9, 12, 15, 18;

    4º: 4, 8, 12, 16, 20;

    5º: 5, 10, 15, 20;

    6º: 6, 12, 18;

    7º: 7, 14;

    8º: 8, 16;

    9º: 9, 18;

    10º: 10, 20;

    11º: 11

    12º: 12

    13º: 13

    14º: 14

    15º: 15

    16º: 16

    17º: 17

    18º: 18

    19º: 19

    20º: 20

     

    Logo: as pistas que serão lidas por nº impar de concorrentes são:

    1: uma vez

    4: três vezes

    9: três vezes

    16: cinco vezes

     

    Gabarito: Letra A

     

    http://rlmparaconcursos.blogspot.com.br/

  • Utilizei os divisores de cada número :

    1- {1} - Ímpar

    2-{1;2}- Par

    3-{1;3} - Par

    4-{1;2;4}-Ímpar

    ...

    20-{1;2;4;5;10;20}- Par


    Resposta={1,4,9,16} Todos possuem número de divisores ímpares, consequentemente serão lidos em um número ímpar de vezes.

  • "A QUANTIDADE DE PISTAS LIDAS POR UM NUMERO IMPAR DE CONCORRENTES É"

    SO EU OU MAIS ALGUEM PERCEBEU QUE PERGUNTA CONFUNDE OU ESTÁ ERRADA??

    DESSA FORMA, DÁ A ENTENDER QUE O ENUNCIADO DESEJA SABER QUAIS FORAM OS CONCORRENTES IMPARES E ASSIM SOMAR TODAS AS SUAS PISTAS DISPONÍVEIS

    ACREDITO QUE O CERTO SERIA:

    A QUANTIDADE DE PISTAS LIDAS POR UM NUMERO IMPAR DE VEZES

    PISTA 1 : LIDA APENAS 01 VEZ

    PISTA 4 : LIDA 03 VEZES

    PISTA 9 : LIDA 3 VEZES

    PISTA 16: LIDA 5 VEZES

    SOMENTE ESSAS 4 PISTAS FORAM LIDAS IMPARES VEZES


ID
695617
Banca
FCC
Órgão
TRF - 2ª REGIÃO
Ano
2012
Provas
Disciplina
Raciocínio Lógico
Assuntos

Sabe-se que exatamente quatro dos cinco grupos de letras abaixo têm uma característica comum.

BCFE - HILK - JKNM - PQTS - RSUV

Considerando que a ordem alfabética adotada é a oficial, o único grupo de letras que NÃO apresenta a característica comum dos demais é:

Alternativas
Comentários
  • A característica em comum que as sequencias tem é a seguinte: LETRA X, LETRA X+1, LETRA X+4, LETRA X+3
    obs: X é a posição da 1ª letra do grupo em relação da alfabeto

    Ex: 1ª letra = B, 
    logo x = 2

    Portanto: 2, 3, 6, 5
    Substituindo as posições pelas letras fica: B,C,F,E

    Concluindo: encontrada a característica em comum percebe-se que na letra "e" (RSUV) o certo seria (RSVU)

  • A sequência sempre, a partir da 1º letra, pega a próxima letra do alfabeto, depois pula duas letras e pega a próxima e, por último, volta para a letra anterior, vejamos:
    obs: tomemos o novo alfabeto como base.
    " A, B, C, D, E, F, G, H, I, J, K, L, M, N, O, P, Q, R, S, T, U, V, W, X, Y e Z."

    Sequência 01: BCFE 
    A, B, C, D, E, F, G...
       1º 2°  4°  3°     (CERTO)
    Sequência 02: HILK

    ...H, I, J, K, L, M...
      1º 2°    4°  3°      (CERTO)
    Sequência 03: JKNM
    ...J, K, L, M, N, O...
      1º 2°   4° 3°      (CERTO)
    Sequência 04: PQTS
    ...P, Q, R, S, T, U...
     1º  2°  4° 3°      (CERTO)
    Sequência 05: RSUV
    ...R, S, T, U, V, W... (ERRADO)
     Vejam que, por um discuido, o concurseiro desatento acaba errando a questão. Na sequência 05, a FCC trocou a ordem, o certo seria RSVU.
  • Questão fácil, mas se o candidato não estiver atento acaba perdendo a questão!

  • Na minha lógica nada matemática, resolvi da seguinte forma: escolhi a única sequência que não tinha as 2 últimas letras invertidas e cheguei na mesma resposta. Se era pra achar uma diferença, essa era uma!!

    Mas não recomendo o método por ser falho, mas na hora do desespero pode ajudar!!
  • Letra E.
    .
    .

  • As letras possuem uma sequência lógica que seguem na ordem de dois em dois, a primeira parte conta da esquerda para a direita e pula uma letra, a segunda conta da direita para a esquerda.

    Vejamos:   

    B C (D) F E ; H I (J) L K : J K (L) N M ; P Q (R) T S : R S (T) U V nesse caso o certo seria  R S (T) V U.
  • Para facilitar escrevi o alfabeto assim:

    A1 – B2 – C3 – D4 – E5 – F6 – G7 – H8 – I9 – J10 – K11 – L12 – M13 – N14 – O15 – P16 – Q17 – R18 – S19 – T20 – U21 – V22 – W23 – X24 – Y25 – Z26

    Logo transformando as sequências de letras em sequencias de números:

    BCFE - HILK - JKNM - PQTS - RSUV
    2-3-6-5 / 8-9-12-11 / 10-11-14-13/16-17-20-19/18-19-21-22


    Em cada sequência de números que entre os dois primeiros números e os dois últimos números falta o número imediatamente maior do que o segundo da sequência E que os dois últimos ficam na ordem ivnersa. Veja:

    2 – 3 (pula o 4) – 6-5 (inverte a ordem)
    8 –  9 (pula o 10) – 12-11
    (inverte a ordem)
    10 – 11 (pula o 12) – 14-13 (inverte a ordem)
    16 – 17 (pula o 18) – 20-19 (inverte a ordem)

    Porém a última sequência desrespeitou a regra, veja:

    18 – 19 (pula o 20) – 21-22 (aqui a ordem não foi invertida)

    Letra E
  • Gabarito: E

     

    BCFE - HILK - JKNM - PQTS - RSUV 

    1º grupo: BCFE = BCDEF --> Podemos observar que as duas últimas letras se inverteram.

    2º grupo: HILK​ = HIJKL --> Podemos observar que as duas últimas letras se inverteram.

    3º grupo: JKNM= JKLMN --> Podemos observar que as duas últimas letras se inverteram.

    4º grupo: PQTS = PQRST --> Podemos observar que as duas últimas letras se inverteram.

    5º grupo: RSUV = RSTUV  --> Podemos observar que as duas últimas letras NÃO se inverteram, portanto, é o único grupo de letras que NÃO apresenta a característica comum dos demais

     


ID
695620
Banca
FCC
Órgão
TRF - 2ª REGIÃO
Ano
2012
Provas
Disciplina
Raciocínio Lógico
Assuntos

Considere que os termos da sucessão seguinte foram obtidos segundo determinado padrão.

(20, 21, 19, 22, 18, 23, 17, ...)


Se, de acordo com o padrão estabelecido, X e Y são o décimo e o décimo terceiro termos dessa sucessão, então a razão  Y/é igual a

Alternativas
Comentários
  • a8=24
    a9=16
    a10=25=x
    a11=15
    a12=26
    a13=14=y

    14/25

    x=0.56
  • Se alguém não conseguiu vizualizar os padrões,
    vou demonstrar os três mais fáceis de serem
    observados.

    Somando-se os termos de dois em dois, teremos
    o resultado 41

    20+21=41
    19+22=41
    18+23=41
    17+24=41

    e por ai vai

    A questão apresenta um padrão decrescente: ( X - 1 )

    20;21;19;22;18;23;17;24;16;25;15;26;14

    E um outro crescente: ( X+1 )

    20;21;19;22;18;23;17;24;16;25;15;26;14

    O décimo termo é 25, que corresponde a X.
    O décimo terceiro termo é 14, que corresponde a Y.

    A razão entre Y / X = 14/25 = 0,56
    0,56 * 100  =  56% 


  • observe que nessa sequencia 20,21,19,22,18,23,17... um número é somado e outro é subtraído a partir do 1 ,o módulo desse número sendo crescente,observe o exemplo para ficar mais claro: (20+1)=21/(21-2)=19/(19+3)=22/(22-4)=18/(18+5)=23/(23-6)=17
    veja que é só ir somando na sequencia por 1,-2,3,-4,5,-6.....ou seja o módulo vai crescendo 1,2,3,4,5,6....sendo que um é positivo e outro negativo,um positivo e outro negativo.assim encontramos o  termo 13=14 e o termo 10=25 dividimos 14/25 que é igual a 0,56 ou 56%.resposta da questão.
     

  • Letra C.
    (20, 21, 19, 22, 18,  23, 17...). A sucessão encontrada foi +1, -2, +3, -4, +5, -6,
    A sequencia da lógca seguirá até encontrar o 10o e 13o termo.
    (24, 16,  25,  15, 26, 14) continuando a sucessão encontramos X = 25 e Y = 14.
    Imagem 005.jpg= 14/25 dividino achamos 56.
  • Eu vi um jeito mais fácil de fazer

    Olhem so as duas sequencias:
    A de cima sempre diminui um número:
    20 - 19 - 18 - 17 - 16 - 15 - 14
    A de baixo sempre aumenta um número:
    21 - 22 - 23 - 24 - 25 - 26 - 27

    Montando este esquema iniciaremos pela sequencia de cima, já que 20 está lá, e depois colocaremos o 21 (sequencia de baixo), e assim indefinidamente colocaremos sempre um número de cima e um número de baixo. Ficará assim. 

    20 21 19 22 18 23 17 24 16 25 15 26 14 27 (...)

    A questão pede X (Décimo Número)  que é 25
    E pede também o Y (Décimo Terceiro Número) que é 14.

    Por fim pede para dividir Y por X, que ficará assim 14/25.

    O resultado de 14/25 é 0,56 ou seja 56%.

    Deste jeito fiz acho que simplifica muito mais a questão, pois montando este esquema em menos de 2 minutos resolve esta questão, e na hora da prova, tempo é tudo, principalmente em matemática ou raciocínio lógico.
  • Observamos que:

    a2=a1+1
    a3=a1-1             
    a4=a1+2                                
    a5=a1-2                                  
    a6=a1+3
    a7=a1-3
    .
    .
    .



     NOTA 1: OS  TERMOS "a" IMPARES SEMPRE SUBTRAEM (ex; a3,a5,a7..)
     NOTA 2: AS SEQUÊNCIAS  SEGUEM SEMPRE O PADRÃO "2 A  2" (ex; +1,-1,+2,-2,+3,-3...)

                                              PORTANTO: a10= a1+ 5 ; a13= a1 -6 
    a10=20 +5=25 ; a13=20-6=14

    14/25=0,56=56%
  • a1   a2     a3     a4     a5     a6     a7     a8    a9    a10  

    20   1      -2     3      -4     5      -6     7     -8    9  

    a10 = 20 -4 + 9 = 25

    a1     a2     a3     a4     a5     a6    a7    a8    a9    a10   a11   a12   a13

    20     1      -2     3      -4     5     -6    7     -8    9     -10   11    -12

    a23 = 20 - 6 = 14

    14/25  = 28/50 = 56/100


  • 20 (1) + 1 = 21 (2)

    21 - 2 = 19 (3)

    19 + 3 = 22 (4)

    22 - 4 = 18 (5)

    18 + 5 = 23 (6)

    23 - 6 = 17 (7)

    17 + 7 = 24 (8)

    24 - 8 = 16 (9)

    16 + 9 = 25 (10)

    25 - 10 = 15 (11)

    15 + 11 = 26 (12)

    26 - 12 = 14 (13)


    14/25 = 0,56

  • De acordo com o enunciado, pode-se dividir os termos da sucessão em pares e ímpares.

    a1 = 20  a2 = 21

    a3 = 19  a4 = 22

    a5 = 18  a6 = 23

    a7 = 17  a8 = 24

    a9 = 16  a10 = 25

    a11 = 15

    a13 = 14


    Sendo assim, verifica-se que o décimo termo (a10) e o décimo terceiro (a13) são:

    X = 25 e Y = 14

    Finalmente, a razão Y/X é igual a:

    Y/X = 14/25 = 0,56 = 56%

    Resposta C.


  • 20(+1)21 (-2)19 (+3)22 (-4)18 (+5)23 (-6)17 (+7)24 (-8)16 (+9)25 (-10)15 (+11)26(-12)14...

    Atenção: 20 soma com o +1 que resulta no próximo número da sequência, 21, e este subtrai com -2 que resulta também no próximo número 19...

    Contando em ordem numérica os números fora dos parênteses, logo temos:

    O décimo: 25 = x

    O décimo terceiro: 14 = y

    Y/X = 14/25 multiplica 100/x

    25x=1400

    x=1400/25

    x=56%

    LETRA "C"


  • n = 20

    n, (n+1), (n-1), (n+2), (n-2)...

    logo, 10º = (n+5) e 13º (n-6)

    =>(14/25)100 = 56%

    letra C 

  • Eu ordenei os números: 14,  15, 16, 17, 18, 19,  20,  21,  22,  23,  24,  25,  26

    e ficou assim:              13º, 11º, 9º, 7º, 5º, 3º, 1º,  2º,  4º,  6º,  8º, 10º, 12º                                

    Muito mais simples que somar e diminuir um monte de números.

    13º=14 e 10º=25

    14/25= 0,56

    Gab: C

  • eu fiz assim:

     

    (20, 21, 19, 22, 18, 23, 17, ...)

    1- TERMO= 20,

    a partir dele a sequencia é assim: +1, -2, +3, -4,+5, -6,+7,-8,+9 -10, +11,-12

    20, (20+1)21, (21-2)19,(19+3)22,(22-4)18, .... 17,(17+7)24, (24-8)16,(16+9)25 ---> esse é o 10 TERMO. O 13 É IGUAL A 26-12=14.

    X=DECIMO=25

    Y= DECIMO TERCEIRO = 14

    14/25= (14X4)= (25X4) = 56/100= 56%. 

     

    erros, avise-me.

    GABARITO ''C''

     

  • 20, 21, 19, 22, 18, 23, 17.... 24 16 25 15 26 14 

    Sequência azul = subtraindo 1

    Sequência vermelha = somando 1 

     

    x = 25

    y = 14

     

    y/x = 14/25  (x 4) =>  56/100 ou 56%

  • Explicação em vídeo: https://www.youtube.com/watch?v=0hnkqqoGrO4


ID
696460
Banca
FCC
Órgão
TJ-RJ
Ano
2012
Provas
Disciplina
Raciocínio Lógico
Assuntos

Na primeira fase de um jogo de computador, um gato verde e outro vermelho perseguem um ratinho, controlado pelo jogador, por toda a tela. Cada vez que o jogo muda de fase, o número de gatos verdes na tela é duplicado e surgem três novos gatos vermelhos. Ao se iniciar a décima fase do jogo, o ratinho será perseguido por um total de

Alternativas
Comentários
  • A questão basea-se na soma da quantidade de gatos verdes e de gatos vermelhos após as 10 fases de um determinado jogo. 

    Quanto a quantidade de gatos verdes trata-se de uma duplicação simples, onde na primeira fase temos apenas um gato e assim deveremos duplicar até chegar a 10º fase. O que pode levar muitos candidatos ao erro é achar que se trata de multiplicar por 2 dez vezes, mas na verdade a primeira fase tem apenas um gato. Para se achar o nº de gatos verdes o cálculo deve ser da seguinte forma: 
    1x2x2x2x2x2x2x2x2x2=512 

    Quanto a quantidade de gatos vermelhos é mais simples, o enunciado afirma que só existe um gato na primeira fase e a cada fase que se passa, até a décima, serão acrescentado mais 3 gatos vermelhos então o cálculo será da seguinte forma: 
    1+3+3+3+3+3+3+3+3+3= 28

    No final soma-se a quantidade de gatos verdes mais a quantidade de gatos vermelhos e chegamos ao número de 540 gatos. 

  • Gatos vermelhos (progressão aritmética)

    An = ? | A1 = 1 | n = 10 | r = 3
    An = A1 + (n – 1)r
    An = 1 + 9*3 = 28
    Gatos verdes (progressão geométrica)

    An = ? | A1 = 1 | n = 10 | q = 2
    an = a1 * q^(n-1)
    An = 1 * 2 ^ 9 = 512

    512 + 28 = 540 (alternativa b)
  • an=?
    a1=1
    q=2
    n=10
    an=a1*q^n-1
    an=1*2^9
    an=512

    an=?
    a1=1
    r=3
    n=10
    an=a1+(n-1)*r
    an=1+9*3
    an=28

    512+28=b) 540 gatos.
  • 29 + 1 + 27 = 512 + 1 + 27 =  540


  • Logo, 512 + 28 = 540
  • Se tivesse uma resposta ''539'', por desatenção, eu erraria kkkk


ID
703432
Banca
AOCP
Órgão
BRDE
Ano
2012
Provas
Disciplina
Raciocínio Lógico
Assuntos

Complete o elemento faltante, considerando a sequência a seguir:
1 2 4 8 ? 32 64

Alternativas
Comentários
  • D) 16

    Questão pra constar se estudou alguma vez na vida.

    Progressão de 2

    N x 2 , (N x 2) x 2 ...

    1x2 = 2 , 2 x 2 = 4, 4x2 = 8, 8x2 = 16, 16x2 = 32, 32x2 = 64

  • 16.
    Potências de base 2.
    1 = 2^0
    2 = 2^1
    4 = 2^2
    8 = 2^3
    16 = 2^4
    32 = 2^5
    64 = 2^6
  • simples, e so usar progressão x2

    Rumo a aprovação!!
  • PENSEI QUE NÃO EXISTIA QUESTÃO ASSIM.
  • Quando olhei essa questão, não acreditei...

  • É uma sequência cuja razão é ×2. Bons estudos. 

  • letra D) 16

    1,2,4,8,16,32,64,128........

  • 5% ainda erram... deve ser sacanagem...kkkkkkk


ID
718210
Banca
PC-SP
Órgão
PC-SP
Ano
2012
Provas
Disciplina
Raciocínio Lógico
Assuntos

Qual o próximo número da sequência (16, 25, 36, 49, 64,...)?

Alternativas
Comentários
  • Potências de n°:

    4²,5²,6²,7²,8².9²...

    16, 25, 36, 49, 64, 81...
  • Pessoal questão tranquila, apenas acrescentando o cometário do colega.

    a sequência de números foram aumentandos de 2 em 2
    .
    16-25 =  9       16 + 9 = 25
    25-36 = 11       25+ 11 = 36
    36-49 = 13       36 + 13 = 49
    49-64 = 15       49 + 15 = 64
    64-81 = 17
              64 + 17 = 81
  •   A sequência é formada pelos números elevados ao quadrado, ou seja, cada número multiplicado por ele próprio. O último da sequência era 8, que multiplicado por ele próprio resulta em 64. Logo, o próximo é o 9, que multiplicado por ele próprio é 81.
     
      Alternativa "b".
  • É só perceber que a sequência se dá por somar os números citamos na questão pelos números ímpares.
    Para somar primeiro, perceba a diferença entre 25 e 16 que é 9. Agora, dai em diante somamos pelo próximo número ímpar.
    16+9=25
    25+11=36
    36+13=49
    49+15=64
    64+17= 81.


  • 16..............................25..............................36..............................49..............................64

    ..............+9..............................+11..............................+13.........................+15............................+17

    64 + 17 = 81

  • eu acabei pensando assim:

    4x4=16

    5x5=25

    6x6=36

    7x7=49

    8x8=64

    9x9=81


ID
726829
Banca
INSTITUTO CIDADES
Órgão
TCM-GO
Ano
2012
Provas
Disciplina
Raciocínio Lógico
Assuntos

Observe a seguinte série:

53, 53, 40, 40, 27, 27, ...

Com base na sequência acima, indique a alternativa com o próximo número

Alternativas
Comentários
  • Existe uma sequencia lógica para a questão em si:
    Suponhamos que seja:

    X, X, X-13, X-13, X *2*(-13), X *2*(-13), X *3*(-13), X *3*(-13)
  • Há apenas uma diferença de "13" entre os termos
  • Logo, próx. 27-13=14

  • De dois em dois : 53, 53,  40, 4027, 27, ...

    Como o valor está diminuindo, devemos subtrair os números.

     53,  40   :

       53 - 40 = 13 

     4027    :

    40 - 27 = 13 - logo 13 é o número padrão da sequencia, que deverá ser usado para achar o valor exato.

    27 - 13 = 14  - Resposta!!

    Letra A






  • Bem rapidinho.
     Veja que: 
    53-40=13 
    40-27=13
     Assim o próximo número será 
    27 -____= 13 
     Portanto: 27-14=13
     Letra "a"
  • 53-40=13

    40-27=13

    27-13=??? 

    Gabarito A

  • Que as questões que caiam no TCM-RJ  sejam assim!!! kkkkkkkk


ID
749416
Banca
VUNESP
Órgão
TJM-SP
Ano
2011
Provas
Disciplina
Raciocínio Lógico
Assuntos

Dadas as sequências A e B.

Sequência A: 7; 13; 25; 49; . .

Sequência B: 5; 9; 17; 33; . .

A diferença entre o 2.° termo da sequência A e o 1.° termo da sequência B é 8. A diferença entre o 3.° termo da sequência A e o 2.° termo da sequência B é 16. A diferença entre o 10.° termo da sequência A e o 9.° termo da sequência B é um número entre

Alternativas
Comentários
  • Resposta Correta: Letra C

    Observem que:
    (2°A) - (1°B) = 8;
    (3°A) - (2°B) = 16;
    Ao analisar que (4°A) - (3°B) = 32
    Vejam que a diferença sempre vai se ampliando em 2 vezes!!
    Podemos deduzir que (10°A) - (9°B) = 8 * 2^8 = 2048
    2^8 , pois (a diferença entre o 10° elemento de A e o 2° elemento são 8 termos!)
    Conferindo:
    (1°A) - (0°B) = 4; (em que 0°A = 4 e 0°B = 3)
    (2°A) - (1°B) = 8;
    (3°A) - (2°B) = 16;
    (4°A) - (3°B) = 32;
    (5°A) - (4°B) = 64;
    (6°A) - (5°B) = 128;
    (7°A) - (6°B) = 256;
    (8°A) - (7°B) = 512;
    (9°A) - (8°B) = 1024;
    (10°A) - (9°B) = 2048;
  • A>  7; (+7-1)... 13; (13-1)... 25; (+24)... 49; (+48)... 97; (+96)... 193; (+192)... 385; (+384)... 769; (+768)... 1537; (+1536)... 3073

    B>  5; (+4)... 9; (+8)... 17; (+16)... 33; (+32)... 65; (+64)... 129; (+128)... 257; (+256)... 513; (+512)... 1025


    3073 - 1025  =  2048

  • 16 x 2 elevado à sétima = 16 x 128 = 2048.

    Do terceiro do A até ao décimo do A e do segundo do B até ao nono do B são sete dobras de valor; o valor vai dobrar mais sete vezes. Por isso 2 elevado à sétima.

    Não precisa fazer a sequência inteira, não dá tempo na prova.

  • fiz na mão mesmo, teve um padrão que ajudou na resolução: o número anterior era sempre (um) a menos que o seguinte, claro que não faria de cabeça o resto, já estava na m*rda mesmo, mas ajudou a não vacilar na conta.


ID
749428
Banca
VUNESP
Órgão
TJM-SP
Ano
2011
Provas
Disciplina
Raciocínio Lógico
Assuntos

Na sequência a seguir, cada número pertence a apenas uma de duas categorias de números, sendo que o número 1 não pertence a nenhuma dessas categorias. Os números impressos em negrito e sublinhados são elementos de uma das categorias, e observá-los irá facilitar a identificação de como a sequência é formada.

4; 6; 2; 8; 9; 3; 10; 12; 5; 14; 15; 7; 16; 18; 11; 20; 21; 13;; 22; 24; 17;; e segue ilimitadamente segundo a lei de formação.

O primeiro número, em negrito,maior que todos os anteriores da sequência até a sua posição é

Alternativas
Comentários
  • Resposta Correta: Letra C

    Vamos separar cada 3 números na sequência em 3 sequências:

    A = {4,8,10,14,16,20,22,26,28,32,34,38,40...}  -> Soma +2 e Soma +4...Soma +2 e Soma +4
    B = {6,9,12,15,18,21,24,27,30,33,36,39,41...} -> Soma + 3 SEMPRE!
    C = {2,3,5,7,11,13,17,19,23,29,31,37,41,43...} -> Sequência dos números primos!!
     
    Juntando as 3 seguências em uma única estrutura, temos:
    X = {A,B,C} -> Que é a sequência dada no exercício!!

    X = 4, 6, 2; 8, 9, 3; 10, 12, 5; 14, 15, 7; 16, 18, 11; 20, 21, 13; 22, 24, 17; 26, 27, 19; 28, 30, 23; 32, 33, 29; 34, 36, 31, 38, 39, 37, 40, 42, 41!!

    O 37 é o primeiro número da sequência C que é maior que qualquer número da sequência anterior!!
    Comprendi de vez com a ajuda do colega Guilherme!
    Conhecimento multiplicado galera!! rsrs
    Observem:
    Sequência X... Sequência {X -1} ou Sequência anterior!
    3 é menor que 4 e 6;
    5 é menor que 8 e 9;
    7 é menor que 10 e 12;
    11 é menor que 14 e 15;
    13 é menor que 16 e 18;
    17 é menor que 20 e 21;
    19 é menor que 22 e 24;
    23 é menor que 26 e 27;
    29 é menor que 28 e 30;
    31 é menor que 32 e 33; 
    37 é maior que 34 e 36!!!
    Abraços!
  • Observe que , o numero em NEGRITO  é uma sequencia de números PRIMOS , esse era o X  da questão descobriu isso é só ir fazendo a comparação da sequência de números reais com os números primos .. até chegar em que 37 > 34 .
  • Mas a sequência mostra até o ciclo 22, 24 e 17, menores que 29. Não entendo porque precisa ser um número ainda maior.
  • 22;24; 17 ; 25,26; 19 ; 27;28; 23 ; 30;32; 29 ; 33;34; 31 ; 35;36; 37   
  • Pois é, mas o enunciado não aponta o 33  e 29 pode ser entendido como maior. Fiquei sem entender.

  • Desculpem-me os colegas, mas em questões de sequência, são muito subjetivas; eu pediria anulação pelo próprio comando:

    "Na sequência a seguir, cada número pertence a apenas uma de duas categorias de números, sendo que o número 1 não pertence a nenhuma dessas categorias. Os números impressos em negrito e sublinhados são elementos de uma das categorias,"

    Não existem por definição 3 conjuntos de números, mas apenas 2! O que inutiliza qualquer das sugestões apresentadas. e com 2 conjuntos tem que forçar BASTANTE a barra para conseguir explicar a subjetividade megalomaniaca do examinador,rss.
    Se alguém conseguiu a explicação da BANCA, peço que postem, porque essa questão só faz perder nosso tempo( que já é muito escasso ).

    Bons estudos,colegas!

  • Amigos, a única forma de resolver a questão para que se encaixe no gabarito é dividindo os números em 3 grupos, algo proibido pela banca. Dividindo os grupos em dois (os números sem efeito de fonte são acrescidos nessa ordem, que se repete: 2+2+1+1 - os em negrito são primos) o primeiro número que supera os outros é 51.

  • Agora vai!
    Preenchendo as colunas, começando pelo 04, seguindo pra baixo, depois para a próxima coluna, contem 04, 06, 08, 09, 10, 12... (só não contei os primos, pois esses estão na terceira linha.
    -
    04|08|10|14|16|20|
    06|09|12|15|18|21|
    02|03|05|07|11|13|17|19|23|29|31|37|41...
    -
    04|08|10|14|16|20|22|25|27|30|33|35|
    06|09|12|15|18|21|24|26|28|32|34|36|
    02|03|05|07|11|13|17|19|23|29|31|37|41

    Realmente a coluna do 37... ele é maior que 36 e 35.

    Basta pegar desta sequência os primos, colocá-los na base, depois preencher os demais por coluna:

    -
    04,(05) 06,(07) 08,09,10,(11) 12,(13) 14,15,16,(17) 18,(19) 20,21,22,(23) 24,25,26,27,28,(29) 30,(31) 32,33,34,35,36... tomados de dois em dois, alocados por coluna!

    -
    04|08|10|14|16|20|22|25|27|30|33|35|
    06|09|12|15|18|21|24|26|28|32|34|36|
    02|03|05|07|11|13|17|19|23|29|31|37|41

  • O enunciado deu uma dica: Na sequência a seguir, cada número pertence a apenas uma de duas categorias de números, sendo que o número 1 não pertence a nenhuma dessas categorias.

    Que duas categorias são essas? Números primos e números compostos.

     

    os que estão em negrito: números primos

    os que NÃO estão em negrito: o resto dos números! É a sequência normal, só que sem os números primos. 

     

    4; 6; 2; 8; 9; 3; 10; 12; 5; 14; 15; 7; 16; 18; 11; 20; 21; 13; 22; 24; 17; 25; 26; 19; 27; 28; 23; 30; 32; 29; 33; 34; 31; 35; 36; 37

    37 - maior de todos (gabarito C)

  • gastei tempo que só, porém ainda sem entender...

  • "Cada número pertence a apenas uma de duas categorias de números, sendo que o número 1 não pertence a nenhuma dessas categorias"

    • números compostos → divisíveis por eles mesmos, por 1 e por seus múltiplos
    • números primos → divisíveis por eles mesmos, por 1 e por mais "ninguém"

    Estamos falando de números naturais, ok?

    • com divisão de resto zero e quocientes inteiros / não decimais
    • se um número não for primo, então ele será composto (possível de ser fatorado)

    Dada essa breve introdução, vamos à sequência: tá ligado o Silvio Santos, quando ele fala "PIN"?

    • então, você vai montar a sequência assim:
    • sempre dois termos compostos antecederão cada termo primo: essa é a estrutura

    • 1, ignora / 2 é primo / abrir, sempre, duas casas antes de cada número primo lançado
    • ___; ___; 2;

    • 3 é primo / abrir, então, duas casas antes dele / 4 é composto / vai entrar na 1ª casa vazia
    • 4; ___; 2; ___; ___; 3;

    • 5 é primo / abrir, então, duas casas antes dele / 6 é composto / vai entrar na próxima vazia
    • 4; 6; 2; ___; ___; 3; ___; ___; 5;

    • 7 é primo / abrir, então, duas casas antes dele / 8, 9 e 10 são compostos / preenchendo na ordem...
    • 4; 6; 2; 8; 9; 3; 10; ___; 5; ___; ___; 7;

    • 11 é primo / abrir, então, duas casas antes dele / 12 é composto / vai entrar na próxima vazia
    • 4; 6; 2; 8; 9; 3; 10; 12; 5; ___; ___; 7; ___; ___; 11;

    • 13 é primo / abrir, então, duas casas antes dele / 14, 15 e 16 são compostos / preenchendo na ordem...
    • 4; 6; 2; 8; 9; 3; 10; 12; 5; 14; 15; 7; 16; ___; 11;___; ___; 13;

    • 17 é primo / abrir, então, duas casas antes dele / 18 é composto / vai entrar na próxima vazia
    • 4; 6; 2; 8; 9; 3; 10; 12; 5; 14; 15; 7; 16; 18; 11; ___; ___; 13; ___; ___; 17;

    • 19 é primo / abrir, então, duas casas antes dele / 20, 21 e 22 são compostos / preenchendo na ordem...
    • 4; 6; 2; 8; 9; 3; 10; 12; 5; 14; 15; 7; 16; 18; 11; 20; 21; 13; 22; ___; 17; ___; ___; 19;
    • 23 é primo / abrir, então, duas casas antes dele / 24, 25, 26, 27 e 28 são compostos
    • 29 é primo / abrir, então, duas casas antes dele / vamos lá, preenchendo na ordem as casas vazias...
    • 4; 6; 2; 8; 9; 3; 10; 12; 5; 14; 15; 7; 16; 18; 11; 20; 21; 13; 22; 24; 17; 25; 26; 19; 27; 28; 23; ___; ___; 29;

    • 30 é composto / 31 é primo / já sabe: abri duas casas antes para lançar compostos
    • 32, 33, 34, 35 e 36 são compostos e, finalmente, 37 é primo

    4; 6; 2; 8; 9; 3; 10; 12; 5; 14; 15; 7; 16; 18; 11; 20; 21; 13; 22; 24; 17; 25; 26; 19; 27; 28; 23; 30; 32; 29; 33; 34; 31; 35; 36; 37 → primeiro primo a superar os termos anteriores


ID
760516
Banca
CEPERJ
Órgão
DEGASE
Ano
2012
Provas
Disciplina
Raciocínio Lógico
Assuntos

Na sequência 0, 1, 2, 4, 7, 12, x, o valor de x é:

Alternativas
Comentários
  • SACADA!---------------------->Cada terrmo é igual à soma dos dois anteriores + 1

    então: x=12+7  +1  =20

    Letra D

    até mais!
    ;)
  • O padrão é: a próximo n° é a soma dos 2 anteriores + 1:

    0+1=1
    2+4=6
    7+12=19

    Porque x é a soma dos 2 anteriores + 1,logo x=19+1=20
  • de onde saiu esse +1 meu deus
  • Respondendo à sua dúvida: há várias maneiras de resolver a questão. Deve-se, para tanto, encontrar uma lei de formação que defina a sequência apresentada.
    Primeiramente, observemos a diferença existente entre determinado número da sequência e o número imediatamente anterior. Isso ajudará a ter uma ideia da tal lei de formação.
    0,1,2,4,7,12, X -- > diferenças (1-0),(2-1),(4-2),(7-4),(12-7)
    0,1,2,4,7,12, X -- > diferenças 1,1,2,3,5
    Agora a grande sacada
     12 = 7 + 3 + 2 (número anterior mais as duas diferenças anteriores)!
      7  = 4 + 2 + 1 (número anterior mais as duas diferenças anteriores).
    Assim será para X:
    X = 12 + 5 + 3 = 20
    ALTERNATIVA D
     Somente uma observação:
    caso a questão pedisse para fornecer a lei de formação, o procedimento seria o seguinte:
    an = a n-1 + (a n-1 - a n-2) + (a n-2 - a n-3)
    an = a n-1 + (a n-1 - a n-3)
    an = 2 * a n-1  -  a n-3
     Traduzindo:
    12 = 2*7 - 2 = 14 - 2
    07 = 2*4 - 1 = 08 - 1



  • Ilustrando as explicações dos colegas:
    - por Diêgo e Gloomy:

    ou
    - por Bruno (foi como eu fiz também):
  • Ninguém merece esses tipos de questões...
  • Pior que tenho maior dificuldade com numeros ¬¬ , mas vou me empenhar em aprender >_<
  • Podiam ser só assim! Pra mim está ótimo!

    0+0=0 (+1) 1

    0+1=1 (+1) 2

    1+2=3 (+1) 4

    2+4=6 (+1) 7

    4+7=11 (+1) 12

    7+12=19 (+1) 20

    Resposta letra D

    Não sei se consegui explicar...


ID
814096
Banca
FAPERP
Órgão
TJ-PB
Ano
2012
Provas
Disciplina
Raciocínio Lógico
Assuntos

Na sequência infinita (1,3,5,7,1,3,5,7,1,3,5,7, ...) a posição 15.618 é ocupada pelo algarismo:

Alternativas
Comentários
  • (1,3,5,7,1,3,5,7,1,3,5,7, ...)
    A sequência anda sempre de 4 em 4 -> 1,3,5,7.

    Achar a posição mais próxima de 15618 -> 15616/4 =  3904 -> Ok, número inteiro, sabemos que termina em 7.
    Andar mais 2 posições para chegar em 15618, teremos -> 1, 3
    Então na posição 15618, o número será o 3.

  • Resolvido:

    https://youtu.be/xquBVFmt8xs


ID
835840
Banca
FDC
Órgão
MAPA
Ano
2010
Provas
Disciplina
Raciocínio Lógico
Assuntos

A sequência de letras apresentada abaixo obedece a certa regra lógica: B, O, E, K, H, G, K, ..., ... . Seguindo-se a sequência e mantendo-se a mesma lógica, as duas próximas letras que a completam são, respectivamente:

Alternativas
Comentários
  • Olá amigos do QC, temos que adimitir que esses caras das bancas têm uma criatividade incrível para formularem questões, essa por exemplo é uma delas. Resolvi da seguinte maneira, vejam que viagem:

     

    A   B   C   D   E   F   G   H   I   J   K   L   M                           ---> parte 1

    N   O   P   Q   R   S   T   U   V   W   X   Y   Z                       ----> parte 2

    O alfabeto está dividido em duas partes com 13 letras cada, pegamos a segunda letra de cada parte. Da parte 1 pegamos a letra B, daí em diante pegamos sempre a terceira letra posterior----> B   E   H   K   N

    Da parte 2  pegamos a letra O, daí em diante pegamos sempre a quarta letra anterior-----> O     K     G    C

    O k está de verde porque serviu para as duas partes.
    Grande abraço e fé em Deus.




  • Realmente a criatividade foi longe aí...
  • Sempre que um problema trouxer Letras em sequência é interessante analisarmos as posições que estas se encontram no alfabeto.
    Veja como torna fácil o problema:
    B   O   E   K   H   G   K   __     ___
    2   15   5  11   8   7   11    3       14
    Letras Azuis = Subindo de 3 em 3 posições
    Letras Pretas = Descendo de 4 em 4 posições
    Logo, as próximas letras serão a de posição #3 ( C ) e #14 ( N )

ID
853993
Banca
FCC
Órgão
TJ-PE
Ano
2007
Provas
Disciplina
Raciocínio Lógico
Assuntos

Assinale a alternativa que substitui corretamente a interrogação na seguinte seqüência numérica: 8 12 24 60 ?

Alternativas
Comentários
  •  de 8 a 12 = 4
    de 12 a 24 = 12
    de 24 a 60 = 36

     sempre o triplo
    3x4=12
    3x12=36
    3x36=108

    108+60= 168
  • Alternativa E. 

    É a razão da lógica anterior vezes 3. 
    8 para 12 - razão 4
    12 para 24 - razão 12 (4 vezes 3)
    24 para 60 - razão 36 (12 vezes 3)
    60 para x - razão 108 (36 vezes 3)

    x = 168. 
  • Resolvi de forma mais prática e rápida.

    DIMINUINDO O NÚMERO POR 4 e MULTIPLICANDO POR 3, o resultado sempre será o número seguinte.

    (Número - 4) x 3 = Número seguinte.

    Na sequência 8 12 24 60 X

    (8 - 4) x 3 = 4 x 3 = 12

    (12 - 4) x 3  = 8 x 3 = 24

    (24 - 4) x 3 = 20 x 3 = 60

    (60 - 4) x 3 = 56 x 3 = 168

    Achei essa solução simples e prática.
  • Nessa lógica fica subentendido a multiplicação por 3 da distância entre os números.


    8 e 12 = 4 x 3 = 12

    12 e 24 = 12 x 3 = 36

    24 e 60 = 36 x 3 = 108

    como último número é 60, soma-se com 108

    168.

  •              1ª                               x3                              x3                             x3
                                                    
                 2ª                    4                              12      =========>                    36    + 24 aí de baixo(3º termo)= 60 e assim vai..         
                                                       
                                                         
                3ª          8( 1ºtermo )        12    (2º termo)                    24 (3º termo)                60 (4º termo)         





     pega o primeiro(3x4=12,logo 12+12 (esse é o 2º termo) = 24      3x12=36,  logo 36 +24=60 .   para descobrir qual numero virá depois do 60 só é fazer a mesma coisa.    multiplica o numero da 1ª coluna pelo da 2ª   =  3 x 36= 108 e soma ao numero 60 = 168.
  • 8 12 24 60 168
    +4 +12 +36 +108  
    +4.1 +4.3 +4.9 +4.27  
    +4.(3)^0 +4.(3)^1 +4.(3)^2 +4.(3)^3  

    Os valores da sequência na primeira linha. Na segunda os valores que foram adicionados para que se chegasse ao proximo termo da sequência.

    Abaixo os valores que foram adicionados em outra forma. Quando se chega a (4,3) percebe-se que (4,4) só pode ser +4.(3)^3 daí faz-se o caminho inverso.
     
  • multiplicando por números ímpares crescentes a partir de 3 os termos intercalados:

    8 x 3 = 24

    12 x 5 = 60

    24 x 7 = 168

    60 x 9 = 540

    e vai...

  •  4(vem antes)8.........12........24...........60,....?......

    3 x (4).......5x(12).....7x(60)....

    =12........60......160.

    SÓ MULTIPLICA O 4,12 E 60,OU SEJA,PULA UMA CASA DE CADA. O mesmo são para os numeros que irão multiplicar 1,2,3,4,5,6,7( pula uma casa de cada / 3,5 e7/ e multiplica por 4,12 e 60) 3x4/5x12 e 7x60.

    Há várias formas de fazer esse problema..

  • A FORMA QUE CONSEGUIR RESOLVER FOI SUPER PRÁTICO 

    8, 12, 24, 60 ?       "8 E 24"   "'12 E 60"   '"24 E 168"

    8 X 3 =  24

    12 x 5 =60

    24 x 7 = 168 

    OBS: OS NÚMEROS FORAM MULTIPLICADOS ALTERNADAMENTE E A DIFERENÇA DOS VALORES A SER MULTIPLICADOS FOI DE 2, OU SEJA, DE 3 PULOU P 5 DE 5 PULOU PARA 7

    HILDIANY KELLY 


ID
853996
Banca
FCC
Órgão
TJ-PE
Ano
2007
Provas
Disciplina
Raciocínio Lógico
Assuntos

Assinale a alternativa que completa a série seguinte:
J J A S O N D ?

Alternativas
Comentários
  • Alternativa A. 
    Quando houver sequências de letras que parecem absurdas, devemos pensar que elas são as iniciais de alguma sequência comum do cotidiano: dias da semana, alfabeto, etc. Nesse caso, a sequência se refere a meses do ano: junho, julho, agosto, setembro, outubro, novembro, dezembro. A resposta, portanto, é a alternativa A, pois o mês seguinte seria janeiro. 
  • J J A S O N D X

    JUNHO JULHO AGOSTO SETEMBRO OUTUBRO NOVEMBRO DEZEMBRO JANEIRO

    Gabarito: letra A
  • nunca imaginaria que as letras se refeririam às iniciais dos meses, obrigado pela ajuda!
  • Questão difícil,poderia dar mais dica no enunciado.
    Tentar descobrir o que o examinador quer não é nada fácil. 
  • Estava muito óbvio e não consegui ver!

  • Caramba...viajei

    tinha pensando assim: C C  V C V  CC ? V ( consoante, consoante, vogal, consoante, vogal, consoante, consoante então a próxima seria vogal e como nas alternativas so tinha uma vogal eu jurava que tinha acertado rsrs.

    Obg Nina pela explicação

  • Observando com atenção, percebe-se que são as letras iniciais dos meses do ano. Assim:

    Junho, Julho, Agosto, Setembro, Outubro, Novembro, Dezembro e.... Janeiro.


    Logo a alternativa correta é a letra A.


  • Esse é o tipo de questão que a gente faz o sinal da cruz e... chuta!!! rs...

  • Resolvo essa e outras questões similares aqui nesse vídeo

    https://youtu.be/_59ETIxrzXE

    Ou procure por "Professor em Casa - Felipe Cardoso" no YouTube =D

  • Meses do ano

    J - Junho, J - julho, A - agosto, S - setembro, O - outubro, N - novembro, D - dezembro,...

    O próximo é janeiro

    J


ID
866689
Banca
ESPP
Órgão
BANPARÁ
Ano
2012
Provas
Disciplina
Raciocínio Lógico
Assuntos

Verificando a sequência 3, 2, 3, 8, 3, 48, 3 ,384, 3, ..., o valor do próximo termo é:

Alternativas
Comentários
  •  Olá amigos do QC, essa questão notei o seguinte desenvolvimento:
    Repare que entre os 3, esta acontecendo o seguinte processo:
    2 . 4 = 8
    8 . 6 = 48
    48 . 8 = 384
    Então só nos resta dizer que:
    384 . 10 = 3840 que é o gabarito.
    A sequência de vermelho está aumentando de 2 em 2 e múltiplicando. Fique atento nesse tipo de questão, se a sequência estiver diminuindo trabalhe com subtração ou divisão e se a sequência estiver aumentando trabalhe com soma ou multiplicação, podendo também utilizar potência, o que é mais difícil ocorrer.

    Força nos estudos, não desanime e fé em Deus.
  • Para resolver a questão foi necessário pensar na relação entre os números, 2, 8, 48 e 384. Além de estarem aumentando, esse aumento foi não somente do resultado, como também da razão entre um e outro resultado. De 2 para 8, aumentou-se  o resultado. De 8 para 48 aumentou-se também a razão (de 4 para 6) e assim por diante.
     
    Bons estudos!    
  • Acho que a grande "sacada" era perceber que o algarismo 3 está apenas intercalado na sequência, e, a partir daí fica mais fácil.

    Bons Estudos!
  • fórmula:

    para todo número em uma posição ímpar, seu valor é automaticamente 3.

    para todo número em uma posição par, seu valor é automaticamente o número na posição par multiplicado pela posição do número em questão.


    para todo n-ímpar = 3

    para todo n-par = valor do n-2 x n

  • perfeito o comentario do batalhador!

  • GABARITO: C

  • Resolvido:

    https://youtu.be/KT8Hp8CVRW0


ID
867160
Banca
ESPP
Órgão
BANPARÁ
Ano
2012
Provas
Disciplina
Raciocínio Lógico
Assuntos

Verificando a sequência 8, 10 , 11, 14, 14, 18, 17, 22, ..., o valor do próximo termo é:

Alternativas
Comentários
  • Percebam que há um intercalamento de operações:
    -uma começando a partir de 2 e sempre aumentando de 1 em 1;
    -outra começando a partir de 1 e sempre diminuindo de 1 em 1.


    8 (+2) 10 (+1) 11 (+3) 14 (+0) 14 (+4) 18 (-1) 17 (+5) 22, logo, a próxima operação é subtrair 2:
    20

  • Percebam que há um intercalamento de operações:
    -uma começando a partir de 2 e sempre aumentando +1
    -outra começando a partir de 1 e sempre diminuindo -1


    8 (+2) 10 (+1) 11 (+3) 14 (+0) 14 (+4) 18 (-1) 17 (+5) 22, logo, a próxima operação é subtrair 2:
    20
  • Eu resolvi verificando que ao primeiro número é  adicionado 3, assim como ao terceiro e quinto números.
    Ao segundo é adicionado 4, assim como ao quarto e ao sexto.
    Logo, ao sétimo número devemos adicionar 3, de acordo com essa ordem!

    Bons estudos!
           
  • Olá amigos do QC , observando atentamente a numeração verificamos que os números que estão nas posições ímpares( 8, 11, 14, 17) a sequência aumenta de 3 em 3. Já os que estão nas posições pares(10, 14, 18, 22) a sequência aumenta de 4 em 4.

    Grande abraço.
  •  Olá amigos do QC, observando atentamente a numeração vemos que os números estão nas posições ímpares(8, 11, 14, 17) a sequência aumenta de 3 em 3. Já os números que estão nas posições pares(10, 14, 18, 22) a sequência aumenta de 4 em 4.

    Grande abraço.
  • Eu tentei várias sequências e nada... quando tava saindo fumancinha da minha caixola...eu pensei vou de dois em dois ( em pares) pra ver e ai fikou assim:
    (8,10) de 8 pra 10 tenho: (intervalo de + dois números ou seja: +2)
    (11,14) de 11 pra 14 tenho: (intervalo de + três números ou seja: +3)
    (14,18) de 14 pra 18 tenho: (intervalo de + quatro números ou seja: +4)
    (17,22) de 17 pra 22 tenho: (intervalo de + cinco números ou seja: +5)
    Ai eu pensei a próxima sequencia tem que ser com + 6, ou seja o total dos intervalos o resultado teria que dar +6 (Obedecendo a sequencia:+2,+3,+4,+5,+6...)  mas essa informação não me dava o próximo número entre as alternativas oferecida pela questão.
    Começar tudo de novo... vou continuar com pares, olhei novamente para a sequencia oferecida  e percebi a proximidade do 10,11   14,14,  18, 17. Vou fazer eles ficares pares pra ver. Então ficou assim:

    (10,11) de 10 pra 11 tenho: (intervalo de + um número ou seja: +1)
    (14,14) de 14 pra 14 tenho: (intervalo ? Eita, não tem intervalo e agora? Se não tem nada, nada pra mim é zero.ou seja: 0)
    (18,17) de 18 pra 17 tenho: (intervalo de  um número. Mas não era pra somar mais um intervalo, era pra tirar um intervalo ou seja: -1)
    Como o próximo número que eu tenho na sequencia pra analisar é o 22 e o intervalo de -2, cabe na sequencia que eu encontrei ( +1, 0 , -1...) fiz a seguinte continha: 22 - 2 = 20.     /o/Viva!!!! Achei 20. Tem 20? tem na letra E... deu certo. pensa na felicidade!

    É persistir e praticar, acreditar que você também é capaz. E vamos estudar. Um abraço pra quem ta estudando esse conteúdo as 4 da madrugada...rsrsrsrsrsr.

  • Foi o que percebi num primeiro momento. se alguém visualizar algo mais fácil...
  • Letra E - 20
    Também pode ser:
    Sequência dos números nas posições 1, 3, 5, 7:
    8 +3 > 11 +3 > 14 +3 > 17  portanto o próximo será  17  +3 >  20

    Sequência dos números nas posições 2, 4, 6, 8:
    10 +4 > 14 +4 > 18 +4 > 22 portanto o próximo será  22 +4 > 26
  • 8 - 10 ---+2  *
    10 - 11 ----+1 ~
    11- 14-------+3 *
    14 - 14 ------+0 ~ 
    14 - 18------+4 *
    18 - 17-------- -1~
    17 - 22 ------- + 5 *
    22 - 20-------  - 2~
    20 - 26 --------- +6 *
    26 - 23 ---------  - 3~
    23 - 30 ---------- +7*
    30 - 26 ---------  -3~
    ...
  • 8 10 11 14 14 18 17 22

    (8 10 11 14)  (14 18 17 22)

    do 8 até o 14 são 6 números
    do 10 até o 18 são 8 números

    do 11 até o 17 são 6 números
    do 14 até o 22 são 8 números

    Pela sequência, a partir do 14, devemos acrescentar mais 6 números = 20 (letra e)
  •  
      8 + 3 = 11
    10 + 4 = 14
    11 + 3 = 14
    14 + 4 = 18
    14 + 3 = 17
    18 + 4 = 22 
    17 + 3 = 20

    Resposta letra "e" (20)
    Esta foi minha linha de raciocínio.
  • Fiz o seguinte:
    A soma dos 2 numeros até o final.
     
    Ex: 8+10 = 18, 10+11 = 21, 11+14 = 25, 14+14=28, 14+18=32, 18+17=35, 17+22=39

    Aí calculei a diferenca entre cada um desses termos.
    21-18=3
    25-21=4
    28-25=3
    32-28-4
    35-32=3
    39-35=4

    Logo, o próximo termo precisará ter diferenca de 3 do numero 39 , ou seja, será o 42. Que é 22 + 20.

    Não sei se era essa a linha, mas consegui assim.
  • Observando a sequência com atenção, chegamos ao raciocínio como na figura abaixo:

                                                             


    Letra E.


  • É possível resolver de dois modos diferentes.

    https://www.youtube.com/watch?v=4y2Q-Xga4Zg


ID
879997
Banca
IESES
Órgão
PM-SC
Ano
2011
Provas
Disciplina
Raciocínio Lógico
Assuntos

Com base na seguinte progressão geométrica: {2; 4; 8; 16; 32; 64;... } o próximo valor da sequência seria:

Alternativas
Comentários
  • 4/2=8/4=16/8...=r=2

    P.G de razão 2, Logo, 64*2=128 Letra "B"

    até mais!
    ;)
  • POHA NESSA AI NÃO TEM CALCULO É A LÓGICA O DOBRO DE 2 >4 O DOBRO DE 4 >8 O DOBRO DE 8>16 O DOBRO DE 16>32 O DOBRO DE 32>64
    E O DOBRO DE 64>128 OK
  • Nem precisa calculo, basta olhar para os numeros que a resposta "surge"
  • CONCURSEIRO GUERREIRO FAZ ESSA QUESTÃO COM UM OLHO SÓ E A AINDA PISCANDO.

ID
898981
Banca
FCC
Órgão
TRT - 9ª REGIÃO (PR)
Ano
2013
Provas
Disciplina
Raciocínio Lógico
Assuntos

Uma senha formada por três letras distintas de nosso alfabeto possui exatamente duas letras em comum com cada uma das seguintes palavras: ARI, RIO e RUA. Em nenhum dos três casos, porém, uma das letras em comum ocupa a mesma posição na palavra e na senha. A primeira letra dessa senha é

Alternativas
Comentários
  • Bom, temos que ter em mente as REGRAS impostas pela questão!
    1. Senha formada por 3 letras distintas;
    2. Senha formada por EXATAMENTE 2 letras em comum das palavras ARI, RIO e RUA;
    3. Nenhuma das letras ocupa a mesma posição daquela ocupada nas palavras.
     
    Vamos botar o ?cocuruto? para trabalhar!
    => a letra R aparece nas 3 palavras. Será que ela faz parte da senha? Se pensarmos que a letra R não fará parte da senha, teríamos as letras A e I (da palavra ARI), I e O (da palavra RIO) e U e A (da palavra RUA) como letras da senhas. São 4 letras e a questão fala em apenas 3 (regra 1). Então, obrigatoriamente, a letra R fará parte da senha! E a posição dela deve ser __ __ R (regra 3).
    => Deduzindo isso, também poderemos concluir que as letras A e I NÃO PODEM ESTAR JUNTAS na senha, ok? Olhem para a palavra ARI. Como só podemos ter na 2 letras de cada palavra (regra 2), ou teremos A ou teremos E, correto? Assim:
    => letra A na senha => teríamos ARI, RIO e RUA => Já sabemos a posição da letra R. A letra A já ocupou, nas palavras, a 1a e a 3aposição. Só sobrou a 2a posição. A letra O só não pode ser a 3a posição. A senha ficou assim:
    O A R
     
    => letra I na senha => teríamos ARIRIO e RUA => Já sabemos a posição da letra R. A letra I já ocupou, nas palavras, a 1a e a 2a posição. Só sobrou a 3a posição.
    Epa, PH! Aí já tem a letra R!!!
    Perfeito, companheiro! Por este motivo, a letra I  NÃO fará parte da senha!
     
    Assim, a única senha possível é O A R. Desse modo, a 1a letra será a letra O.
     
    Gabarito: letra D
     
    Fonte: EuVouPassar
  • Acredito que na parte que o colega acima citou que a letra I ja tenha ocupado a primeira e a segunda posição nas palavras, ele tenha se equivocado, pois nas palavras ARI e RIO onde ela aparece, ocupa a terceira e segunda posição respectivamente.
     Dito isto, conforme já muito bem explicado pelo colega, a terceira posição já está ocupada pela letra R, e a segunda pela letra A, sobrando então as letras I e O para ocupar a  primeira posição .
    Como a regra diz que somente duas letras podem ser comuns às palavras, a letra I não pode ser a escolhida, senão repetiria as 3 letras da palavra ARI, logo a letra correta e necessária para compor a senha, é a letra O.
    Resposta correta letra D.
    Não sei se fui muito clara, mas consegui enxergar dessa maneira.
  • LETRA D - "O"
    1. Senha formada por três letras distintas
    2. Possui exatamente duas letras em comum com cada uma das seguintes palavras: ARI, RIO e RUA.
    3. Em nenhum dos três casos uma das letras em comum ocupa a mesma posição na palavra e na senha.

    Logo as senhas poderão ser composta pelas letras: R - A - O, cuja ordenação será: 
    R - Não pode aparecer na 1ª e nem na 2ª posição, portanto:       R
    A - Não pode aparecer na 1ª e nem na última posição, portanto: 
       A R
    O - Não pode aparecer na última posição, portanto: O A R

    Portanto a primeira letra dessa senha é: O


    Poderia ser a combinação de R - I - U, porém não há um arranjo entre elas que satisfaça a condição 3.
  • Não consigo  entender! Será que alguém pode me explicar.
    Por que a senha não pode ser IAU ou IOR, não consigo entender o porquê que a letra R tem que ser a última.
    Por favor, alguém me ajude!!!!!!
  • Informações da questão:
    Senha com 3 letras distintas ___ ___ ___
    Senha precisa conter necessariamente duas letras em comum com cada uma das palavras ARI RIO RUA
    Em nenhum dos casos a letra ocupa a mesma posição na palavra e na senha

    Respondendo a colega Marcela: 
    A letra R já ocupou as posições 1 e 2, logo ela só poderá estar na posição 3, pois em nenhum dos casos a letra ocupa a mesma posição na palavra e na senha
    ARI - R na 2 posição
    RIO e RUA - R na 1 posição

    Porque não pode ser IAU? Porque a senha tem que ter duas letras em comum com cada uma das 3 palavras dadas na questão e não em comum com apenas uma delas. Nesse caso IAU só tem uma letra (I) em comum com a palavra RIO.
    O mesmo ocorre com IOR, que só tem uma letra em comum com a palavra RUA.
    Portanto a senha tem que ser:
    R na ultima posição
    e as outras letras tem que combinar de forma que siga as orientações da questão
    Logo, só será possível OAR
    Espero ter ajudado!


  • Obrigada Maria Cecilia. Somente com sua explicação, eu entendi.
  • Perdõem minha insegurança, mas poderia ser IOR. obrigado
  • Perdõem minha afobação. entendi.
  • * SENHA __ __ __
     
    * Palavras:
    ARI
    RIO
    RUA
     
    1º PASSO: começar pelo final da enunciado que diz:
    Em nenhum dos casos a letra ocupa a mesma posição na palavra e na senha, ou seja:
    A letra “A” apareceu na e na posição. Então só poderá estar na 2ª
    A letra “R” apareceu na 2ª e na 1ª posição. Então só poderá estar na 3ª
    A letra “I” apareceu na 3ª e na 2ª posição. Então só poderá estar na 1ª
    A letra “O” só apareceu na 3ª posição. Então poderá estar na 1ª e na 2ª
    A letra “U” só apareceu na 2ª posição. Então poderá estar na 1ª e na 3ª
    Visualmente, poderão estar na seguinte ordem:
     
    __   __   __
      I      A     R
     O     O
     U            U
     
    2º PASSO: seguir a regra do enunciado que diz:
    Possui exatamente duas letras em comum com cada uma das palavras
     
    IAR
    ARI (não pode, porque tem 3 em comum)
    RIO
    RUA
     
    IOR
    ARI
    RIO (não pode, porque tem 3 em comum)
    RUA (não pode, porque tem 1 em comum)
     
    IAU
    ARI
    RIO (não pode, porque tem 1 em comum)
    RUA
     
    IOU
    ARI (não pode, porque tem 1 em comum)
    RIO
    RUA (não pode, porque não tem nenhuma letra em comum)
     
    OAR
    ARI
    RIO
    RUA (conseguimos!!! Cada uma das palavras tem duas letras em comum)

    Obs.: cheguei a esse raciocínio com ajuda dos colegas que comentaram antes! Obrigada a todos que fazem do site QC um ótimo ambiente de estudo!
  • errei essa questão, mas o pessoal caprichou tanto na explicação, que eu entendi demais!!!

    para decidir: "possui exatamente duas letras em comum"

    A R I         -> duas já coincidem - >  I tá fora

    R I O        -> sobra a letra O      - > 
    I tá fora

    R U A       -> duas já coincidem  - > U tá fora

    o R só pode ocupar a 3ª posição
    o
    A só pode ocupar a 2ª posição

    _____      A       R    


    Bons Estudos!
  • Pessoal, é triste mas eu ainda estou com dúvida! :(
    Embora todos os comentários tenham sido brilhantes! O problema está em mim mesmo...
    Eu entendi que não se pode reperir a posição das letras, mas não entendi porque as letras O A R que necessariamente compõem a senha. Porque o I e o U foram descartados? Como chegaram a essa conclusão?
  • Prezada colega Juliana

    detalhes importantes da questão:  A senha de 3 letras
    - deve ter 2 letras iguais a de cada palavra dada
    - não deve repetir a posição usada nas palavras
    por esses motivos a única possibilidade de senha é necessariamente OAR
    Analisando ARI RIO e RUA
    observa-se: R já ocupou a 2ª (ARI) e 1ª(RIO e RUA)  posição, logo só poderá ocupar a 3ª posição
    A letra A já ocupou a 1ª (ARI) e a 3ª(RUA), logo só poderá estar na 2ª posição
    Já a letra O como ocupou apenas a 3ª posição (RIO) poderia ficar na 1ª e 2ª posições, porém devido a posição das letras anteriores só restou vago a 1ª posição.
    Dessa forma a senha formada obedece aos requisitos da questão 
    OAR tem exatamente duas letras iguais a cada uma das palavras dadas
    ARI (A e R )
    RIO (R e O)
    RUA (R e A)
    e não ocupa a mesma posição usada nas palavras.

    As letras I e U foram descartadas porque não é possível formar uma sequência utilizando essas letras com as exigências da questão.
    lembre-se: a senha deve conter 2 letras iguais em cada uma das palavras dada e não deve repetir a mesma posição usada nas palavras dadas inicialmente.

    usando o I:
    já foi usado na 3ª posição  (ARI) e na 2ª posição (RIO), logo só poderá ser usado na 1ª posição
    I ___ ___
    utilizando a letra I teremos que escolher outras letras de forma a fazer com que tenha 2 letras iguais as das palavras
    se escolher a letra U, está já foi usada na segunda posição, logo só poderá ser usada na 1ª e 3ª posição, e como a 1ª está com a letra I só poderá ser usado na 3ª posição.
    I __ U
    agora teremos que colocar uma letra no meio
    exemplo: A. pode ficar no meio 
    IAU, porém a senha não contém 2 letras iguais a cada uma das palavras.
    e assim vai, por tentativa e erro nunca será possível utilizar o I e U, porque não fazem a sequência exigida pela questão. 

    é meio complicado mas espero ter ajudado.
  •  primeiro vamos tentar sem a letra "a"
     
    *ri
    ri*
    ru*
     
    i*r - impossível sem a letra "a", pois sobraria somente a primeira posição para as letras "u" e "i".
     
     
    sem a letra i:
     
    ar*
    r*o
    r*a
     
    oar - possível sem a letra i
    primeira letra: o
     
    sem a letra "r":
    a*i
    *io
    *ua
     
    impossível sem a letra "r", pois sobram quatro letras e a senha só tem três
     
    sem a letra "u":
     
    ar*
    r*o
    r*a
     
    oar - possível sem a letra "u"
    primeira letra: "o"
     
    nos dois casos possíveis a letra "o" é a primeira.
  • sinceramente não consegui entender

  • É bem certo que os matemáticos puristas de plantão me critiquem, mas é inegável também a importância das técnicas de resolução de prova. Em vista disso, proponho a seguinte resolução para os que sentiram dificuldade. 

    Observem as respostas das alternativas. Agora, vejam o que o enunciado diz: "palavras: ARI, RIO e RUA. Em nenhum dos três casos, porém, uma das letras em comum ocupa a mesma posição na palavra e na senha."
    Assim, 
    "A" já ocupou a 1ª e a 3ª posições (das palavras ARI e RUA); sobrou-lhe a 2ª posição
    "R" já ocupou a 2ª e a 1ª posições (das palavras ARI, RIO e RUA); restando-lhe a 3ª posição.
    Nesse panorama, ficou assim: Senha: ___, A, R. 
    Logo, de plano, já se descartam as alternativas: "b", "c" e "e".
    Ficamos com as alternativas "a" e "d", ou seja, a primeira letra da senha só pode ser "I" ou "O" (Aqui já se teria 50% de chance no chute, em menos 3 minutos de leitura atenta). 
    Sobram portanto as seguintes possibilidades: IAR ou OAR. 
    Mas percebam que IAR não pode pois viola o seguinte trecho do enunciado: "possui exatamente duas letras em comum". IAR naturalmente tem três letras em comum com ARI. 
    Logo, a resposta só pode ser OAR. Letra "d". 
    Bons Estudos. Sigamos à frente!

  • QUE SE DANE OS MATEMÁTICOS DE PLANTÃO !!!! bizu é acertar e com a menor enrolação !!! 


    perfeito o comentário de "ed mat"

  • Na minha visão a forma que esse o enunciado foi escrito ficou ambíguo, porém vamos focar na Solução:O que o problema realmente quer?
    A primeira letra da senha, sabendo que:- são formadas por 3 letras distintas, ou seja a senha não possui letras repetidas dentro da Senha;- Cada uma das palavras tem exatamente duas letras que compõem a senha e não está na mesma posição que na palavra.
    Como temos as palavras ARI,  RIO e RUA Sabemos que temos as seguintes letras para formar a senha A R U O I.
    Letra A está em duas palavras, uma no inicio (ARI) e na outra palavra está no fim (RUA), Então na senha só pode estar no meio(  _ A _  )
    Letra R só pode ser utilizada na ultima posição, pois ela encontra-se nas no meio da palavra ARI e no começo da palavra RIO e RUA ( _ A R), Até o momento temos:
    ARI (foram utilizadas 2 Letras para senha A R)
    RUA ( Foram utilizadas duas letras para a Senha R e A)
    RIO ( Até o momento Somente a Letra R foi utilizada, então a primeira é O ou I)

    Vamos testar a Letra I, o I não pode ser, pois se utilizar ele estaria utilizando a terceira letra da palavra "ARI", então a só resta a opção "O", assim utiliza somente duas letras de cada palavra.

    Resposta: Opção D (Letra O)

  • Parabéns ao pessoal que fez comentários explicativos . Ajudou-me sobremaneiramente para entendimento e resolução.  Muito obrigada! Principalmente ao ED Mat.

  • O ultimo que sair apague a luz

  • Parabéns aos comentários, mas a questão diz que são três letras distintas de nosso alfabeto e que possui exatamente duas letras em comum com cada uma das palavras...... ou seja ..... em nosso alfabeto existem 26 letras......  a questão deixa claro que só usariamos duas letras das palavras formadas. Logo a alternativa (e) estaria correta tbm por trazer a letra "L".  Deveriam ter anulado.

  • Como quer saber "primeira letra da senha", vamos definir primeiro a 2ª e a 3ª."Em nenhum dos três casos, porém, uma das letras em comum ocupa a mesma posição na palavra e na senha." ARI, RIO e RUA

    o A já ocupou o 1º e o 3º lugares, então A no 2º.

    o R já ocupou o 1º e o 2º lugares, então o R no 3º

    "Uma senha formada por três letras distintas de nosso alfabeto": já podemos excluir as alternativas b e c. "Possui exatamente duas letras em comum com cada uma das palavras". Portanto, usando A e R, em ARI e RUA, já utilizamos 2 letras. Sobra apenas a palavra RIO. Como já utilizamos a R, sobram a I e a O. Não podemos usar a I, pois haverá 3 letras em comum com a palavra ARI, portanto: "A primeira letra dessa senha é" O. Resposta: Letra (D)

  • Entendi nada... Mas é um tipo de questão que só caiu uma vez

  • Questão pra não perder tempo...melhor chutar e seguir em frente!


ID
910168
Banca
FCC
Órgão
DPE-SP
Ano
2013
Provas
Disciplina
Raciocínio Lógico
Assuntos

Considere um sistema de referência com apenas duas posições que serão chamadas de vertical (V) e horizontal (H). Imagine um trem de brinquedo composto por apenas quatro vagões. Esses vagões estão ordenados e ocupam apenas as posições V e H. Uma primeira informação sobre as posições dos vagões mostra o primeiro vagão na posição V, o segundo na posição H, o terceiro na posição V e o quarto na posição V, formando a sequência: V, H, V, V para os vagões. Serão fornecidas outras cinco novas informações sobre as posições dos vagões em forma de sequência das letras V e H.

2a informação: V, V, H, V;
3a informação: H, V, V, H;
4a informação: V, H, V, V;
5a informação: V, V, H, V;
6a informação: H, V, V, H;

Supondo que as informações sobre as posições continuem seguindo o mesmo padrão, a sequência que identifica a 10a informação é

Alternativas
Comentários
  • Tendo em vista que:

    1a informação é: V, H, V, V
    2a informação é: V, V, H, V
    3a informação é: H, V, V, H

    As demais informações são repetições das informações anteriores, seguindo o seguinte padrão:


    4a informação = 1a informação
    5a informação = 2a informação
    6a informação = 3a informação

    Logo, as demais informações serão:

    7a informação = 4a informação
    8a informação = 5a informação
    9a informação = 6a informação

    E a nossa 10a informação só poderia ser igual à 7a informação que é igual à 4a e a 1a informações.

    Resposta certa: LETRA D (V, H, V, V)

    Bons estudos para todos, companheiros!
  • Pessoal, a maneira que encontrei para resolver:

    As 2 letras "V V" não se separam e caminham para o lado direito

    A letra "H" separa a dupla "V V" (como se fosse uma barreira) para não existir a sequência de 3 V "V V V"

    Seguindo esta sequência lógica até a 10ª informação, chegamos na opção "d".


    espero ter ajudado.

  • 1ª informação

    V

    H

    V

    V

    2ª informação

    V

    V

    H

    V

    3ª informação

    H

    V

    V

    H

    4ª informação

    V

    H

    V

    V

    5ª informação

    V

    V

    H

    V

    6ª informação

    H

    V

    V

    H

    7ª informação

    V

    H

    V

    V

    8ª informação

    V

    V

    H

    V

    9ª informação

    H

    V

    V

    H

    10ª informação

    V

    H

    V

    V

    Atente que a letra V mostra um padrão apresentando-se sempre junta com outra igual (VV) e estas estão sempre intercaladas pela letra H.

    Bons estudos a todos.

  • GABARITO: D

    Eu resolvi assim: montei a sequência e reparei na repetição das letras, veja:

    1a:  V H V V

    2a.: V V H V

    3a:  H V V H

    4a:  V H V V

    5a:  V V H V

    6a:  H V V H

    7a:  V H V V

    8a:  V V H V

    9a:  H V H V

    10a: V H V V 

    Perceba que as sequências que se repetem na vertical são:

    V V H ; H V V ; V H V ; V V H. Por isso a resposta da 10a.sequência: V H V V 


    É isso. E vamos que vamos!! :)

  • Olá pessoal, eu fiz da seguinte forma: o padrão se repete de 3 em 3, e a questão pede a décima sequência, então eu dividi 10/3, resultado 3 e resto um. Então peguei a sequência 3 e somei mais uma, resultando na primeira sequência, letra D. 

  • Segui a seguinte lógica começando da segunda formação: 2 V ( VV) na frente:  2a informação: V, V, H, V;  , 2 V no meio: 3a informação: H, V, V, H; , 2 V no final:  4a informação: V, H, V, V;   e repete até o 10º.


  • demorei 10 minutos pra identificar a lógica kkk

  • Só procurar a repetição do padrão...


ID
910171
Banca
FCC
Órgão
DPE-SP
Ano
2013
Provas
Disciplina
Raciocínio Lógico
Assuntos

A sequência chamada a partir de agora de DS (dobro da soma) é: 1; 1; 4; 10; 28; 76; .... Os dois primeiros termos da sequência DS são o número 1 e os termos seguintes são criados com a regra: dobro da soma dos dois termos imediatamente anteriores. Assim, o terceiro termo é 4 pois 4 é o dobro da soma entre 1 e 1. O quarto termo é 10 porque 10 é o dobro da soma entre 4 e 1. E a sequência segue dessa maneira ilimitadamente. Sabendo que o 8o termo de DS é 568 e o 10o termo de DS é 4240, o 9o termo dessa sequência é

Alternativas
Comentários
  • Alternativa "B"

    1º      2º      3º       4º      5º       6º       7º          8º          9º         10º
    1;      1;       4;       10;     28;      76;      ?;          568;        ?;         4240

    7º = (28 + 76) x 2 = 208

    9º = (208 + 568) x 2 = 1552
  • Eu resolvi o problema a partir das alternativas apresentadas na própria questão. De acordo com o enunciado: “Sabendo que o 8o termo de DS é 568 e o 10termo de DS é 4240, o 9termo dessa sequência é:” 

    Testei até encontrar o resultado. Dessa forma, analisei os DS:  8º          9º         10º.

    EX:

    a) 2120 + 568 = 2688
    2688 * 2 = 5376
     

    b) 1552 (9º termo) + 568 (8º termo) = 2120
    2120 * 2 = 4240 (10º termo)


    Gabarito = 1552

    Trata-se de uma forma de resolução a partir de testes com os resultados apresentados na questão.
  • Pegando os dados da questão para exemplificar.

    10/2= 5       5= 1+4    Se, por exemplo, quisessemos saber o termo anterior temos que diminuir. 10/2= 5     5-4=1 ou 5-1=4. O número que você vai diminui dependerá de qual termo quer conseguir. 

    No pedido da questão: 

    4240/2= 2120     2120 - 568= 1552   
  • Sabendo que o dobro da soma do 8º e do 9º termos temos como resultado o 10º termo, então:

    2 * (8º termo + 9º termo) = 10º termo 
    2 * (568 + 9º termo) = 4240
    568 + 9º termo = 4240 / 2
    9º termo = 2120 - 568
    9º termo = 1552 (Letra B)
  • Essa questão não é difícil. Agora tem que pegar o X da questão que é:

    O número X/2 menos 2 números antes de X é igual ao número antes de X:

    EX: O quarto número (10) dividido por 2 = 5, menos o segundo número (1) é igual ao 3 = 4.

    Assim o décimo número dividido por 2 (4240/2) = 2120, menos o oitavo número (568) é igual ao nono número.

    4240/2 - 568 = 1552
  • Para responder a questão vamos considerar o seguinte:
    a8 = 568; a10= 4240;
    e a questão nos pede a9.

    Portanto,
    a10 =(a8 + a9)*2

    Agora é só substituir:
    4240 = (568 + a9)*2

    E calcular:
    4240/2 = 568 +a9
    2120 = 568 +a9
    2120 - 568 = a9
    a9 = 1552.

    Sendo assim a alternativa "b" a correta.
  • fiz com a ideia de eduardo, sendo que mutipliquei o 8º numero e diminui do 10º, e dividi por 2 o resultado.. {4240 - (568 x 2)  } /2

  • GABARITO: B

    Temos aqui a chamada equação do segundo grau. Podemos responder assim:

    2 . (568 + x) = 4240

    568 + x = 4240 / 2

    x = 2.120 - 568

    x = 1552

  • Eu resolvi da seguinte forma: chamei a sequência de o contrário da soma (CS).

    Portanto, CS = (10°)/2 - 8°

                   CS = (4240)/2 - 568

                   CS = 1552.



ID
911476
Banca
FCC
Órgão
DPE-SP
Ano
2013
Provas
Disciplina
Raciocínio Lógico
Assuntos

A sequência chamada a partir de agora de DS (dobro da soma) é: 1; 1; 4; 10; 28; 76; ... . Os dois primeiros termos da sequência DS são o número 1 e os termos seguintes são criados com a regra: dobro da soma dos dois termos imediatamente anteriores.
Assim, o terceiro termo é 4 pois 4 é o dobro da soma entre 1 e 1. O quarto termo é 10 porque 10 é o dobro da soma entre 4 e 1.
E a sequência segue dessa maneira ilimitadamente. Sabendo que o 8o termo de DS é 568 e o 10o termo de DS é 4240, o 9o termo dessa sequência é

Alternativas
Comentários
  • Pra quem preferir uma equação, ei-la:
    4240 = (568 + x) * 2
    4240 = 1136 + 2x
    2x = 4240 - 1136
    x = 3104 / 2
    x = 1552
  • De acordo com o enunciado, tem-se:

    a3 = 2*(a2 + a1)

    a4 = 2*(a3 + a2)

    a5 = 2*(a4 + a3)

    a6 = 2*(a5 + a4)

    ....

    an = 2*(an-1 + an-2)  termo geral


    Sendo assim, segundo o termo geral da sequência, tem-se:

    a10 = 2*(a9 + a8)

    4240 = 2 (a9 + 568)

    2 a9 = 3104

    a= 1552


    (Resposta B)



ID
922705
Banca
CETRO
Órgão
TJ-RS
Ano
2012
Provas
Disciplina
Raciocínio Lógico
Assuntos

Assinale a alternativa que apresenta o próximo termo da sequência abaixo.

7, 15, 24, 34, ...

Alternativas
Comentários
  • RESPOSTA: LETRA D.

    7, 15, 24, 34,  45

    De 7 para 15 aumentou 8;
    De 15 para 24 aumentou 9;
    De 24 para 34 aumentou 10;

    Então seguindo a sequência: De 34 aumenta 11 e se chega a 45.
  • 7+8=15
    15+9=24
    24+10=34
    34+11=45
    ...


  • De 7 a 15 são 8

    De 15 a 24 são 9
    De 24 a 34 são 10
    De 34 a 45 são 11
    Aumentam 1 em cada número aumentado anteriormente, ou seja, 34 + 11=45
    Letra d

ID
944401
Banca
VUNESP
Órgão
FUNDUNESP
Ano
2013
Provas
Disciplina
Raciocínio Lógico
Assuntos

Na sequência 1, 2, 1, 4, 2, 1, 8, 4, 2, 1, 16, 8, ... a diferença entre o 17.º e o 13.º termos é igual a

Alternativas
Comentários
  • Olá amigos do QC, cada questão que aparece...


     O nº 1 aparece sempre  na seguinte sequência, 1 pula uma casa, 1 pula duas casas, 1pula três casas, 1 e por aí vai;
    Logo depois do nº1 aparece outro número dobrando: 2, 4, 8, 16, 32, ...
    Mas esses números também têm uma sequência para aparecerem:
    O nº 2 aparece pula duas casas aparece de novo, pula mais três casas e aparece, pula mais quatro casas e aparece....
    O nº 4 aparece pula três casas aparece de novo, pula mais quatro casas e aparece, pula mais cinco casas e aparece....
    O nº 8 aparece pula quatro casas aparece de novo, pula mais cinco casas e aparece,......
    O nº 16 aparece, observe que ele pulará cinco casas antes de aparecer de novo, pois cada número que aparece, no primeiro pulo, sempre pula uma casa a mais que o antecessor e depois todos seguem pulando uma casa a mais que o primeiro pulo.
    Então ficará assim:
    1   2   1   4   2   1   8   4   2   1   16   8   4   2   1   32   16

    16 - 4 = 12

    Grande abraço e bons estudos.

  • Pessoal, para ficar melhor de visualizar é só fazer o seguinte:

    1,
    2, 1,
    4, 2, 1,
    8, 4, 2, 1,
    16, 8, 4, 2, 1
    32, 16, 8, 4, 2, 1

    Notem que o primeiro elemento da linha (n+1) é sempre o dobro do elemento da linha (n), e os elementos de cada linha são a metade do anterior até chegar no valor 1.
  • SIMPLE WAY

    Perceba que o próximo numero é encontrado pela soma da decomposição do número anterior, considerando a soma do intervalo entre 1 a 1:

    1, 2, 1, 4, 2, 1, 8, 4, 2, 1, 16, 8   

    1, 2, 1, 4, 2, 1, 8, 4, 2, 1, 16, 8, 

    1, 2, 1, 4, 2, 1, 8, 4, 2, 1, 16, 8,

    1, 2, 1, 4, 2, 1, 8, 4, 2, 1, 16, 8, 4, 2, 1    (termo grifado corresponde ao 13ª termo)

    1, 2, 1, 4, 2, 1, 8, 4, 2, 1, 16, 8, 4, 2, 1, 32, 16 ( termo grifado corresponde ao 17º termo)

     

    LOGO: 16 - 4 = 12   >>>> Gabarito E

  • Essa ordem dos números segue o resultado de potência na base 2, ou seja, o número 1 pode ser substituído por 2^0, o número 2 é 2^1, o número 4 é 2^2, o número 8 é 2^3 e o número 16 é 2^4.

    Assim, quando mudamos a ordem que está escrita no enunciado (1, 2, 1, 4, 2, 1, 8, 4, 2, 1, 16, 8...) pelas potências, percebe-se que o expoente tem um padrão que se repete, e fica: 2^0, 2^1, 2^0, 2^2, 2^1, 2^0, 2^3, 2^2, 2^1, 2^0, 2^4, 2^3.

    Agora fica mais fácil de visualizar o padrão repetido pelos expoentes, que tem a seguinte ordem: 0, 1, 0, 2, 1, 0, 3, 2, 1, 0, 4, 3, 2, 1, 0, 5, 4, 3, 2, 1, 0......

    Sabendo disso, percebe-se que o 13º número é 2^2, e o 17º é 2^4.

    Logo, 2^2 = 4

    2^4 = 16

    Fazendo a subtração, fica: 16 - 4 = 12

    Gabarito: E


ID
950908
Banca
FUNRIO
Órgão
DEPEN
Ano
2009
Provas
Disciplina
Raciocínio Lógico
Assuntos

Os conjuntos A, B e C possuem elementos em comum. As quantidades de elementos de todas as possíveis interseções definidas a partir desses conjuntos, juntamente com as quantidades dos elementos dos conjuntos A, B e C, formam uma Progressão Aritmética de sete termos de razão R não nula. Sabendo-se que a interseção dos três conjuntos possui R elementos, a quantidade de elementos pertencente à união dos conjuntos A, B e C é

Alternativas
Comentários
  • Como o enunciado faz alusão a conjuntos, então da Teoria dos Conjuntos podemos escrever:

    n(AUBUC) = n(A) + n(B) + n(C) – n(AB) – n(AC) – n(BC) + n(ABC)

    Note que temos aqui uma P.A. de 7 termos que não estão numa sequência certa. Podemos colocá-la em ordem crescente da seguinte maneira:

    O 1.° elemento (conjunto) dessa P.A. é a intersecção dos três conjuntos n(A∩B∩C), pois não há um termo (conjunto menor que esse). Então:

    P.A. = { n(A∩B∩C); n(A∩B); n(A∩C); n(B∩C); n(A); n(B), n(C)}

    As quantidades de elementos de todas as possíveis intersecções definidas a partir desses conjuntos, juntamente com as quantidades dos elementos dos conjuntos A, B e C, formam uma progressão aritmética de sete termos de razão R não nula.

    Fazendo n(B∩C) = x, teremos:
    {x – 3R; x – 2R; x – R; x; x + R; x + 2R, x + 3R}

    Como a intersecção dos três conjuntos possui R elementos, entâo:

    n(A∩B∩C) = R = x – 3R
    x – 3R = R
    x = R + 3R
    x = 4R, logo:

    {x – 3R; x – 2R; x – R; x; x + R; x + 2R, x + 3R} =

    {4R – 3R; 4R – 2R; 4R – R; 4R; 4R + R; 4R + 2R, 4R + 3R}=

    {R; 2R; 3R; 4R; 5R; 6R; 7R}= { n(A∩B∩C); n(A∩B); n(A∩C); n(B∩C); n(A); n(B), n(C)}, usando a fórmula, fica:

    n(AUBUC) = n(A) + n(B) + n(C) – n(A∩B) – n(A∩C) – n(B∩C) + n(A∩B∩C)

    n(AUBUC) = 5R + 6R + 7R – 2R – 3R – 4R + R
    n(AUBUC) = 18R – 9R + R
    n(AUBUC) = 9R + R
    n(AUBUC) = 10R, letra E
  • Seria melhor simplificar o raciocínio, não tem tempo na prova para fazer tanta conta: Faça a representação gráfica dos conjuntos (A,B e C), aquelas 3 bolinhas clássicas, na intercessão dos três coloque o valor R (Este pode ser o primeiro termo da PA), como o enunciado determinou, na intercessão de A com B fixe o segundo termo da PA 2R, bom este conjunto inclui o R da interceção então será apenas R, pois R+R = 2R, repita a operação para as outras duas intercessões, B com C e C com A, sempre somando uma razão (R), teremos R + R + 2R + 3R = 7R, daí só falta os conjutnos completos, e só somar os elementos já inseridos no gráfico de bolinhas e continuar a PA, vai notar que vai acresser R + R + R = 3R, daí 7R +3R = 10R.
  • Não há um método mais rápido para fazer isso? Eu não conseguiria perder todo esse tempo para resolver apenas uma questão na prova...


ID
955690
Banca
FUNRIO
Órgão
DEPEN
Ano
2009
Provas
Disciplina
Raciocínio Lógico
Assuntos

Os números naturais da seqüência X1, X2, X3, X4,...,XN seguem uma ordem lógica crescente. Sabendo que a soma e o produto dos três primeiros termos dessa seqüência valem, respectivamente, 12 e 48, e que a soma e o produto dos segundo, terceiro e quarto termos valem 18 e 192, respectivamente, o centésimo termo dessa seqüência é igual a:

Alternativas
Comentários
  • A sequência diz que os três primeiros números tem soma 12 e produto 48. Os únicos números que satisfazem essas duas condições são 2, 4 e 6. 
    Então, já temos que o segundo e o terceiro números são 4 e 6. Sabe-se também que a soma do segundo com o terceiro e com o quarto dá 18, portanto 4 + 6 + 4º = 18, o quarto número é 8. 
    Confirmemos: 4 x 6 x 8 = 192. Com efeito. 
    Portanto, a sequência diz que o valor do número equivale a duas vezes a sua posição. O 1º número vale 2, o 2º vale 4, e, por lógica... o 100º vale 200. 

    Resposta B. 

    Espero ter ajudado. =].

  • x1 ,x2,x3,x4           

    2+4+6= 12

    2x4x6=48

                              x2,x3,x4

                              4+6+8=18

                             4x6x8=192

                            192 +2 = 200 (A sequência diz que o valor do número equivale a duas vezes a sua posição).

     

    Gabarito letra b

     

     

     

     

     

  • Bora:]

     

    1º tu tens de achar os respectivos valores de: X1, X2, X3, X4,...,XN para depois entender aplicar a formula de PA:

    Únicos numerais que somado dão 12 e o produto (multiplicação) 48 são respectivamente 2,4,6,logo tu substitui: x1=2,x2=4,x3=6.

    2º ele pede  a soma e o produto do,2º,3º e 4º termo,se tu reparar são 2,4,6... é uma PA com razão 2,logo o 4 nº será 8.

     

    Descoberto a razão é só jogar na formula:An=A1(an-1).r    ....   A100=2(100-1).2 

    Resultado =200 GABA B

    Se me equivoquei,me corrijam!

     

  • SOMA (+)

    PRODUTO (X)

    PA:

    an = a1 + (n -1) x r


ID
958669
Banca
CEPERJ
Órgão
SEFAZ-RJ
Ano
2010
Provas
Disciplina
Raciocínio Lógico
Assuntos

Observe a seqüência de números abaixo.

3 2 1 0 5 4 3 2 1 0 5 4 3 2 1 0 5 4 3 2 1 0 5 4 ...

O 100º número dessa seqüência é:

Alternativas
Comentários
  • 1  fila de  10  =   321,054,321,054,321,054,321,054,321,054  =  30 numeros
    2  fila de 10  =                                       .......                                       =   60    ''
    3  fileira  10  =                                      ........                                        =  90     ''
    agora  10 ultimos  numeros    321,054,321, 0
  • 3210= 4 numeros +543210=6 numeros>> 6.x+4=100, 6x=96, x=16
    entao deixando fixo os quatro primeiros numeros iremos repetir 16 vezes a sequencia de 6 numeros, como a divisao deu um numero exato, indica q se repete ate o final com o ultimo numero igual a zero.
  • 24 x 4 = 96 (são exibidos 24 números, dai o 96º será o 4, logo o 100º é o 0)
  • 100 = centésima posição
    6 = quantidade de números de cada sequencia

    100 dividido por 6 = 16 com resto 4

    Então, pega a quarta posição da sequencia...

    3 2 1 0 5 4

    Resposta = A
  • 321 054 321 054 321 054 321 054

    321 054 321 054

    Dividi 100/3 = 33 sobra 1=34ª posição